Jump to content

ناصر الأثر

الأعضاء
  • Content Count

    40
  • Joined

  • Last visited

Everything posted by ناصر الأثر

  1. هذه براهين الطيار نادر جنيد على أن الأرض لا تدور أخذت من موقعه http://www.naderjneid.com ولا فوتكم الفيديو على الرابط http://www.naderjneid.com/files/earth.rar من خلال 21 برهاناً علمياً من قوانين الطيران والفيزياء والميكانيك، يثبت الكابتن نادر جنيد (كابتن على الطائرات الأمريكية) أنَّ الأرض ثابتة لا تدور حول نفسها ولا حول الشمس بمدار إهليليجي، ويؤكِّد أنَّ الليل والنهار يتشكَّلان نتيجة دوران الشمس حول الأرض، وتتشكَّل الفصول الأربعة نتيجة حركة الشمس اللولبية بين مدار السرطان ومدار الجدي أثناء دوران الشمس حول الأرض، ليدحض النظرية الحالية التي يسير عليها العالم وتُبنى على أساسها العلوم، والتي تُدرَّس في كل جامعات العالم، بالإضافة إلى الأبحاث الفضائية القائمة، وكذلك النظرية التي جاء بها العالِم (كوبر نيكوس) والتي تقول: «إنَّ الشمس ثابتة، وإنَّ الأرض تدور حول نفسها 360 درجة خلال 33 ساعة و56 دقيقة و4 ثوانٍ، وتنتقل الأرض حول الشمس كل يوم 2450000 كيلو متر خلال 3 دقائق و56 ثانية، لتكتمل ساعات اليوم (24 ساعة) ولتكتمل الدائرة الإهليليجية 360 درجة خلال 365 يوماً وربع، وبهاتين الدورتين يتشكَّل الليل والنهار، وتتشكَّل الفصول الأربعة.. وإنَّ القمر يدور مرة حول الأرض ومرة حول نفسه كلّ 28 يوماً، بحيث يبعد القمر عن الشمس 12 درجة يومياً».. واستدلَّ على انتقال الأرض حول الشمس بتغيير أبراج التنجيم، ثم طبع كتاباً حول النظرية من دون أن يحمل اسمه خشية بطش الكنيسة، لأنه خالف ما درجت عليه.. ليأتي بعده العالم الإيطالي (غاليلو)، مؤكِّداً صحة نظرية «كوبر نيكوس» ثم تراجع عن نظريته لينجو من حكم الإعدام.. ثم جاء العالم (كبلر) ووضع قوانين تعرف بقوانين كبلر في حركة الأجسام في الحقول المركزية وأصبحت من المُسلَّمات العلمية.. واليوم، يثبت الكابتن جنيد، أنَّ كلَّ ذلك عارٍ عن الصحة، من خلال بحثه في حساب زوايا الفجر وإثبات بداية الشهر الهجري من خلال الملاحة الجوية، كما أنه يثبت صحة ما يقوله بـ 19 برهاناً علمياً البرهان الأول: «إذا كانت جاذبية الشمس أقوى من جاذبية القمر، لظهرت تأثيرات جاذبية الشمس على الأرض، مثل المد والجزر؛ حيث إننا نلاحظ حدوث المد والجزر عندما يكون القمر عمودياً على الأرض, وليست الشمس». البرهان الثاني: «فسَّر علماء الفلك حركة الأرض والقمر حسب نظرية كوبر نيكوس بطريقة حسابية خيالية وغريبة جداً؛ حيث جعلوا الشمس ثابتة، وللمحافظة على مدة طول اليوم 24 ساعة, قسَّموا مدة اليوم إلى قسمين: القسم الأول تدور فيه الأرض حول نفسها 360 درجة خلال 23 ساعة و56 دقيقة و4 ثوانٍ. والقسم الثاني، تنتقل الأرض فيه حول الشمس مسافة 2450000 كيلومتر خلال 3 دقائق و56 ثانية، ولو لم يجعل علماء الفلك هذا الرقم الخيالي (بأنَّ الأرض تنتقل حول الشمس وتدور الأرض أيضاً حول نفسها 0.98 من الدرجة) لكان عدد أيام السنة الميلادية 366.25 يوم». البرهان الثالث: «إذا كانت الأرض تدور حول نفسها، فإنَّ الولايات المتحدة الأمريكية- حسب قوانين الملاحة الجوية- كذبت على العالم بأنها هبطت على سطح القمر، وأنَّ الصور التي أرسلتها هي مِن صنع مدينة هوليوود السينمائية.. أما إذا كانت الأرض ثابتة، فإنَّ أمريكا قد هبطت فعلاً على سطح القمر». البرهان الرابع: «إذا كانت الأرض تنتقل حول الشمس والقمر يدور حول الأرض، ونتيجة دوران القمر حول الأرض، يتغيَّر موقع القمر بالنسبة إلى الأرض، ومن الثابت أنَّ القمر والأقمار الصناعية لا تحتوي على قوة ذاتية تستطيع أن تزيد أو تنقص من سرعتها». البرهان الخامس: «حسب نظرية كوبر نيكوس، فإنَّ الغلاف الجوي يُعدُّ قطعة من الأرض, وبالتالي، فإنَّ الغلاف الجوي والأرض يدوران مع بعضهما حول مركز الأرض وحول الشمس، حيث إنَّ أقصى سرعة لمركبة الفضاء 27,000 كيلومتر في الساعة، ومعدل سرعة دوران الأرض حول الشمس 100 ألف كيلومتر في الساعة, لذلك ستجد مركبة الفضاء صعوبة كبيرة عند العودة إلى الأرض، مع العلم بأنَّ مركبات الفضاء تخرج وتعود إلى الأرض عبر الغلاف الجوي بسهولة، بدليل أنَّ الرحلة إلى القمر استغرقت ستة أيام». البرهان السادس: «للقمر ثلاث دورات؛ فالدورة الأولى تكون حول نفسه، والدورة الثانية تكون حول الأرض, والدورة الثالثة هي الدورة الظاهرية، أو منازل القمر، ولو أنَّ الشمس ثابتة والقمر يدور حول الأرض, فحسب قوانين المرايا وانعكاس الضوء، يجب أن تظهر جميع منازل القمر من الهلال إلى البدر ثم المحاق يومياً، وليس مرة واحدة في الشهر». البرهان السابع: «لو أنَّ الأرض تدور حول نفسها بسرعة زاوية 15 درجة في الساعة, والقمر يدور حول الأرض بسرعة 14.5 درجة في الساعة, فيكون فرق السرعة هو الزاوية بين الأرض والقمر (0.5 درجة في الساعة)، وبالتالي سيكون فرق الزاوية بين القمر والأرض 6 درجات فقط، خلال 12 ساعة, وليس 180 درجة كما في الواقع». البرهان الثامن: «يوجد في كلِّ ثانية أربعة مواقع للشمس بالنسبة إلى الأرض وهي: شروق الشمس وغروب الشمس وشمس منتصف النهار وشمس منتصف الليل، حيث تمَّ تحميل أوقات شمس منتصف النهار على أوراق ميليمترية بين مدار السرطان ومدار الجدي، وتبيَّن خلال سنة ميلادية أنَّ حركة الشمس بين مدار السرطان ومدار الجدي بالنسبة إلى وقت زوال الشمس، قد رسمت رقم 8 بالإنكليزية.. فلو كانت الأرض تنتقل حول الشمس بشكل إهليليجي لحدوث الفصول الأربعة، فحسب القوانين الميكانيكية، يجب أن ترسم الشمس على الأرض خلال سنة ميلادية شكلا إهليليجياً». البرهان التاسع: «إنَّ الأقمار الصناعية التلفزيونية ثابتة البعد عن الأرض، ولو تحرَّك القمر الصناعي من مكانه بمقدار متر واحد، فإنَّ الجاذبية الأرضية غير قادرة على إعادته إلى مكانه، فكيف إذا كانت الأرض تتحرَّك حول الشمس بسرعة100 ألف كيلومتر في الساعة». البرهان العاشر: «يوجد في السيارات التي تنقل المياه حواجز في خزاناتها كمخمّدات لحركة المياه، للمحافظة على مركز توازنها، لذلك لو كانت الأرض تنتقل حول الشمس بمدار إهليليجي فسوف تحدث حركة مياه عنيفة، وستكون أعنف من تسو نامي». البرهان الحادي عشر: «الطائرات عندما تكون على الأرض تُعدُّ قطعة من الأرض، وعند طيرانها في الغلاف الجوي تقوم بإزاحة الهواء من أمامها، ولو تحرَّكت الأرض مثل الطائرة في الغلاف الجوي، لأزاحت الهواء من أمامها، وتجاوزت الغلاف الجوي في ثلاث ثوانٍ ونصف». البرهان الثاني عشر: «الغلاف الجوي المحيط بالأرض جسم غازي ولا نستطيع الإمساك به، إلا إذا تمَّ حصره. ولو كان الغلاف الجوي ينتقل مع الأرض، لكانت سماكة الغلاف الجوي أمام حركة الأرض حول الشمس أقل من سماكته خلف الأرض, وفي الواقع، فإنَّ سماكة الغلاف الجوي متساوية ومتجانسة» . البرهان الثالث عشر: «عندما تتجاوز الطائرة سرعة 1200 كيلومتر في الساعة, فإنَّ الطائرة تكون قد اخترقت جدار الصوت، وسُمِعَ صوت انفجار قوي، ولو كانت الأرض تنتقل حول الشمس بسرعة 100000 كيلو متر في الساعة، لسمعت أصواتاً قوية عند اختراق الأرض جدار الصوت». البرهان الرابع عشر: «لو كانت الجاذبية الأرضية قادرة على إمساك وتثبيت الغلاف الجوي أثناء دورانها حول نفسها أو دورانها حول الشمس، لمنعت حدوث الرياح، خاصة رياح الأعاصير من الدرجة الخامسة التي تصل سرعتها إلى 250 كيلو متراً في الساعة». البرهان الخامس عشر: «لو كانت الأرض تنتقل حول الشمس بسرعة 100 ألف كيلو متر في الساعة، لارتفعت درجة حرارة الأرض وتبخَّر الماء واحترقت الأرض، كما يحدث مع الأجرام السماوية إذا دخلت الغلاف الجوي للأرض». البرهان السادس عشر: «لو كانت الأرض تنتقل حول الشمس بسرعة 100 ألف كيلو متر في الساعة، لتمَّت مشاهدة مذنب للأرض من قِبل رجال الفضاء الذين طاروا خارج الغلاف الجوي؛ ولصوَّرت الأقمار الصناعية التي طارت لمسافات بعيدة حركة انتقال الأرض حول الشمس». البرهان السابع عشر: «لو كانت الأرض تدور حول نفسها بسرعة 1667 كم/ساعة عند خط الاستواء وكان وزنك80 كيلوغراماً، فسوف يزداد وزنك فوق القطب بسبب تناقص سرعة دوران الأرض وتناقص القوة النابذة، وفي الواقع، إنَّ وزنك يتغيَّر بين القطب وخط الاستواء بمقدار غرامات فقط». البرهان الثامن عشر: «من خلال الطيران في الأجواء العالية, كلما ارتفعنا عن الأرض تزداد سرعة الرياح ويصبح اتجاهها غرباً، ولهذا السبب تزيد مدة الطيران باتجاه الغرب وتنقص مدة الطيران باتجاه الشرق. ولو أنَّ الأرض تدور حول نفسها، لكانت مدة الطيران من دمشق إلى لوس انجلوس أقل من لوس انجلوس إلى دمشق، بسبب تعاكس الحركتين». البرهان التاسع عشر: «إذا كانت الأرض تدور حول نفسها باتجاه عكس عقارب الساعة, فيجب أن تكون سرعة واتجاه دوران الأقمار التلفزيونية عكس عقارب الساعة.. وفي الواقع، يتمُّ القمر الصناعي باتجاه عقارب الساعة بسرعة تقريبية مساوية لسرعة المدار الذي يدور عكس عقارب الساعة، ومن ثم يحرّر القمر الصناعي من الصاروخ، وتعمل محركات صغيرة لتجعل محصلة سرعة دوران الفلك ودوران القمر صفراً»... البرهان العشرون : تتم عملية هبوط الطائرة على مهبط الطائرات , بأن يوجه الطيار مقدمة الطائرة على منطقة الهبوط ولا يأخذ الطيار في حساباته دوران الأرض حول نفسها ولا حول الشمس . البرهان الواحد والعشرون : الأرض تدور حول نفسها وحول الشمس باتجاه عكس عقارب الساعة لكان ظل حركة كسوف الشمس باتجاه الغرب وليس باتجاه الشرق وبمعنى آخر لو أن الأرض تدور باتجاه عكس عقارب وبدأ الكسوف فوق أفريقيا فان الكسوف سوف ينتهي فوق أمريكا بدلا من أسيا وهذا دليل على خطأ نظرية كوبرنيكوس . وٍبهذه البراهين العلمية الواحد والعشرين أثبت نظرية بأن : الأرض ثابتة وجامدة في مكانها لا تدور حول نفسها ولا حول الشمس
  2. هذا هو عنوان كتاب جديد للشيخ محمد الإمام صدر عن دار الآثار بصنعاء وهو متومفر في دار التدمرية بالرياض وفي درب الأتراك في القاهرة
  3. يقوم بعرض التقويم الهجري (مع/ أو بدلا) من التقوم الميلادي التفاصيل على الرابطhttp://www.msofficeword.net/bb/viewtopic.php?f=18&t=38
  4. مميزات سكربت التاريخ الهجري HejriDate 1.0 يستعرض التاريخ الهجري من متغير التاريخ Unix timestmp وكذلك يقوم بتعريب التاريخ الميلادي. يستعرض التاريخ الهجري باللغة الإنجليزية. يحول التاريخ الهجري غلى unix timestamp مما يمكن بعد ذلك تخزينه في قواعد البيانات أو معاملته بدوال php التي تتعامل مع التاريخ. يعتمد التقويم الهجري الاصطلاحي الذي اعتمده الفلكيون القدامى، ويعتمد الكبس في السنة 15 لا 16. ملف السكربت هو hejridate.php يجب تضمين الملف في الصفحة المراد تشغيل دوال السكربت فيها مثلاً: كـود PHP include "hejridate.php"; دوال السكربت: 1- الدالة adate تعمل نفس عمل الدالة date في لغة php ، راجع ,وثائق عمل الدالة date في موقع php مع الاختلافات التالية: 1- تم إضافة أحرف تنسيق خاصة بعرض التاريخ الهجري وهي: _j اليوم بدون أصفار دليلية (1-30) _d يوم مع أصفار دليلية( 01-30) _z رقم اليوم في السنة بداية من الصفر (0-354) _M,_F اسم الشهر (محرم، صفر،..) _m رقم الشهر مع أصفار دليلية (01-12) _n رقم الشهر بدون أصفار دليلية (1-12) _t عدد الأيام في الشهر (29-30) _L السنة كبيسة أم لا، 1=كبيسة، 0=ليست كبيسة _Y السنة رقم كامل (1430) _y السنة من رقمين (30) 2- تم تحوير عمل أحرف التنسيق التالية الخاصة بعرض التاريخ الميلادي ليتم العرض باللغة العربية: l,D اسم يوم الإسبوع F اسماء الأشهر السريانية (كانون، شباط...) M اسماء الأشهر (تسمية إنجليزية)يناير ، فبراير...) a ,A صباحا ومساء للوقت 3- التنسيق الافتراضي هو '_j _M _Yهـ' فيظهر بشكل( 5 جمادى الأولى 1430هـ) 4- التاريخ الافتراضي هو تاريخ اليوم. مثال: كـود PHP adate("_j _M _Yهـ الموافق j M (F) Yم"); يعرض النص التالي: 3 جمادى الأولى 1430هـ الموافق 28 أبريل(نيسان) 2009 م ================================ الدالة الثانية:edate تعمل نفس عمل الدالة date في لغة php ، راجع ,وثائق عمل الدالة date في موقع php do same of (date) but if can view hejri date ***********hejridate format letters _j Day of the month without leading zeros _d Day of the month, 2 digits with leading zeros _z The day of the year (starting from 0) _M, _F A full textual representation of a month, such as Ramadan _m Numeric representation of a month, with leading zeros _n Numeric representation of a month, without leading zeros _t Number of days in the given month _L Whether it's a leap year, 1 if it is a leap year, 0 otherwise _Y A full numeric representation of a year, 4 digits _y A two digit representation of a year EXample: كـود PHP edate("_j _M _Y"); shows: 3 jumada al Oula 1430 =================================== الدالة الثالثة:hejri2time يحول التاريخ الهجري إلى timestamp يكون التاريخ المدخل بترتيب اليوم ثم الشهر ثم السنة يمكن استخدام الرموز التالية في الفصل بين أجزاء التاريخ - / \ . Convert text Hejridate to Unix timestamp the hejri date must be in order d m y you can use these seprators between date parts - / \ . والفائدة منها أنك تستطيع بعد تحويل التاريخ الهجري إلى timestamp تخزينه في قاعدة بيانات أو إجراء عمل دوال php الخاصة بالتاريخ فيه مثال: كـود PHP hejri2time("29-1-1430"); يحول يوم 29 محرم 1430 إلى متغير timestamp برمجة: أبي الحارث الحاسوبي، مشرف موقع صفحات العلامة مقبل بن هادي الوادعي رحمه الله الإصدار: 1.0 لا تخفي اسم المبرمج والحقوق حتى لا تكن متشبعًا بما لم تعطَ. hejridate.zip
  5. بسم الله الرحمن الرحيم ونحب أن نبشر الأخوة بقرب صدور كتاب جديد حول نظريات نشوء الكون ونظرية دوران الأرض للعلامة ريحانة اليمن الشيخ المجاهد محمد بن عبدالله الإمام القائم على مركزه العلمي بمعبر-اليمن وهو بعنوان نقد النظريات الكونية وسيكون بإذن الله حسب معلومات متوفرًا في الأوساق خلال ستة أشهر - نشر دار الآثار بصنعاء. وحسب اطلاعي على بعض المواضع من مسودة الكتاب فإنه سيشكل ثورة علمية دليلية ساحقة للنظريات الفلكية المصادمة للأدلة الحسية والنقلية والكتاب ربما يكون في 400 صفحة
  6. هذا هو الإثبات الرياضي لنظرية 2=1 س=ص س^2=س ص /بضرب الطرفين ×س س^2-ص^2= س ص - ص^2 / بطرح ص^2 من الطرفين (س-ص)(س+ص)=ص(س-ص) / بتحليل الطرفين س+ص=ص /بقسمة الطرفين ÷ (س-ص) ص+ص=ص /بإبدال س بقيمتها المفترضة من بداية المسألة 2ص=ص 2=1 /بقسمة الطرفين ÷ ص إذن هذه نظرية تضاف إلى النظريات الرياضية والفلكية 2=1 ؟؟!!!!
  7. ناصر الأثر

    نقد النظريات الكونية

    صدر الكتاب باسم نقض النظريات الكونية متوفر في السعودية في دار التدمرية بالرياض متوفر في مصر في درب الأتراك جوار جامع الأزهر
  8. نريد من الأخوة أهل الفلك أن يتحفونا بالأدلة الحسية على دوران الأرض، خصوصًا إني اعتبر أن هذا الدوران ما هو إلا محض خرافة كوبرنيكوسية نابعة عن تعظيم وعبادة للشمس
  9. ناصر الأثر

    إثبات أن 2=1

    نشكرك جميعًا على المشاركة والأمر كما قال الأخوة (بسمة فمن بعدها) وانتبهوا في المرة القادمة
  10. ناصر الأثر

    دوران الأرض خرافة أم حقيقة

    نشكر جميع الأخوة الذين ساهموا في إثراء هذا الموضوع، والكفة الراجحة إلى الآن مع من قال بثبوت الأرض وجريان الشمس، وللأسف لم يأت أصحاب القول بدوران الأرض بدليل حسي واحد ولكن هنجمة وإرهاب فكري. ولا زلنا نطالب بالأدلة الحسية على دوران الأرض حول الشمس، أما الهنجمة بأن هذا قول كل أهل الأرض أو قول علماء الفلك، اتركوها فإنها منتنة نريد براهين... أدلة محسوسة واعلموا أن عقولنا ليس بيد الغرب وأننا أطوع الناس للحجج والبراهين
  11. نشكر الأخ ولد جدة على هذه المواضيع الجذابة المثمرة المكللة بالحجج والبراهين التي لا تقبل الجدل ونقول جزاك الله خيرًا
  12. ناصر الأثر

    دوران الأرض خرافة أم حقيقة

    بسم الله الرحمن الرحيم نشكر الأخوة الذين شاركوا في هذا الموضوع مشاركات علمية، ونخص منهم الأخوة الذن نقلوا الأدلة الشرعية وفتاوى العلماء في هذا الموضوع. ونحب أن نبشر الأخوة بقرب صدور كتاب جديد حول هذا الموضوع، وحول نظريات نشوء الكون للعلامة ريحانة اليمن الشيخ المجاهد محمد بن عبدالله الإمام القائم على مركزه العلمي بمعبر-اليمن وهو بعنوان نقد النظريات الكونية وسيكون بإذن الله حسب معلومات متوفرًا في الأوساق خلال ستة أشهر. ونحيل الأخوة على أهم المراجع التي تكلمت على هذا الموضوع : 1- الأدلة الحسية والنقلية على ثبات الأرض ودوران الشمس/ العلامة المجدد عبدالعزيز بن عبدالله بن باز رحمه الله مفتي المملكة العربية السعودية ورئيس هيئة كبار العلماء. 2-الصواعق الشديدة على أصحاب الهيئة الجديدة/ للعلامة حمود التويجري والكتاب بتقديم الشيخ محمد بن إبراهيم آل الشيخ مفتي المملكة العربية السعودية سابقًا، وبتقديم الشيخ عبدالله الحميد إمام المسجد الحرام. 3- كتاب هداية الحيران في مسألة الدوران-للشيخ عبدالكريم الحميد. 4- كتاب المورد الزلال فيما انقد على تفسير الظلال- للشيخ عبدالله الدويش رحمه الله. ومن آخرها صدورًا بإذن الله كتاب الشيخ محمد الإمام المرتقب. وجزاكم لله خيرًا.
  13. هذا الكتاب يفيدك بإذن الله ولو تقوم بترجمته فهذا ممتاز primer.pdf
  14. هذا الرابط يضاف إلى من نفى وصلوا الأمريكان إلى القمر http://www.geocentricity.com/ba1/no90/gotomoon.html
  15. ناصر الأثر

    دوران الأرض خرافة أم حقيقة

    كلام الأخ دافينشي طيب جدًا، وقريبًا إن شاء الله سننشر موضوعاً عن الموقف الشرعي من نظرية دوران الأرض، وكتاب قصة الخلق عليه بعض الملاحظات ولكنه في الجملة طيب، وللأسف بعض الأخوة الذين علقوا على هذا المقال مثلهم كمثل مشجعي الكرة، يا إخوان نحن في حوار علمي بحت ولسنا في ملعب كرة هنا يشجع هذا الفريق وهذا يشجع ذاك الفريق، ونحن نعف ألسنتا واقلامنا عنهم ويكفيهم قول الشاعر: لو كل كلب عوى ألقمته حجرًا # # # كان الحصى كل مثقال بدينار يا أخي إذا كنت ترى الموضوع وصاحبه أحمقان فاربأ بنفسك عن الحمقى وابتغ السلامة لدينك وعرضك. ونحن نناقش الأخوة الأفاضل الذين يرون بعلم وفهم مثل الأخ: الجعيدي، والأخ محمد مجدي والأخ المستور وغيرهم من الذين أتوا بأدلة أو إيردات على ثبوت الأرض ومركزيتها للكون، أما مشجعي الكرة فمكانهم الملاعب. ونشكر الأخ محمد مجدي على تعقيبه واللطيف ونرجو من أن يدخل إلى صميم المناقشة. وهذا الرابط متعلق بهذا الموضوع http://qasweb.org/qasforum/index.php?showtopic=1953
  16. اسم الجمعية: ASSOCIATION FOR BIBLICAL ASTRONOMY الموقع: http://www.geocentricity.com/ أهم النشاطات: إثبات مركزية الأرض للكون بالأدلة التجريبية، ونشر الكتب والمجلات الدورية المدافعة عن هذه النظرية، نقض النظرية الكوبرنيكوسية والنظرة الداروينية لنشوء الكون التي يتبناها علم الفلك الحديث عنوان الجمعية: أوهايو الولايات المتحدة
  17. وكنت أظن أن هذا المناقشة خاصة بأدلة التجريبة فقط ثم ظهر لي أنها تعرضت بالإضافة إلى ذلك لموقف الكنيسة وبعض النصوص من الإنجيل في هذا الموضوع، وعلى كل فهذا المناقشة من أحسن ما رأيت في نسف الاعتقاد السائد بأن هناك دليلاً تجريبيًا على دوران الأرض وقد تم نشر هذه المناقشة على شكل ملف أكروبات (pdf) على هذا الرابط http://qasweb.org/qasforum/index.php?act=A...pe=post&id=5804 وهو الأفضل من أجل الطباعة، وقد استخلصت النص المنشور هاهنا من الملف الأكروبات والملف يحتوي على صور توضيحية لم أستطع نشرها هنا فيمكن أن ينظرها الأخوة في ذاك الرابط المشار إليه. وملفات pdf تحتاج منك لقراءتها إلى برنامج adobe reader وهو منشور في موقع شركة adobe مجاناً
  18. مناظرة ممتعة حول دوران الأرض وطرح الأسس العلمية لنظرية ثبوث الأرض ومركزيتها للكون إلى من يعتد أن هناك دليلاً تجريبيا على دوران الأرض هذه المناظرة نقلتها كما هي باللغة الإنجليزية ونرجو من ممن لديه مقدرة على الترجمة ترجمتها لما فيها من فائدة: About scientific (& theological) aspects of Geocentricity – the view whose system has a non-orbiting & non-rotating earth at exact center of the universe Introduction It has been about 4 centuries since Fr. Nicolaus Copernicus suggested a paradigm-shift from Geocentrism to Heliocentrism [now A-centrism] as an explanation of “how the heavens go”. We believe those who suggest that Sacred Scripture tells us only how to go to heaven & not how the heavens go must be dismissed by the fact that Scripture makes a number of references to “how the heavens go” & a number of those support Geocentrism. It is also a matter of Church history that because of those references, the Fathers & Councils of the Church – prior to the aftermath of the Church’s Galileo affair – were geocentrists. Church decrees against earth movement have never been abrogated or reversed by any ecclesiastical pronouncement, although opinions have not been in short supply. In 2002 ACs [a-centrists – persons who believe earth moves] were invited to present what they consider to be scientific proof that the earth moves – orbits &-or rotates. The following pages allow the reader to view ACs presenting their arguments, along with a GEO [geocentrist – person who believes earth does not move & is the center of the universe] refuting those arguments. The GEO refutations reveal many of the scientific aspects of the geocentricity model. Those scientific aspects are difficult to find, so this is a rare opportunity to learn about them. Some theological aspects are also clarified by the GEO. The GEO documents that the very scientists, whose work is generally cited as “the proof” for the AC position, admit that actual proof does not exist for earth rotating &-or orbiting. The reader will note that in the absence of scientific proof that the earth moves, the frustrated AC often resorts to such emotional terms as “absurd”, “ridiculous”, “incredible”, “irrational” & “illogical” when it is pointed out to him that mathematical constructs for arriving at a desired assumption do not constitute scientific proof of what is moving in the relative motion framework of the universe in which we live. In the 20th century the cosmology paradigm shifted from Heliocentrism [Galileo’s belief] to Acentrism [a universe without center], but in both systems for the cosmos, the earth is alleged to be moving – contrary to what Scripture tells us. Heliocentrism contended that the Sun did not move & was at the center of the universe, whereas A-centrism now admits that Sun is moving [all over the place], and also alleges there is no center of the universe. Although average readers may not be able to follow some [or much] of the technical information in these discussions, they will at least get to better understand that earth movement is not as “cut & dried” as the global scientific community would have us believe. You be the judge. Scientific aspects of Geocentricity are in very short supply here in the 21st century, due mainly to the current world-view espousing A-centricity & Evolutionism, so here is a rare opportunity to learn something about “the rest of the story”. We pray there will be a scientific renaissance in this century. The following discussions are a move toward such openness & honesty in science. The Webmasters, origins@ev1.net * * * * * Contents About satellites ..........................................................4 Some basic physics ....................................................9 Proof lacking for rotation & orbiting .......................10 More on satellites.....................................................14 Earth at the center .................................................... 27 More on satellites in claim to prove earth moves ....28 Scripture and the Church .........................................46 Inertial field & more claims that earth moves .........53 More on the Church’s position ................................62 Recap of the debate.................................................. 69 Pope John-Paul II reading PAS speech on Galileo .. 70 More on satellites.....................................................83 Some NASA flight patterns ......................................87 Drag forces in space.................................................91 Chandra, Marisat-3 & others .................................. 93 The pendulum & parallax ........................................94 Stars & the speed of light.........................................95 Star shift...................................................................96 Philosophy of science; whose errors? ...................... 96 Some problems for earth-movers...........................108 The rest of the Einstein affair.................................113 About parallax........................................................117 More about satellites & oscillation ........................118 Chandra again........................................................121 About NASA’s satellite tracking system ................123 More about inertial effect on satellites...................126 More on the ways of Chandra ...............................134 More on satellites...................................................136 More on Coriolis & the pendulum.........................137 Shuttlecraft.............................................................148 Starlight, parallax & aberration..............................149 Gravity & where God “hung” earth.......................151 Some planetary motions.........................................155 Sagnac & an Einstein cover-up..............................156 More on gravity .....................................................157 Church position on geocentricity ...........................157 Geocentricity in the 17th c. Other matters now .....161 What science is (& is not) about ............................162 Laws of physics & geocentricity............................164 Model commonly accepted for geocentricity ........168 vs J L Case ............................................................170 Newton, space travel, planetary orbits, etc ............205 vs Hutton Gibson – space probes ...........................207 vs W Savina – sun’s travel & speed.......................217 vs K Cole – sun’s movement ................................218 vs M Healy – NASA, planetary loci, earth seasons....220 Total file [222pp] size is 1.1 MB, available by email or on a data-CD, in either MS Word .doc or .pdf format. origins@ev1.net Following are attempts by several “a-centrists” to give “scientific proof” that the earth moves [rotates &-or orbits] AC Letters designate the a-centrist – a person who believes the earth to be moving [orbiting the sun &-or rotating] in an a-centric universe whose center is neither the earth nor the sun. GEO Letters designate the geocentrist – a person who believes the earth to be the exact center of the universe and that it is not moving [neither orbiting nor rotating]. About satellites AC In your article on the rotation of the earth you said, The GPS satellite is stationary over the earth because the earth is stationary. The GPS satellite doesn’t need adjusting very often because there are few things that interfere with its stationary position at 22,236 miles above the earth. It doesn’t need to use large amounts of fuel, because it doesn’t move. What I’d like to know is how it became stationary. The people who launched the GPS satellites believed that the earth rotates, and so they placed their satellites into an orbit at which they circle the earth once every day, believing that this would result in a geosynchronous orbit. But if, as you say, those satellites are currently not moving at all, if they somehow went from 6,856 mph to 0 mph (without anybody noticing), what stopped them? ======================== GEO I will answer your question from several angles, the Third of which is the more complete. First: On the basis of “proof” required of the challenge, the question you propose does not “prove” the case of heliocentrism. It merely poses the hypothetical problem of how a GPS (satellites of both the Global Positioning System and the Global TV Satellites) creator or administrator could properly send a satellite into position if he is doing so under the presumption that the earth is rotating. I will answer that question more in the “Third” answer. But for now, your question must ASSUME that the earth is rotating and/or ASSUME that the GPS is moving at 6,800 mph in order for your question to reach the level of proof you are requiring from it. But in either assumption, you are begging the question. What you will need as proof for your claim is direct evidence from the GPS creators or administrators that the GPS is actually moving at 6800 mph (which it must if the earth is rotating). Once that is proven, then you can make a good case that the earth is in rotation. My assertion that the GPS would HAVE to be moving at 6800 is merely the physical requirement for it to be geosynchronous with the earth, if, as it is supposed, the earth is in 24 hour rotation. And I must caution that proof cannot be merely a statement from the GPS creators and administrators that the GPS MUST be moving at 6,800 since it must keep up with the earth’s rotation, for that is also begging the question, being that it assumes one unproven fact in order to prove another. The evidence that the GPS is moving at 6,800 mph must be an independent, technical and verifiable source of information apart from the mere opinion of the creators or administrators. But here’s the rub: The only way one could prove that a GPS satellite is moving at 6,800 is to first prove there exists a stationary inertial framework against which to calibrate a speed of 6,800 mph. Since in a heliocentric system there is no absolute inertial framework due to the fact that heliocentric theory posits that all the heavenly bodies are in relative motion, then there is no absolute inertial framework to measure a speed of 6,800 mph. Also, let me make a correction to the above. I have used the designation GPS for the satellites of both the Global Positioning System and the Global TV Satellites. The latter are the ones at 22,000 miles up, and do not move relative to the earth. The former are at about 11,000 miles up, and move relative to the earth in 12-hour “orbits.” To do so, they would have to be traveling 7,800 mph if they orbit with the “rotation” of the earth, and travel at about 500mph against the “rotation” of the earth. Also, it is worthy to note that the GPS system works on the basis of triangulation. In other words, at least three (usually four) GPS satellites must simultaneously send/receive a signal to a precise location on earth so that the computer can calculate distance and location for any given object by using Pythagorean proportions. But this only strengthens our case against the GPS having to be moving at supersonic speeds, since the difficulty of having three satellites maintaining that high velocity, along with the necessary course corrections requiring the constant speeding up or slowing down of the GPS, would be near impossible. An article from Physics Today (“Relativity and the Global Positioning System”, May 2002) confirms this. It states: “Furthermore, because none of the orbits is perfectly circular, a satellite speeds up or slows down to conserve angular momentum as its distance from Earth varies along its orbit.” Second: To answer your question from another angle, the mathematics required to send up a satellite into orbit or a rocket to Mars is precisely the same whether one makes the calculations from a Heliocentric framework or a Geocentric framework. In other words, a technician may send up a satellite under Heliocentric assumptions, but since these assumptions fit the results of Geocentric assumptions, then the satellite can be successfully launched and targeted. The reason for this is that, since both the Heliocentric model and the Geocentric model must both account for all the motions we see in the sky, then that means that the mathematics used to derive both models must produce the same results. Although the Heliocentric system may, by way of analogy, use the formula 5 + 5 = 10; whereas the Geocentric system may use 6 + 4 = 10, the point remains that both systems will arrive at the number 10, since, by analogy, the number 10 is the only number that corresponds to the precise motions we see in the sky. Thus, the left side of the Heliocentric and Geocentric calculations will differ, but the resulting figures on the right side will always be the same. Be that as it may, over the years, ad hoc adjustments made to the Heliocentric model have resulted in a very cumbersome and sometimes unpredictable system. Although the heliocentric model seems simple at first, a large number of motions must be imposed on the heavenly bodies to actually predict what is finally observed. Even at that, the match is not exact. In fact, the sighting coordinates of telescopes and the trajectories of space probes must routinely be corrected to avoid missing their targets. That is a fact that NASA doesn’t divulge nearly enough. More on the GPS: As a background, satellites are an anomaly for scientists. They know they work, but they are not quite sure how. In a similar way, they know that gravity works, but they don’t know how. All they really know about gravity is that its force is proportional to the inverse square of the distance, but they don’t know what “causes” gravity. Newton himself admitted this. The only thing Newton did is measure the force of gravity and put it into a mathematical formula, not explain the nature of gravity. Unfortunately, most scientists today think that merely because they have a mathematical formula to explain the results of a certain phenomenon, this necessarily means they have discovered the reality, but that is not the case at all. The same reality can be arrived at by many different mathematical formulae. In regards to the GPS, scientists know that it requires little thrust and little adjustment to keep the satellites where they are. They can’t explain it, for their classical understanding of physics requires sufficient amounts of thrust because of the speed required, as well as the necessary adjustments required against the centrifugal and Coriolis forces acting upon the GPS, and the required adjustments against solar disturbances such as solar wind, etc. But instead of admitting this anomaly, they just keep thinking that the earth is rotating and that the satellite somehow manages to keep in alignment with the earth and can be adjusted with little difficulty. In order to compensate for the supposed effects of a rotating and revolving earth, GPS scientists use what they understand to be “relativistic” calculations based on Einstein’s theory of Relativity. But they really don’t need these “relativistic” calculations at all. They incorporate them because they already believe the earth is rotating, and their math cannot work in a rotating earth without incorporating Relativity. (In fact, Relativity was postulated in order to avoid having to adopt the results of the Michelson-Morley experiment of 1887 which showed that the earth was stationary). But the fact is that the GPS works more by trial-and-error than by Relativity. One could easily send up the GPS satellites and, by trial-and-error, seek the best “fit” just by adjusting and readjusting them. Since there are fifty earth-based stations with atomic clocks to help find the best “fit,” the GPS technician is bound to find one that works. Since the atomic clocks only deviate in time by about 4 nanoseconds, the possibility of finding the best “fit” is very easy, and that’s why the GPS work so well. Third: All this leads to the conclusion, or at least an equally plausible conclusion, that, from the Geocentric perspective, what is REALLY happening with regards to the GPS is that the GPS satellites are moving against the inertial framework of the stars and their forces, not the earth. By “inertial framework” we mean the foundation from which a moving body exerts its escape force and thereby moves away from that foundation. In other words, the GPS is revolving every 24 hours with respect to the stars, but not the earth, since the earth is stationary. In the Geocentric framework, it is the stars which are moving in circular orbit around the earth, and it is the gravity of the stars (or any forces caused by revolving stars) which provide the inertial framework for any moving object on or near the earth. Hence, in the Geocentric framework, when the technician sends up his GPS, he is encountering real forces – forces against which he must operate the GPS. He must calculate how much thrust he needs; the inertial values; and all the other things that will be required to keep the GPS moving against the tidal forces of the stars (although, because he believes the earth is rotating, he thinks he is merely making calculations against the centrifugal and Coriolis effects between the object and the earth). Since the inertial force from the stars at 22,000 miles would be in equilibrium with the gravity of the earth, the GPS satellite can virtually hover above the earth at 22,000 miles with little thrust and little adjustment. The GPS would only require enough power and adjustment so that it can remain in position against unpredictable solar forces. This also leads to the fact that in modern heliocentric physics and cosmology, the centrifugal force, which is supposedly the only thing keeping the GPS in orbit, is really a fictitious force, since centrifugal force regards only relative motion, not independent motion. If motion is relative, then all you have are relative effects, not real forces. Some scientists, knowing it is fictitious, have called it the “centrifugal effect.” But in the geocentric model, as physicist Hans Thirring showed, the centrifugal force is real. The GPS is held aloft by REAL forces, that is, the differential forces created between the stars and the earth. In effect, an earth which is standing still provides the same physics as an earth that rotates. As the famous astrophysicist Fred Hoyle said: “We know that the difference between a heliocentric theory and a geocentric theory is one of relative motion only, and that such a difference has no physical significance.” Let me elaborate on this point. I am going to quote from a few paragraphs in a recent article in Physics Today (May 2002) regarding how the GPS works. It reads: In Earth’s neighborhood, the field equations of general relativity involve only a single overall time variable. While there is freedom in the theory to make arbitrary coordinate transformations, the simplest approach is to use an approximate solution of the field equations in which Earth’s mass gives rise to small corrections to the simple Minkowski metric of special relativity, and to choose coordinate axes originating at the planet’s center of mass and pointing toward fixed stars. In this Earth-centered inertial (ECI) reference frame, one can safely ignore relativistic effects due to Thomas precession of Lense- Thirring drag... (Physics Today, p. 42) Did you catch that?! Let me translate for you. “General relativity allows the physicist to use all kinds of fudge factors to account for the results he sees. [The major fudge factors are the Fitzgerald Contraction and the Lorentz-transformation equations which allow you to change time, length, distance and mass, in order to arrive at the answer you want, but we won’t get into those right now]. But we are going to dispense with all those “arbitrary” transformations! We are going to use the Earth as the inertial frame of reference! In other words, we’re going to pretend that the Earth is standing still to figure out how the GPS works, and we can do so because the Lense-Thirring results said we could!” In other words, this scientist, although believing that the Earth rotates against the stars, says that it is easier to do his calculations based on a fixed Earth, and that he can do so because a fixed Earth produces the same results as one that rotates against fixed stars! How deceiving for the layman! He is told that scientists are going to use a fixed-Earth model of the solar system for all his satellite and rocket ship launches, yet he writes in all his textbooks that the earth IS rotating and that there is no way it could be fixed. Give me a break! Later in the same article he says much the same thing: Generally, however, the transmissions arrive at different times. The navigation messages then let the receiver compute the position of each transmission event in the Earth-fixed WGS-84 frame. Before equations can be solved to find the receiver’s location, the satellite positions must be transformed to a common Earth-centered inertial frame, since light propagates in a straight line only in an inertial frame... (Ibid., 45). In other words, calculating the GPS position cannot be solved using HIS equations; rather, he must use equations that are based on a stationary earth that is inertial. Why? Well, he had already told us in an earlier paragraph that “the principle of constancy of “c” [the speed of light] cannot be applied in a rotating reference frame, where the paths of light rays are not straight, they spiral” (p. 44). So rather than admit that his Relativity theory does not really answer the question of light traveling in a rotating frame of reference, he just borrows from Earth-fixed inertial equations, and no one is the wiser. In another paragraph he states: Thus, for each atomic clock, the GPS generates a ‘paper clock’ that reads T. All coordinate clocks generated in this way would be self-consistently synchronized if one brought them together – assuming that general relativity is correct. That, in essence, is the procedure used in the GPS”. (Ibid, p. 43) Notice that he said, “assuming that general relativity is correct.” In other words, this scientist, although he is writing an article titled “Relativity and the Global Positioning System” and believing that Relativity is the basis for it, makes a casual admission that there is a possibility that General Relativity is NOT correct. He only assumes it is correct. Why? Because there has been a lot of discussion in recent years whether Relativity is indeed correct. I’ll just give you two examples. (1) One of Einstein’s more popular “proofs” for Relativity was the precession of the orbit of Mercury. Although Einstein’s figures successfully predicted the precession of Mercury, it was discovered that this was only by accident, since Einstein’s formula incorrectly predicts all the other precessions of the remaining planets! In one of the planets, Einstein’s figures have the recession going in the opposite direction! (2) Einstein predicted the bending of light around the sun (but others did this in 1810). In the 1920 experiment, some of the deflected star light fit Einstein’s theory, but most of the other starlight did not fit, and still others were deflected in the wrong direction! But the pro-Einstein advocates only kept the stars that gave the right answer! This experiment has been done many times, but Relativists still use the 1920 results because the current results are worse than the 1920 results. On top of this, P. Moon and D. E. Spencer (Philosophy of Science, 1959) explained the precession of Mercury from a Geocentric perspective without Relativity or non-Euclidean geometry (pp. 125-134) – the very opposite of Einstein’s theory. Now, here’s another paragraph from the same article in Physics Today: In the equation 3, the leading contribution to the gravitational potential Theta is the simple Newtonian term GM/r. The picture is Earth-centered, and it neglects the presence of other Solar-system bodies such as the Moon and Sun. That they can be neglected by an observer sufficiently close to Earth is a manifestation of general relativity’s equivalence principle. (Ibid., p. 43) This is interesting. Even though scientists believe that the earth is kept in its orbit around the sun due to the sun’s strong gravitational pull, and that the tides on earth are caused by the strong gravitational pull of the moon, this scientist claims that such forces can be neglected when sending up satellites. Oh really? If the moon can pull on the earth’s water with such tremendous force, how is it that it can’t pull on a satellite that is 22,000 miles closer to the moon than it is to the earth? Notice also that he again makes reference to the “Earth-centered” frame of reference. How can he do so this time? Because he has commandeered “general relativity’s equivalence principle.” What is the equivalence principle? It’s the principle that allows them to change frames of reference at will; whatever one suits them will be fine. It says, for example, that, if you fall to the ground, you can’t tell whether you fell toward the ground or the ground came up and hit you. Both are “equivalent,” and in a universe with only relative motion, not inertial motion, one cannot prove one proposition over the other. Do you see how much absurdity is created when you deny that the Earth is fixed? One can say that the Earth hit him, not that he fell to the ground! We put people in insane asylums for less than that! How does this benefit the author of the Physics Today article? Well, by the principle of equivalence, he can discount all the forces he knows to be in the solar system and beyond, and then transfer all those supposed forces as if they were forces coming from the Earth only, and thus his math works! The author then refers to another anomaly he can’t explain between Relativity and the GPS. He writes: One of the most confusing relativistic effects - the Sagnac effect - appears in rotating reference frames. (See Physics Today, October 1981, page 20)....Observers in the non-rotating ECI inertial frame would not see a Sagnac effect. Instead, they would see that receivers are moving while a signal is propagating...Correcting for the Sagnac effect in the Earth-fixed frame is equivalent to correcting for such receiver motion in the ECI frame... Yes, the author is right. It is “confusing.” Unfortunately for him, the reason it is “confusing” is that Relativity has never explained the Sagnac effect, found by Georges Sagnac in 1913, nor its follow up experimental verification performed by Michelson-Gale-Pearson in 1925. In fact, according to Dean Turner in The Einstein Myth and the Ives Papers, he writes: “I pause to note that one may scan Einstein’s writings in vain to find mention of the Sagnac or Michelson-Gale experiments. The same can be said of general physics textbooks and of the 1971 McGraw-Hill Encyclopedia of Science and Technology... Such an oversight in these distinguished encyclopedias constitutes a stinging indictment of professional scientific reporting” (p. 44). Why were they not mentioned in Einstein’s writings? Simple. Because they give experimental evidence for the falsity of Relativity theory. Einstein not only did this with Sagnac and Michelson-Gale, he also did it with Joos, Ives, Miller, Kennedy-Thorndike, and many other scientists who questioned or rejected his theory. Some basic physics What is the Sagnac effect? It is the result of an experiment that showed the earth to be in some type of movement against another substance. The “movement” is termed “rotation” and the substance is some aether-type medium that scientists had discarded when Einstein developed his Relativity in 1905. (Thus, we can see why Einstein would have ignored Sagnac’s results). But although Einstein neglected its results, other scientists did not, including the author of the article in Physics Today (May 2002). How does the author account for the Sagnac effect? By using the same Relativistic “transformations” that he told us he wasn’t going to use in a previous paragraph! This is what he writes: The Sagnac effect also occurs if an atomic clock is moved slowly from one reference station on the ground to another...Observers at rest on the ground, seeing these same asymmetric effects, attribute them instead to gravitomagnetic effects – that is to say, the warping of space-time due to spacetime terms in the general-relativistic metric tensor... (Ibid., p. 44). Clear as mud, right? This is the kind of ‘begging the question’ mumbo-jumbo you see often in theoretical physics of the Relativity variety. What he just said, in case you missed it is, although Relativity cannot account for the Sagnac effect, we are still going to attribute the discrepancies in GPS calculations to Relativistic effects, namely, the warping of “spacetime due to spacetime terms in the general-relativistic metric tensor.” You see, he is locked into a system that doesn’t give him the answers he needs, but since he doesn’t want to admit that they could all be answered by assuming a stationary earth and a revolving aether-type medium, then he will continue to push Relativity as the answer; and all his readers will bob their heads up and down and confirm his gospel, as they have done since 1905. The author more or less admits the effects of these unanswered questions when in one of his final paragraphs he writes: “Historically, there has been much confusion about properly accounting for relativistic effects. And it is almost impossible to discover how different manufacturers go about it!” Ah, yes, and now we can see why there is so much confusion, because no one knows what the heck they are doing! They know their Relativity equations are just fudge factors to explain the things they simply cannot understand under the scenario of a moving earth. Yet they have the audacity to borrow non-moving or “Earth-fixed” equations in order to give the appearance that an Earth in Relativity works! Now you wonder why I’m on the warpath with Geocentricity? One more thing before I leave this topic. The difference between the Geocentric and Heliocentric concept is important, for one of the major flaws in modern heliocentric theory is the failure to account for the effect of the stars on all the motions we see. Modern science has virtually dismissed the effect of forces from the stars, and instead has based its solar cosmology almost entirely on the so-called “centrifugal effects” created by the planets in motion. But this is inevitable, since once you posit that the stars are “fixed” (as modern cosmology does) then the only thing you have left to determine why solar and terrestrial objects move in the rotational paths they do is by the supposed centrifugal effect. And thus, all of the modern heliocentric physics seeking to understand rotational motion is based on a fictitious force, which is not very comforting for anyone wishing to have solid answers for why things work the way they do. Proof lacking for rotation & orbiting AC Assume that the Earth does not rotate about its own axis. (This is the assertion to be disproved.) Since the Earth does not rotate about its own axis, and since we see the heavenly bodies traversing the sky each night, we therefore conclude that the heavenly bodies rotate about the earth. Since we see the heavenly bodies in roughly the same positions from night to night (e.g. at 10 PM Jupiter is at about the same place as it was last night at 10 PM.) we therefore conclude that the heavenly bodies rotate about the Earth with a period of roughly twenty-four hours. (Here – in order to keep the math simple – we assume a circular orbit for the heavenly bodies and a period of exactly twenty-four hours.) Since any given heavenly body traverses a circle about the Earth in twenty-four hours, and since the circumference of that circle is 2*pi*r (r being the distance from Earth to the body) the velocity of the body will be (2*pi*r)/(24 hours). It can be shown (You’ll trust me on the math, I hope. I’ll submit it if you insist.) that any body orbiting the Earth at a distance of more than 4.125x10^12 metres (a couple AU less than the distance between here & Neptune) must be travelling at more than 3.0x10^8 metres per second. Since Neptune & the further bodies can be shown to be traveling at more than 3.0x10^8 metres per second, and since 3.0x10^8 metres per second is the speed of light in a vacuum, and since no material body may travel at or above the speed of light in a vacuum we are faced with an absurdity. And we can therefore conclude that our initial assertion is false. Since we have shown it to be false that the Earth does not rotate about its own axis, we can infer that it does. Much to my horror I have discovered that I have left a clarifying point out of my proof; i.e. my proof – at least the way I’ve worded it – applies only to those heavenly bodies in the Zodiac. Those would be the sun, the planets, with the exception of Pluto, and the fixed stars in the Zodiac. The same argument could be applied to the other stars in the sky, but the math would be different, so I won’t include them here. ======================== GEO What you postulate as proof of a rotating and revolving earth does not prove it at all. First, you assume a few things as proven which have not in fact been proven. One is your assumption that the speed of light (I assume in a vacuum) is constant, either here or anywhere else in the universe. Second, you assume that the planets (and in your second letter, the stars) themselves travel at or beyond the conventional speed of light in order to complete their journey. Let me explain both of these issues by starting with a little history of physics. In 1887, Michelson and Morley did an experiment to detect any difference in the speed of light between north-south travel and east-west travel. A difference in speed was expected because they assumed that the Earth was orbiting the Sun in a stationary aether. From our perspective on Earth, the aether would blow past us like a wind in an east-west direction. Michelson and Morley reasoned that we should notice changes in the speed of light in east-west travel, but fixed speed in north-south travel. The experiment failed to measure any difference in speed, no matter when and where they tried it. Scientists were baffled. Rather than admitting the possibility that the earth was stationary with respect to the aether, scientists dispensed with aether and claimed that the speed of light was constant. In fact, the speed of light was claimed to be the only constant in the universe, whereas mass, length, distance, time, and anything else became relative. This became know as the Relativity theory. But all the Michelson-Morley experiment showed was that aether wind was either too small to measure or was non-existent. Michelson and Morley, however, demonstrated nothing about the constancy of the speed of light through space. Added to this is the experiment performed by Georges Sagnac. A writer for Physics Today writes: “One of the most confusing relativistic effects – the Sagnac effect – appears in rotating reference frames. (See Physics Today, October 1981, page 20) … Observers in the non-rotating ECI inertial frame would not see a Sagnac effect. Instead, they would see that receivers are moving while a signal is propagating ... Correcting for the Sagnac effect in the Earth-fixed frame is equivalent to correcting for such receiver motion in the ECI frame...” Yes, the author is right. It is “confusing.” Unfortunately for him, the reason it is “confusing” is that Relativity has never explained the Sagnac effect, found by Georges Sagnac in 1913, nor its follow-up experimental verification performed by Michelson-Gale-Pearson in 1925. In fact, according to Dean Turner in The Einstein Myth and the Ives Papers, he writes: I pause to note that one may scan Einstein’s writings in vain to find mention of the Sagnac or Michelson- Gale experiments. The same can be said of general physics textbooks and of the 1971 McGraw-Hill Encyclopedia of Science and Technology... Such an oversight in these distinguished encyclopedias constitutes a stinging indictment of professional scientific reporting. (p. 44). Why were they not mentioned in Einstein’s writings? Simple. Because they give experimental evidence for the falsity of Relativity theory. Einstein not only did this with Sagnac and Michelson-Gale, he also did it with Joos, Ives, Miller, Kennedy-Thorndike, and many other scientists who questioned or rejected his theory based on the results of their verified experiments. What is the Sagnac effect? It is the result of an experiment that showed the earth to be in some type of movement against another substance. The “movement” is termed “rotation” and the substance is some aether-type medium that scientists had discarded when Einstein developed his Relativity in 1905. (Thus, we can see why Einstein would have ignored Sagnac’s results). But although Einstein neglected its results, other scientists did not, including the author of the article in Physics Today (May 2002). The Michelson-Gale experiment of 1925 [A. A. Michelson and H. Gale, “The effect of the Earth’s Rotation on the Velocity of Light,” The Astrophysical Journal, Vol LXI, No. 3, April 1925, pp. 137-145] measured a difference in the speed of light at two different latitudes. He concluded that the aether-wind speed changed with latitude due to the rotation of the Earth in a stationary aether. (This is because the radius of rotation decreases with increasing latitude). This experiment disproves the constancy of the speed of light assumption and provides adequate evidence for the existence of the aether, just as Georges Sagnac found. Dalton Miller did even more comprehensive studies to confirm these results. There is quite a collection of letters between Einstein and Miller in which the former is trying to persuade the latter not to put credit in the results. Heliocentrists might be tempted to say that Michelson-Gale provides “proof” of the rotation of the Earth, but that would be presumptuous. The only thing Michelson-Gale provided for us is that either the Earth is moving with respect to an aether, or the aether is moving against a stationary Earth. Not only did Sagnac and Michelson-Gale show the possibility of aether, but an experiment performed by Carl Anderson in 1932 showed another anomaly to Relativity theory. Relativity theory postulated that space was a vacuum – nothing existed between the heavenly bodies. But Carl Anderson showed that a 1.02 million electron volt charge distributed anywhere in space produced a free positron and electron. When the 1.02 Mev was reapplied, the positron and electron disappeared. Einstein’s explanation of this phenomenon was that matter was created and then annihilated. (This may have been where today’s scientists postulate that the universe began from the singularity [“nothingness”] of the Big Bang). Rather than reason that space was filled with positron-electron pairs, in order to save his Relativity theory, Einstein maintained that matter was created and destroyed. So how could the planets and stars revolve around the Earth each day if the Earth is fixed in space? One of the more cogent explanations is that the planets, sun and stars themselves are not moving; rather, they are all embedded in a medium that itself rotates once every 24 hours. This medium would contain the so-called aether or even the Anderson positron-electron pairs, and as some rightly hold, particles in the Planck dimensions. In fact, Hans Thirring, famous for the Lense-Thirring effect, found that for a rotating shell of matter, the interior field of the shell is similar to the field in a rotating system of coordinates, leading to gravitational forces similar to the centrifugal and Coriolis effects in the Heliocentric system. The constitution of the rotating medium would be that coincident with the Planck dimensions found in black holes. Modern science is familiar with such mediums. For example, in The Very Early Universe (Gibbons, et al, 1983) astrophysicist Markov defines the particle he calls the “maximon,” which possesses the 10 to the 94th grams per cubic centimeter associated with Planck dimensions. Also noteworthy in this respect is the work of Dr. Robert Moon, Chicago University physicist, who in his article “Space Must Be Quantized,” shows that the prevailing theory that space is a vacuum is not supported by the evidence. The reason? Because space has an impedance of at least 376 ohms, something not predicted or accounted for in conventional science, but coincident with the spatial mediums of Geocentric understanding. Princeton’s John Wheeler is credited with being the first to describe what is now called “spacetime foam.” This is Wheeler’s theory that space is occupied by ultra-dense particles. Stephen Hawking has postulated something similar. Both Wheeler’s and Hawking’s “foam” reasons that the particles are at Planck dimensions. Thus, this is not something confined only to Geocentric scientists. In an article by J. P. Vigier, “De Broglie Waves on Dirac Aether” in 1980, he writes: “Since Dirac’s pioneer work it has been known that Einstein’s relativity theory (and Michelson’s experiment) are perfectly compatible with an underlying relativistic stochastic [read aether] model.” In fact, the 3 degree Kelvin radiation discovered by Pensias and Wilson is not the remnants of the Big Bang at all, but is more likely the subatomic vibration inherent in this Dirac aether or Wheeler-Hawking “foam.” Moreover, Vigier’s work, along with colleague Petroni, published “Causal Superluminal Interpretation of the Einstein-Podolsky-Rosen Paradox” in Physical Review Letters in 1981. He reports the existence of faster-than-light interactions between an atomic beam of calcium and krypton ion laser, and shows that these are best explained by the stochastic model of space (i.e., aether) rather than the vacuum of conventional physics. There are many other scientists and experiments that could be mentioned to support these findings. Just recently (2001), Princeton scientists showed that a pulse of laser light traveled through cesium vapor at 310 times the distance it traveled in a vacuum. To rotate this spherical body within 24 hours, we can suppose that there is a massive shell at the outer limits providing sufficient gravity to pull the Sun and the stars in their orbits. The aether, like water in a spinning bucket, would rotate along with the universe. Hence, to those inside the shell, there would be no way to measure the rotation; the entire frame of reference would be pulled around by the rotating shell. This concept is not a novelty. It is known in conventional physics as “frame pulling” or “frame dragging,” and was discovered by Einstein, Lense and Thirring, and remains an area of active research. A rotating inertial frame of reference would abide by Kepler’s laws of planetary motion, as well as explain the rotating Foucault pendulum, centrifugal and Coriolis forces. In fact, a rotating universe would explain something that conventional science cannot explain. It is known by scientists that, in order to account for the so-called expanding universe theory, sufficient matter is needed. But scientists have found only 1% of the matter needed. To compensate for this, Einstein (again to save Relativity theory) created his “Cosmological Constant” – a fudge factor to allow the universe to keep expanding. Today scientists account for the missing matter by referring to it as Dark Matter, but they haven’t found it yet. I guess it must really be “dark.” The concept of a rotating universe deals quite nicely with this issue. The less mass the better. And the mass that is present does not collapse in on itself because the centrifugal force (which is a real force in a Geocentric model) causes the heavenly bodies to move outward in just the right balance to compensate for the pull of gravity inward. Hence the mass of the universe (the “1%” conventional science has found) and the spin of the universe (24 hour cycle) is enough to achieve equilibrium. As for faster-than-light action, the rotating universe would have stars traveling in excess of the speed of light, since with respect to the rotating aether, the stars are not moving and there is no difficulty of exceeding the local speed of light. Moreover, in 1955, the astronomer Van de Hulst writes: “In 1930, astronomers discovered with some shock that as the light of stars passes through certain regions of interstellar space it is dimmed and scattered in various directions... If there was indeed an interstellar haze which dimmed the light of distant stars or made them altogether invisible, then many of their calculations of star distances were wrong. Further studies proved that the fear was justified. Starlight passing through the crowded regions of our galaxy loses roughly half its energy by absorption and scattering in every two thousand light years of travel. As a result, even with our most powerful telescopes, we cannot see the center of our galaxy...Beyond about six thousand light years from our observing station most or our studies of the galaxy are literally lost in the fog.” In 1981, the astronomer Baugher wrote: “Much of the galaxy is...hidden from our view, making the study of its structure quite difficult.” There are many other statements like these from astronomers. I think it is also noteworthy to point out that conventional physics and astronomy also have problems with the speed of light. For example, Hubble’s Constant was formulated (H = 100 km/s/megaparsec) based on the proportionality of the red-shift to the distance of the star. The problem, of course, came in when telescopes were able to see beyond 50 gigaparsecs, which would require the galaxies to be receding at many times the speed of light. Then when telescopes were able to see to 500 gigaparsecs, this means that the galaxies would have to be receding at hundreds of times the speed of light. Thus, something is obviously wrong with the whole concept. This evidence certainly doesn’t lend itself to making the conventional wisdom of Heliocentrism sacrosanct by any stretch of the imagination. In fact, things work much better in the Geocentric model. More on satellites AC As I understand it, the real issue we’re discussing here isn’t GPS satellites in particular, but geosynchronous satellites in general, right? I mean, I’m assuming you just chose GPS satellites as a convenient example, right? Well, if so, we’re going to have to pick a different example because I did a little research, and it turns out that the GPS satellites are not in geosynchronous orbits. There are 24 satellites in the GPS network, operating in six different orbital planes, but each GPS satellite orbits at an altitude of only about 12,000 miles (about half the altitude of a geosynchronous satellite) and makes two complete orbits of the earth in less than 24 hours. So let’s forget the GPS satellites and consider instead a truly geosynchronous satellite, such as a Telstar communication satellite. ======================== GEO I already explained this difference in a previous exchange. I said I had used the term GPS for both the satellites at 11,000 and 22,000 miles, the latter being geosynchronous. For the future, I will use GSS for the GeoSynchronous Satellites. ======================== AC And my assertion is that a geosynchronous satellite must move at about 6,800 mph whether the earth rotates or not. That’s simply the speed it has to maintain in order to maintain its orbital altitude of 22,240 miles. Any slower and it would fall into a lower orbit. Any faster and it would rise to a higher orbit. A satellite orbiting a celestial body follows a very simple equation of orbital motion, and that equation is independent of the rotational velocity (if any) of the celestial body itself. Put simply, a satellite in orbit around the earth doesn’t care whether the earth is rotating beneath it or not. It moves at a velocity proportionate to its distance from the earth, and that is just as true of the Telstar satellite orbiting at 22,240 miles as it is of a Space Shuttle orbiting at only 300 miles. Each of those machines will move around the earth according to the equation v = SQRT (GM / r), where v is the velocity of the satellite, G is the universal gravitational constant, M is the mass of the earth, and r is the distance of the satellite from the center of the earth. It’s easy to determine from this equation that in order for the Space Shuttle to maintain an orbital distance from the earth of 300 miles, it must travel at a velocity of 17,058 mph. And in order for the Telstar satellite to maintain an orbital distance from the earth of 22,240 miles, it must travel at a velocity of 6,879 mph. That’s true whether the earth is rotating or not. The fact that such satellites appear not to move relative to the surface of the earth simply proves that the earth IS rotating. You know as well as I do that a satellite has to keep moving in its orbit or it will fall (in fact, an orbit is nothing but a free-fall toward a planet whose surface is always curving out of the way), and so in order to maintain that geosynchronous satellites don’t actually orbit the earth at all, but just levitate up there in space, you assert that as luck would have it there just happens to be a mysterious gravitational force at 22,240 miles from the earth that just happens to precisely balance the gravitational attraction of the earth at that altitude. Now, it seems to me, with all due respect, that you are simply manufacturing "facts" to fit your theory, pulling imaginary forces out of thin air simply because you need such forces to exist. But assertion is not proof. I’ve proved from simple orbital mechanics and from the fact that equatorial satellites with a 24-hour orbital period are stationary with respect to the surface of the earth that the earth does rotate. You need to prove that the hypothetical (and suspiciously convenient) gravitational force you’ve made up really does exist. ========================
  19. K Cole No. You are misunderstanding Newton’s laws. Premise (A1) outlined two of Newton’s laws: his Second Law of Motion: F=m*a and his Universal Law of Gravitation: (Gravitational Force)=G*m1*m2/(radius)^2. I would like to clarify that these laws alone state that more massive objects revolve around less massive objects. When you stated, “Newton’s law that smaller bodies revolve around larger bodies...” you implied that Newton had an orbital law distinct from the two in Premise (A1). In this discussion of orbital mechanics, Newton’s Second Law and Universal Law of Gravitation apply to everything that has mass, without exception. There are no special cases in which they do not hold. It is misleading to say that they only apply to “isolated” systems, since the universe itself can be considered an isolated system. Additionally, these laws apply to any system, regardless of number of bodies it contains. It can have two or three bodies, or even nine or “N” number of bodies. When you say that Premise (A1) “...is true only in isolated systems in which there is one large body and one small body,” you are incorrectly referring to the classic “N-body problem.” The “N-body problem” addresses the fact that every body exerts a gravitational force on every other body in a given system. All these bodies are moving and changing their forces on each other. Once you have more than three bodies (or “N” bodies) in a system, it gets to be a headache trying to predict where any one body will be and how it is moving at some point in the future, and calculations get complex and messy, rather than elegant and clean. Here’s how the “N-body problem” would apply to our discussion. If you sat down to calculate the motion of the solar system, say 100 years from now, it could take you eons to come up with a solution. But let’s say you wanted to stop before that. The longer you keep calculating, the more accurate a solution you will get. But at no point between now and the eons it would take to find a solution would you calculate the sun to be orbiting the earth. The underlying Newtonian physics that the N-body problem assumes will never allow it. Which means that my Premise (B2) is not overturned for any number of bodies in the system. Regardless of the N-body problem, you can get an extremely accurate idea of the motion of the solar system in the future and there are a couple of good practical ways to do it. One is to just let a computer handle the messy calculations for you, and the other is to forget about factoring in bodies that exert a negligible force. Some excellent examples of scientists doing this are: 1) The motion of Jupiter’s moons around Jupiter. This is a wonderful example of how Newton’s laws work, and how the N-body problem is solved. In this case, Jupiter has a whopping three dozen moons! That’s four times as many bodies as the solar system. Yet Newton’s laws hold perfectly (including less massive objects orbiting more massive) and computers can accurately predict the motion of each. This is verified by visual telescopes, radio telescopes, and we can even watch movies of the moons orbiting! 2) The same applied to Saturn’s two dozen moons. 3) The case of spacecraft of Premise (A4). They performed perfectly using Newton’s Laws and in solving the N-body problem. So your objection has no bearing on the validity of my Premise (B2). I will therefore reassert my proof. As it stands, geocentrism is false and I believe I have won your challenge. I appreciate your honest response. ========================== R Sungenis 2: When I used the “N-body” problem, I wasn’t doing so to deny any of Newton’s laws. I was only posing the problem to you to show just how complicated it is to figure out how Newton’s laws are distributed when three or more bodies are involved. Nevertheless, perhaps I didn’t make myself clear, so let me try to explain from a different angle. From the heliocentric perspective, I’m sure you would agree that the sun is a smaller body than the conglomeration of stars at the center of the Milky Way. That being the case, you would have to agree that the sun’s movement is dependent on the force of gravity emanating from the central core of the Milky Way. In your system, the force of those stars is what keeps the sun revolving around the Milky Way to the tune of 500,000 mph. Otherwise, the sun would go streaming off into oblivion. Since that principle is true in your heliocentric system, let’s put the same principle to use in the geocentric system. Let’s start out by saying that the earth is the center of the universe, the sun is 93 million miles away, and the stars are light years away. Now, you would have to agree that, since the Milky Way controls the movement of the sun in a heliocentric system, it would also have to control the sun in the geocentric system, for the Milky Way, in both the heliocentric and geocentric system, would exert the same force on the sun. Now, imagine that the earth doesn’t exist. Imagine that the center of the universe in the above system is just empty space. Would it be possible to construct a universe in which the sun is 93 million miles from the center, and the stars light-years from the center, and have both the sun and the stars revolve around that center point? You would have to agree that the answer is YES. The sun and the stars could be positioned at the precise distances needed so that the centrifugal and gravitational forces from the stars and the sun would balance and thus allow for that kind of universe to exist. To help, a computer could be used to figure out just what kind of masses and distances would be needed to make this model work, and it will work, based on Newton’s laws. Let’s develop the picture a little more. The sphere of stars around the center point fill the entire surface area of the sphere. If we imagine the universe as a big ball, there are stars on the top, bottom, and every where in-between on the surface of the sphere, and in various layers beneath the surface. Thus, the force of gravity from the top to the bottom, and all around the sphere (if the stars are placed correctly), are going to offset each other. They could be placed in such a way where the force of gravity is zero, or almost zero, at the center of the sphere. In fact, there was a study done at Cal Tech about 25 years ago that discovered just that. They had calculated all the known forces in the universe and found that they all canceled each other, but they had one problem – the earth was in the center of the cancellations! It is the same thing that Varshni found in 1975 when he measured all the distances of the 348 known Quasars. He found that they were situated in concentric spheres, and the earth was at the center of each sphere! So if all the gravitational forces, according to Newton’s laws, are offsetting each other, that doesn’t leave too much of a problem in finding just the right balance of forces in that sphere of stars to place the sun at such a point where it was controlled just enough to have it go around the central point. Again, if you know physics, you would have to agree that such a scenario is indeed possible, and a computer could be used to figure out the needed dimensions. If the sun is 93 million miles off-center, then it will require a certain mass and a certain speed to be given to the sun in order to keep it in balance between the sphere of stars that surround it. Now, after all that is done, instead of having nothing at the center, put the earth in the center. The same principle is going to hold, although a slight adjustment to the distance of the sun from the center will be needed in order to compensate for the mass of the earth. All of Newton’s laws would be obeyed. Thus, as you can see, Newton’s laws don’t disprove a geocentric system, rather, all one need to do is find the right configuration of masses and forces and Newton’s law will work quite easily in the geocentric system. Thank you for your submission. Robert Sungenis President of Catholic Apologetics Intl. 24 Mar 03 ========================== M Healy I have no illusions about proving heliocentrism and winning the thousand dollars – I'm not a scientist and my knowledge of science is too limited to enable me to comment on all the factors involved. I am, however, curious about certain points: 1) You mention in some of your rebuttals that NASA uses a geocentric model when making the calculations to launch satellites and space probes. Is this true? If so, how well known is this? ========================== R Sungenis Yes, it is true. I have a letter from them stating so. At other times I have asked them, they have refused to write back. As you can guess, it is not well know in public circles that NASA uses a fixed-earth to make their calculations. ========================== M Healy 2) I was under the impression that the heliocentric model can be used to calculate when Earth and Mars (or Earth and any other planet) come closest to one another in their respective orbits around the sun. If so, would this not constitute evidence for the heliocentric model – or at least show that the math is not the same for both models? I, for one, don't see how (in this instance) the math could be the same – after all, assuming one body is stationary and the other in motion is not the same as assuming that both are in motion. ========================== R Sungenis If I said that 2 + 2 = 4 but you said 3 + 1 = 4, would those two expressions be equivalent? I’m sure you would agree that they would indeed be equivalent. Let’s say that 2 + 2 represented two bodies in motion, but 3 + 1 represented one body in motion against a stationary body. As you can see, the left side of the equation will be different, since you have different things taking place. But when all the motions are added up, they will still equal 4, and thus the two systems will provide the same result. ========================== M Healy 3) Haven't the space probes we've sent out gathered any information that would confirm either the heliocentric or the geocentric model? We're not talking completely blind in this debate, after all, and it seems to me that the space probes and Mars robots must have gleaned something that would point to the actual structure of the solar system. ========================== R Sungenis No, space probes cannot prove the heliocentric model. One of the reasons is that if everything is moving in the solar system (as the heliocentrist claims) then there is no standard from which to measure the rate of movement. Imagine yourself in a room with 20 people moving around trying to determine the center point of their movement. It would be impossible, since the center would keep moving in relation to how the people are moving. The only way you could make real determinations is if one person in the room did not move. That person would be the center, and each person could then measure how far he was from that center person. Without that stationary person, the center would be arbitrary. ========================== M Healy 4) We all know how the heliocentric model explains the seasons of the year: Since the earth is “tilted” on its axis, one hemisphere is tilted toward the sun during half the earth’s orbital period and the other during the other half. The half tilted toward the sun at any given time experiences spring and summer while the other half experiences autumn and winter. It is simple and elegant and accounts for the observations. How does a geocentric model account for the seasons? It would seem (to me, at least) that if the earth were stationary, it would have more uniform climatic conditions (e.g., summer in both northern and southern hemispheres at one and the same time). Thank you for your time. ========================== R Sungenis Instead of the earth tilting 23 degrees, the plane of the sun’s annual orbital precession tilts 23 degrees. This is covered in one of our geocentric challenge posts. Thanks for your questions. Robert Sungenis President of Catholic Apologetics Intl. 24 Mar 03 __________________ NOTE Although the foregoing represents an apparent on-going series of challenges against geocentricity, the providers of this single file containing most of the postings, to date, will not further add to this single file with subsequent challenges. It is felt that this file contains enough of the major scientific aspects of the geocentricity vs acentricity [heliocentricity] controversy to let the reader know that, since neither system proposed for “how the heavens go” is proven or provable by empirical science, what inerrant Scripture has to say about it is the final arbiter in the matter. Photocopies of this compilation are available for $15 [pp] from ............origins@ev1.net The person contending there is no empirical proof that earth moves [orbits or rotates] is the same person throughout these discussions – Robert Sungenis. He may be contacted by e-mail.......................cairomeo@aol.com These exchanges were actual postings [2002-2003] on a website. For further info about that website, contact R Sungenis. March 2003
  20. CASE You are treating the Church like it is a compilation of secular legal statutes where only explicit documents are to be required, when in fact it is a supernatural moral society guided by the Holy Ghost to protect every generation of the faithful. You are accusing the Church for not protecting the faith of many generations who have lived and died being taught, and believing, in the heliocentric model. ========================== SUNGENIS So the Holy Spirit doesn’t guide the Church when She issues “legal statutes” in “explicit documents”? All those “legal” canons and “explicit” anathemas the Church has issued over the centuries against false doctrines are inferior to the “supernatural moral society” that you envision? Know this, Mr. Case: It is precisely through the “legal” and the “explicit” that the Holy Spirit has protected His flock since time immemorial. Every heresy and bad idea ever perpetrated by man has been thoroughly squashed by legal edicts and explicit (not ambiguous) doctrines coming from our forefathers in papal decrees and conciliar statements. The problem here is that you simply don’t like the fact that three popes issued formal and official condemnations of Copernicanism. That is just a thorn in your side that you simply have a difficult time accepting, other than discrediting those popes as being guided by the Holy Spirit and instead appealing to the amorphous “supernatural moral society,” whatever that is. The Holy Spirit doesn’t guide us to truth through popular opinion. That’s the type of religion you’ll find in the New Age movement or Scientology. If you read the Bible carefully, you will find that, like Israel in the Old Testament, the New Testament church was overrun with false prophets. The Christians were so taken by these false teachings that Paul and the other writers constantly warn them that they will lose their salvation if they don’t repent of the false teachings and practices. The Holy Spirit makes no guarantee that Christians will be obedient to His commands. The only thing he guarantees is that no matter how bad it gets, the gates of hell will not prevail in the end. In that realm, the Holy Spirit has been faithful in preserving our doctrine from any tinge of error. But you do a horrendous disservice to the Holy Spirit by suggesting that He led three popes to commit some of the most horrific errors ever known to mankind. Obviously, from your perspective, you don’t believe that Paul V, Urban VIII and Alexander VII were part of the “supernatural moral society”? ========================== CASE All this means is that one cannot say the heliocentric model is harmful to the faith anymore, because extrinsic circumstance have changed. However, you can still promote what model you think is the correct one according to scientific fact. ========================== SUNGENIS So you speak for the Church on this issue? If that is so, please tell me where can I find an official statement from the Church that says that the “heliocentric model is not harmful to the faith anymore because extrinsic circumstances have changed”? ========================== CASE Pope Leo XIII wrote Providentissimus Deus in 1893 and stated, "…the rule so wisely laid down by St. Augustine – not to depart from the literal and obvious sense, except only where reason makes it untenable or necessity requires…" Because the Church has obviously permitted it, necessity indeed requires it to avoid faulting the Church. Be that as it may, on the purely secular side, my position has focused on the concept where "reason makes it untenable", meaning that in a case of only two models, absurdity in one automatically makes the other the reasonable one. Judging from your inconsistent use of terms, I think that you are confusing what is "science" and what is "scientific". A "theory" may be scientific, but it only becomes "science", properly speaking, once it is proven, or demonstrated. Theories are merely unproven possibilities suggested by the mind. They are called "scientific" only because they pertain to science, as a means of attaining fact. Theory is science in the process of being built. For example: theory tells us by solid mathematics that space is infinitely divided in half, such that an inch can be divided to a half, then a quarter, then an eighth, and so on to infinity. However, literally at the drop of a hat we can prove the theory is wrong otherwise there would be no such thing as movement – but there IS. Therefore the theory no longer exists – because it is easily proven there must be a smallest possible distance. And when we bring Church teaching into it we see that our conclusion makes sense since the attribute of "infinity" cannot be attributed to any thing created, which is "finite" by nature. The reason the concept is in our mind, and not in the created world, is because our minds are spiritual and are capable of understanding "infinite". Science properly speaking is "knowledge" (from the Latin word for knowledge) of the aforementioned kind that both St. Augustine and St. Robert requested as necessary – that which is a fact because it can be demonstrated. It seems you have done well to explain what you request for as "proof", but from the proceedings you violate your own rules by your main assertions for geocentrism: 1. The sun is embedded at a distance from the Earth. This is a merely gratuitous, hypothetical invention or assumption with no basis in demonstration and flatly goes contrary to our demonstration that objects in space are as free to move as in space with gaseous atmosphere. You are presenting science fiction not a "cold hard fact". ========================== SUNGENIS First, the “Challenge” is not for me to prove Geocentrism, but for you to prove Heliocentrism, since of the two of us, you’re the one who claims that you can prove his model from science. Thus, if I present a hypothesis for my view that may or may not be true, it does no damage to my end of this challenge, since I don’t have to prove anything. The only thing I have to show is that you have no proof for your model. For my proof I turn to Scripture and Church teaching, not science. The only thing science shows me is that Geocentrism is a plausible model of cosmology. As for assertion that “objects in space are as free to move as in space with gaseous atmosphere,” I’m sure you’ve heard of the thing called GRAVITY. That’s the force in your system that keeps things from flying off into various directions. Granted, within that envelope of gravity objects have some freedom to move about, but they don’t have much. If you don’t believe so, then please tell us why, after so many millions of years, the earth’s orbit (as you believe it performs) has not deteriorated. Second, that the heavenly bodies are “embedded” in an ultra-fine substance is an ongoing scientific concern made by many scientists over the last century, and it is not, as you claim, “merely gratuitous, hypothetical invention or assumption with no basis in demonstration.” I suggest you read up on the Sagnac experiment of 1913 (the one which Einstein totally ignored) which shows that there is a substance permeated in space that effects every heavenly body. These same experiments were repeated by Michelson-Gale in 1925 (whom Einstein also ignored), and Dalton Miller in 1933 (whom Einstein also ignored, and in fact, tried to silence), and Herbert Ives in 1943. I’ve read the literature and I can assure you that my assertions are not pipe dreams, but have some of the most sophisticated experimental evidence ever gathered by science. Even Newton himself, contrary to popular opinion, said he never wanted to be understood as advocating a totally vacuous state between heavenly bodies. ========================== CASE 2. The stars in the universe push and control the sun around the Earth. Merely hypothetical "assumption" or "cold hard fact"?. When you consider vector forces that would be required to point to the center of the Earth, the forces would all negate each other from the other side of the universe, which makes it absurd. Like 10 men surrounding a large ball and all pushing on it from different angles! And when you apply this assumption to the other planets revolving around our sun, those forces, you would have to say, are not pushing towards the Earth but are pushing all of them RATHER toward the center of the sun WHILE it is moving 24 million miles/hour around the Earth! ========================== SUNGENIS Again, I don’t have to present any “cold hard fact.” In fact, I’m the one saying that there are NO “cold hard facts,” for either you or me. That is why I’m saying that it’s useless for you to try to prove Heliocentrism, since you don’t have any “cold hard facts” at your disposal. But not only do you not have any “cold hard facts,” but you don’t have any Scripture, Patristics or official Papal statements supporting your view. In effect, you have nothing, except the claims of some scientists, most of which claim no allegiance to God. As for “theories,” you haven’t read the literature on Le Sagean gravity, have you? My guess is that you haven’t even heard of it until I mentioned it to you. For that matter, you know precious little about Newton’s theory of gravity. In fact, Newton and Le Sage knew each other and exchanged notes on this issue. My guess is you know relatively little science, since from the way you express yourself and the evidence you bring forth you show a remarkable ignorance of current ideas. Yet you like using words like “blasphemous” and “absurd” to make it look like you know what you are talking about. Unfortunately for you, I can see through it very easily. I suggest that before you start critiquing the theory, you might want to read up on it first. As for your statement that “the forces would all negate each other from the other side of the universe,” you’re closer than you think. As I mentioned in my last post (which you never answered), a study done at Cal Tech about 25 years ago found that, when they added up all the forces in their telescopic field, they found that they all cancel each other out, but with one very interesting result – the earth was in the neutral zone of the cancellations. In other words, the earth was found to be the center of all the forces, and thus the center of the universe. They found that to be, in their own words, “a horrible conclusion,” since it supported Geocentrism, not Heliocentrism. Perhaps that is why the Bible says in Job 26:7 that “he hangs the earth upon nothing.” Another study done by Varshi of the known 348 Quasars found that all of them were arranged in concentric spheres, but with one interesting feature – the earth was in the exact center of each sphere. ========================== CASE 3. The Earth is unique among the planets. Merely gratuitous in order to escape having to apply known and proven physical laws that have already been demonstrated as true science, while you insist others present a natural and physical explanation. ========================== SUNGENIS What “proven physical laws” have I, as you say, “escaped”? Flinging accusations is one thing. Backing them up is quite another. ========================== CASE And while we notice that all other planets are revolving on their axes and have their own day and night because of it, you gratuitously escape this by simply claiming "uniqueness" for the Earth, out of thin air, even though the Earth, of comparable size and looks, sits within this natural solar system of God's creation. ========================== SUNGENIS I am not “escaping” anything, since the rotation of the planets has little to do with proving whether the earth rotates, which is your task to prove. In fact, the rotation of the planets is quite strange. Mercury hardly rotates at all, only making three rotations per year. Venus rotates in the opposite direction of the other planets. Uranus and Pluto rotate north/south instead of east/west. The ones that do rotate show a marked centrifugal consequence, such that north/south circumferences are thousands of miles shorter than their east/west circumferences (e.g., Jupiter). Conversely, photographs of the earth from space show no such east/west bulges, but a perfectly spherical shape, which gives evidence that there is no rotation. Satellites are also puzzling. Our moon doesn’t rotate. Some of the moons of other planets rotate, some don’t. In addition, of Jupiter’s 16 moons, four go in the opposite direction to the other 12. One of Saturn’s moons goes in the opposite direction, and two of Saturn’s moons switch orbits every four years. There are many more such anomalies. So, if you are looking for some pattern in the solar system from which you assert that the earth cannot deviate, there is hardly a pattern to rest on. The planets are about as different from one another as trees in a forest. Be that as it may, you assert that I am claiming “uniqueness for the Earth out of thin air.” Have you read the Bible recently, or is this accusation one that you just pulled “out of thin air”? In Genesis 1 you’ll find that the earth was created first, and then the stars and sun were added later to fix times and seasons for the earth, not vice versa. Can you tell me any planet in our solar system that even gets one mention in the Bible? Or why the earth is called the “footstool” of God (Is 66:1)? Or why all the significant locations of space pivot off an earth-centered framework (Phil 2:10)? Or why all the universe is reduced to the phrase “heavens and the earth” (Col 1:16)? Or why Jesus descended to the lower parts of the earth (Eph 4:8-9)? I guess all these logistical references are mere coincidence for you. If you want to accept the teaching of Scientism that the earth is a mere speck of dust floating around in an unending universe, that is your prerogative. All I can say is that you are short-changing yourself and those you influence, and giving a welcome doormat to the naturalism that man so craves in order to push God into the remote recesses of his mind. ========================== CASE 4. Finding a book from a scientist, or appealing to one in history, who make these assumptions merely violates your rule that "expert" testimony is not acceptable. A person who is publicly called a "scientist" doesn't make his "theory" become "science" because he presents it to the public. ========================== SUNGENIS Again, you didn’t read the rules very carefully. It is not I who am limited in this discussion, but you, since you are the one claiming to be able to prove Heliocentrism through science. In accepting our “Challenge,” you accepted the challenge to prove your theory, not merely assert it. ========================== CASE Physics for the Earth is as solid as it is for the other planets orbiting the sun. We say the Earth, between Mars and Venus, is simply acting according to the same laws of physics as those two, and the other planets. All of which laws have been DEMONSTRATED. You simply sweep those laws aside from pertaining to the Earth by saying it is "unique", and thus gratuitously exempt it from the demonstrable laws of physics. And when you enter into gratuitous theories to say what "possibly" could be, absurdity after absurdity arises when you look at how your "theory" explains the other planets going around the sun. ========================== SUNGENIS It is obvious that you are not willing to admit that the only thing you have “demonstrated” is mathematics. Unfortunately, for you, it is not the math with which I have a contention, for I have stated very clearly, several times, that the math in both our models is EXACTLY THE SAME. There is not one “law” I have denied. What you have not “demonstrated” is that the math proves heliocentrism and disproves geocentrism. Until you do, you can talk about “absurdities” all you want, but the only thing absurd is that you keep arguing in circles. ========================== CASE Your Web site states, "the so-called laws of physics that are often used to assert its dominance in science are not laws at all, but also unproven theories." This is plainly NOT true. They have proven those laws with demonstration and because they don't go along with your position, you doubt they are demonstrable. That is contrary to the mind of the Church. ========================== SUNGENIS The Church wants truth, so whatever law you demonstrate as truth, the Church will accept, and so will I. As for my quote above, it was referring to the theory of Relativity. Relativity, if that is what you believe, is an unproven theory. That’s why they call it a “theory.” And being that Relativity was actually an apology to why scientific experiments showed the earth standing still in space, then the burden is on you, not on me. The problem is that those of your persuasion are forced into accepting Relativity, since you have eliminated any point in space as being non-relative. But in a system with an immobile earth, nothing is relative, since everything can be measured against one stable point – the earth. ========================== CASE Elsewhere you state that the math is the same for both models. This is also NOT true. The only math that fits both is for VISUAL RELATIVE POSITIONING. I had admitted that at the beginning. But I repeat, visuals are only a small part of the actual facts. The physical laws of God's creation include vector forces, mass and momentum that cannot be discerned by visual positioning. You accept the math of the visual positioning because it conforms to your belief that the Earth is motionless. Because all the other demonstrable laws of physics don't fit what you believe, you sweep them aside with the gratuitous assertion that "the earth is unique" so as to exempt the earth from those other laws and the associated math that can prove those laws demonstrable and repeatable. There are no absurdities with heliocentrism because it doesn't try to create a "unique" Earth exempt from the same laws that pertain to the other planets around it. ========================== SUNGENIS I implore you to stop speaking in generalities. This debate is not an exercise in making speeches that may sound good to the uninformed. There are no laws of physics which I deny. I merely deny you the prerogative of using mere theories to prove your contentions of a heliocentric solar system. As it stands, there are no established laws of physics which prove heliocentrism and deny geocentrism. If you have one that does, then let us see it. As for the math, if you contend that “vector forces, mass and momentum” deny the geocentric model, then show us how that is so. Until then, you are merely sounding off on things about which you apparently know very little. ========================== CASE All the planets have a speed in orbit around the sun in proportion to their size and distance from the sun. And the Earth fits that same proportion for orbital physical science. But you exempt it, and would have to suggest that all those other planets are actually orbiting the sun while the sun is orbiting the Earth daily at about 24 million miles per hour! Absurd. ========================== SUNGENIS Check back on the last post I presented. There you will find that a heliocentric system demands that the sun move around the galaxy at a half million miles per hour, and that the Milky Way galaxy move about 100 times faster than the sun around clusters of other galaxies, and that the outer most galaxies are moving faster than the speed of light. Now that’s what I call “absurd”. Thus, it is you who has the problem with exorbitant speeds, since your system DEMANDS these impossible speeds for the stars. What you apparently don’t know or are unwilling to admit is that, if you begin with a heliocentric solar system, there is no escape from these exorbitant speeds. This is what I was trying to explain to you in the beginning of this post. You cannot deal with the heliocentric system in a vacuum. Once you adopt the heliocentric system, then you must explain every other motion in the heavens, and they must be explained in accordance with the distances you associate with heliocentrism. Those distances demand much higher speeds for the stars than I have in my system. Moreover, at least in my system the stars don’t have to travel those huge speeds themselves, rather, they are carried in an aether medium that satisfies almost all the speed demands. ========================== CASE All the planets rotate on their axes. That means that each sees the sun rise and set just like on earth as though the sun moved. But the earth, between Mars and Venus, is different? That is pure hypothetical assumption that only brings absurdity in its trail. ========================== SUNGENIS I’ll remind you that my “Challenge” demanded that you prove heliocentrism, not resort to name-calling and accusations merely because you object to an alternate viewpoint. The fact is, you have proven nothing, except that you don’t have any proof. ========================== CASE In short, demonstration is ABOVE literal interpretation of Scripture where it is not of faith. But you violate this. You DISMISS, with a gratuitous "unique" wave of your hand, the physical laws that have been demonstrated, in order to make literal interpretation involving the sun ABOVE that demonstration. ========================== SUNGENIS You haven’t “demonstrated” heliocentrism. You’ve demonstrated hardly anything in this your third try at “proving” heliocentrism. In fact, of all the entries to our challenge, yours is by far the least challenging. I can’t think of one argument you’ve presented that even comes close to proving your point. Instead you resort to accusations that I am dismissing laws of physics, yet true to form, you show no proof for that accusation. ========================== CASE I have seen another challenger mention how earthquakes on earth slow the rotation of the earth by a perceivably small amount that our sensitive instruments can detect now. You answered that person but completely omitted addressing that significant point. Because if you address it, you would have to deny what has been repeatedly demonstrated by scientists nowhere near the earthquake, and you would also have to say that such an earthquake perceivably slows the sun and the whole universe of stars light-years away all at the same time. Absurd. It really just affects the momentum of the rotation of the Earth. ========================== SUNGENIS Earthquakes slowing the rotation of the earth? What proof of this do you have? Please stop making mere assertions without the scientific evidence to support it. Show us the references. Be that as it may, let’s just use your assertion for the sake of argument. That would mean that every time there is an earthquake, the earth would be slowed down from its rotation. But wasn’t it you who, a few essays ago, was telling me that the forces which would inhibit the free rotation and revolution of the earth in its path were quite minimal, such that in all the years the earth has existed there would be no appreciable decrease of its movement? But now, when it’s to your advantage, you claim that even earthquakes can slow that movement. Let’s see. There have been hundreds of major earthquakes just in the last few hundred years. If all those earthquakes were slowing down the rotation of the earth, as you claim, then the cumulative effect of all those earthquakes should have changed our sidereal day of 23 hours, 56 minutes, and 4 seconds quite drastically in that time period. But the sidereal day has never decreased, as far back as we have records. The only thing that changes is that the sidereal day can go from 23 hours, 56 minutes, 4 seconds to 24 hours, 0 minutes, and 15 seconds, and back again, depending on the time of the year. It stays right within that envelope, despite all the commotion on earth, including earthquakes. In fact, since, from your perspective, all the earthquakes on earth might indeed inhibit its rotation, it only serves me well to say that the reason earthquakes don’t decrease the sidereal day is that the sidereal day is not dependent on the earth rotating, but upon the universe rotating, without deviation, around the earth. ========================== CASE You admit all the heliocentric physical laws work for all the other planets going around the sun, so you have no business whatsoever to gratuitously EXEMPT the earth from those same demonstrable, natural laws by denying those laws apply to the Earth by the pure invention that it is "unique" among them. ========================== SUNGENIS No one is “exempting” the earth from any “laws” that the other planets follow. Positioning the earth in the center does absolutely no harm to the “laws” of physics. I follow the same established laws that you do. The only difference is that I put the sun and the planets in a different position than you. For that matter, it is a known fact admitted by all astronomers that, mathematically speaking, we can make any planet in the solar system the center, such that we can develop a working physical model to support it. All the models will follow the same laws of physics. It’s not difficult at all to demonstrate. Any computer with the right program can do it. The question is, which one is right? The answer, if we want to be honest about our science, is that we don’t know, since it is not possible to go outside the solar system to observe which one is the true center. If you think that a sun-centered solar system is the only possible answer, then you’re just fooling yourself. ========================== CASE We are reminded by the Church, "not to depart from the literal and obvious sense, except only where reason makes it untenable or necessity requires". Reason and necessity tell us clearly that the presence of even one absurdity is enough. And even if a Catholic doesn't know the slightest facts of science or physics, he knows the Church has universally approved the teaching of the heliocentric model as both reasonable and harmless. ========================== SUNGENIS Where has the Church officially declared that she “universally approves the teaching of the heliocentric model as both reasonable and harmless”? You are merely making assertions without evidence, and that is very close to telling a deliberate falsehood. ========================== CASE It is rationalistic to fault the Church for Her centuries of permission and approval of books by imprimatur. Imprimaturs participate as a function in the ordinary magisterium of the Church. There is no historical precedent for the entire Catholic Church to not make a condemnation for prevalent harm to the faith ESPECIALLY when that alleged harm was taught to EVERY CATHOLIC in school for generations living and dying in that belief. That would be another absurdity you are implying with your stand. ========================== SUNGENIS The only thing the Church has officially allowed to be published, according to Pius VII, are those who want to write about the “opinions” of those who hold an alternate cosmology. Other than that the Church has never officially sanctioned the heliocentric viewpoint. If you have such an official statement, then show it to us. Until then, you are left with merely the opinions of bishops who have little authority to decide this issue, especially in light of three popes who have already deemed it “contrary to Scripture.” And once again, you are so ready to assert that the magisterium is acting appropriately on the mere evidence of an imprimatur by a fallible bishop, but are so unwilling to grant the same authority to three popes who made formal edicts condemning the view you espouse. Was the Holy Spirit hiding from the Church in the seventeenth and eighteenth centuries? ========================== CASE And these absurdities are making you fall into illogic, as when, in regard to Providentissimus Deus of 1893, you stated, "Thomas Aquinas believed in geocentrism, not heliocentrism. So it is rather difficult to conclude that Leo is using as proof the very man who did not apply "what sensibly appeared" to geocentrism." St. Thomas only stated a universal "principle". He did not have the demonstrable scientific facts of later centuries pertaining to heliocentrism to apply it to. Almost 600 years of science is quite substantial between that pope and that Saint. ========================== SUNGENIS Yes, Thomas Aquinas didn’t have “demonstrable scientific facts of later centuries pertaining to heliocentrism,” but neither do you. You have not presented one shred of proof for your contentions, in now this, your third try. At least Thomas paid attention to the Fathers who went before him, and disregarded the Greeks who taught heliocentrism long before Copernicus. You, on the other hand, repudiate the Patristic witnesses, the popes which condemned your view, and the Scriptures which offer not the slightest proof for your cosmology. You’re allegiance in these areas is Scientism, not science. True science would make you realize that you can’t prove heliocentrism. But it appears that you have decided to cast your whole lot in Scientism’s lap, the very people who, for the most part, have a vested interest in dethroning divine revelation in favor of their own pet theories and concoctions. As it turns out, you have not proven your claims. I hope that after this exchange, you will reconsider those claims, and I pray that God will lead you in that direction. =========================== AC [ not Case ] I'd like to submit a second challenge to geocentrism. I believe my first challenge may have been too technical, but this second one hopefully will be much easier to understand. First, I'll offer your definition of proof called for by the challenge: "By 'proof' we mean that your explanations must be direct, observable, physical, natural, repeatable, unambiguous and comprehensive." Premise (A1): Newton developed his physical laws, which form the basis of orbital mechanics. These equations include his second law of motion: F=m*a and his law of gravitation: (Gravitational Force)=G*m1*m2/(radius)^2. Premise (A2): Since Newton formulated his laws, they have always been verified by the motion of objects travelling much slower than the measured speed of light. There has never been an observable case where Newton's laws did not hold for objects travelling much slower than the measured speed of light. Premise (A3): In order for Newton's laws to correctly predict motion in the solar system, gravitational forces from all massive bodies must be correctly taken into account by scientists. Gravitational force according to Newton is directly related to mass. Gravitational force is also directly related to and varies according to a spacecraft's distance from each of these bodies as seen in G*m1*m2/(radius)^2 from (A1). (These massive bodies include the sun, Earth, moon, planets, etc.) The sum of these gravitational forces equals the "F" in F=m*a. Premise (A4): Scientists send spacecraft (which travel much slower than the measured speed of light), and have done so multiple times, through the solar system using Newton's laws with exact precision. Such spacecraft include Voyager-2, Pioneer-10, and the Apollo moon missions. In other words, the spacecrafts' motion, as described by the "m*a" in F=m*a, was correct or true. Premise (A5): Given (A1), (A2) and since the motion of the spacecrafts were true (A4), then the scientists' calculation of the solar system's bodies' gravitational forces (A3) must also be true. Conclusion (A): Scientists possess a correct and accurate understanding of each body's gravitational force in relation to the spacecraft and, consequently, a correct understanding of each body's mass. Premise (B1): Geocentrism places Earth in the center of the solar system, and all other bodies (including the sun) rotate around Earth. Premise (B2): According to Newton's laws, less massive objects orbit more massive objects. Premise (B3): If (B1) is true, then (B2) predicts that the Sun is less massive than the Earth. Premise (B4): But Conclusion (A) is proven true, and scientists understand that the Sun is more massive than the Earth. Premise (B5): Given (B4), then either geocentrism (B1) is false or Newton's laws (B2) are false. Conclusion (: Either the geocentric view is incorrect or Newton's laws are incorrect. Premise (C1): But (A2) is true. Premise (C2): Your definition of proof is that "...explanations must be direct, observable, physical, natural, repeatable, unambiguous and comprehensive." Premise (C3): (C2) defines (A2) as proof that Newton's laws are correct. Premise (C4): Given (C3) and Conclusion (, then (B1) is false. **Update** Conclusion ©: Geocentrism is false, and Newton's laws are true. Newton laws then correctly predict that Earth orbits the Sun. This clearly and irrefutably disproves geocentrism, and proves that Earth orbits the sun. ========================== GEO You're missing one thing. Newton's law that smaller bodies revolve around larger bodies is true only in isolated systems in which there is one large body and one small body. (In fact, Newton had problems explaining what would happen if a third body, or even a multiple number of bodies, came between two bodies whose mutual force was originally calculated using the inverse square law). But the fact is our universe is not an isolated system. It includes innumerable galaxies. These galaxies directly effect the movement of the sun, which in turn would effect how the sun moves in relation to the earth. For example, in the heliocentric system to which you hold, you believe the sun is revolving around the Milky Way galaxy at 500,000mph. What is it, in your system of mechanics, that holds the sun in this orbit? Obviously, it is the gravitational balance between the Milky Way and the inertia of the sun, according to Newton's laws. Thus, you would have to admit that the sun's movement is controlled by the stars in the Milky Way. That being the case, we can also create a geocentric model of the universe. Using Newtonian mechanics, we can construct a mathematical model of the universe such that the earth is at the very center, the sun is in the middle, and the stars are on the rim. If all these bodies are positioned in the exact places they need to be, with the exact masses they need to have, it would result in a system in which the force of the stars carry the sun around a central earth, much like the rim of a spinning bicycle wheel carries the spokes around the axle. This would not be hard to design at all. A good computer could figure out what the proportions of distance and mass would have to be to satisfy both a Geocentric universe and Newtonian mechanics. You haven't disproved geocentrism. In actuality, you have allowed us to demonstrate once again that the same laws with which you work are the same laws that govern a geocentric universe. =========================== This next challenge from Hutton Gibson – 15 Mar 03 H Gibson The space probe figures are easily ignored or misunderstood unless interpreted. My explanation should sufficiently support the figures. These were compiled at Tidbinbilla, near Canberra, A.C.T., Australia in 1995 and 1996. Pioneer-10, 03/03/72; passed Jupiter 12/03/73, escaped solar system 1986 week.....decl....rt.asc.....earth dist....variant.....sun dist.....variant 39.........25.8.....75.2.......9793.8......................9844.6 35.........25.8.....75.2.......9834.3........+40.5.....9814.7........-29.9 31.........25.7.....75..........9870.3........+36........9784.9........-29.8 26.........25.7.....74.6.......9887.8........+17.5.....9747.5........-37.4 22.........25.6.....74.1.......9868.8.........-19........9717.6........-29.9 17.........25.6.....73.5.......9799.6.........-69.2.....9680.2........-37.4 13.........25.5.....73.2.......9714.0.........-85.6.....9650.3........-29.9 52.........25.7.....73.6.......9418.3.........-295.7....9553.1.......-97.2 =========================== R Sungenis Thank you for sharing data of the several space probes. For the sake of brevity and simplicity in this discussion, I have selected the data for only one of the probes, to which I’ll confine my explanations as to the fallacy in your thesis. =========================== H Gibson Pioneer-10 went relatively straight, is far beyond influence of the solar system to divert it, and not far enough to have encountered stellar interference. There was certainly no reason for it to do into anything remotely resembling an orbit. It continues on its generally straight path and has the same unmoving stellar background. I conclude from this that the background is equally stationary. =========================== R Sungenis Although that certainly remains a plausible scencario, it cannot be proven. Here is the reason: If, as is true in the Geocentric system, the stars are rotating against an immobile earth, Pioneer-10 is rotating with the stars and thus will appear to be traveling in a straight line toward the stars. The reason that Pioneer-10 would move with the stars is that it is embedded in the same gravitational field that, for example, the sun is embedded as it makes its way around the Milky Way in your heliocentric/cosmological system. In our Geocentric/LeSagean cosmological system it is the aether or ultramundane particles in which all the objects of the universe are embedded and within which gravity is created. Since our model holds that the universe rotates around the earth once per day, hence the aether also rotates around the earth once per day, and thus, all the objects we see from earth are rotating with the aether. Again, the reason you see Pioneer going in a straight line toward the stars from your vantage point on earth is that both the stars and Pioneer are rotating around the earth at the same speed. The differences between Pioneer and the stars would be due to Pioneer’s independent movement within the aether, even while it is moving with the aether and the stars. =========================== H Gibson I phoned my contact at Tidbinbilla tracking station on 23 Oct 96. He faxed eight pages of statistics, status of tracking posted every four or five weeks during 1996 and the last week of 1995. I extracted figures which prove geocentrism currently non-existent, and enclose them on a separate sheet. A report from my Australian contact follows: Please note the three-month gap between this and last year’s entries. Please note also in the figures on Pioneer-10 and the Voyagers their steady recession from the sun in comparison with their both greater and less, even minus, recession from earth – the obvious and necessary result of a relatively fixed sun and an orbiting earth. I put it to the observer that heliocentric theory requires an annual blank spot on each probe as earth passes directly opposite with the sun between us and the probe. He even had a term for the blank spot – “superior conjunction” – and volunteered a near date for the next superior conjunction involving Galileo. I include figures on the orbiting probes for comparison to demonstrate that no one can have it both ways when attempting to argue geocentric theory. The report continues: Astronomers locate objects in a celestial grid resembling latitude and longitude. Each star, planet, satellite, and space probe is fixed by use of these coordinates, declination and right ascension. These vary almost imperceptibly in the case of stars, constellations, and galaxies – the greater the radiomeasured distance the less variation, which is partially accounted for by the semi-annual variation in orbit of the terrestrial base of observation by the diameter of the orbit. These figures are given in degrees, and the distances are given in millions of kilometers. I list first the three probes sent out in a single direction; then, for contrast, several which have gone into solar orbit. You will note far greater variations in the latter. Please note that in the cases of Pioneer-10 and Voyagers-1 and -2 their increase in distance from the sun is constant, but their radio-beam-measured distance from earth increases and decreases in regular patterns. As Pioneer-10 receded all through the period covered, the earth began to catch up with the probe. The earth moves faster than the probe’s constant recession. The probe cannot back up, nor change course. Note also that Voyager-1, heading off in a different direction, came in Week-35 to an almost even distance from sun and earth, and over the next four weeks began to recede from the earth faster than from the sun. So the earth moved further around its orbit, back toward the probe’s superior conjunction. Or the probe travels at different speeds simultaneously? =========================== R Sungenis But this doesn’t prove that the earth is revolving around the sun. All it proves is that, at certain times, the distance of Pioneer from the earth is greater and has more variation than Pioneer from the sun. In the geocentric system this is easily explained by understanding that, as the sun revolves around the earth, the whole universe is moving with the sun, with the earth as the center. Hence, any objects within the space between the earth and the rest of the universe are all going to move by the same proportions as the sun. In order to illustrate this, I have made four diagrams of an earth-centered system which incorporates the figures from weeks 1, 13, 26 and 39 from your chart of Pioneer-10’s distances. The illustrations are crude and certainly not to scale, but they will give you an idea of how your numbers can be explained in the Geocentric system. Week 52 was not included since it does not seem to fit the pattern established in the other numbers, since week 52’s numbers are less than all the other numbers, but should be more due to the passage of 13 additional weeks from week 39. However, I cannot vouch for the accuracy of your numbers for Pioneer-10. In fact, I have no way of knowing if they are accurate. Hence, I can only accommodate you by using the numbers you provide, but I only do so only in principle in order to give a rough estimate of how the numbers would fit into the Geocentric model. Nevertheless, this brings up an interesting point of contention. How is it that your man in Australia is able to chart the distance from the sun to the satellite? Surely there is no probe on the sun for him to use. Hence, he must arrive at his figures for the sun’s distance away from Pioneer by using some type of triangulation, but that involves certain assumptions that I don’t think I’m required to accept. In any case, here are the diagrams, which begin on the next page: Diagram 1 Diagram 2 Diagram 3 Diagram 4 ========================== H Gibson Let me again press the point that three tracking stations are needed to track down probes going in a nearly straight line away from the earth. These probes rise and set like the sun and the respective constellation always behind each. If the earth does not rotate, only one tracking station, located near the departure point, would suffice, because the probe would never “set.” =========================== R Sungenis The probes are going to move with the stars because they are embedded in the aether the same as every other body in space. They appear to rise and set to you because they are moving with the sun and stars, while the earth remains immobile. =========================== H Gibson If the sun orbited the earth (at 365 x earth-orbit speed) it could not avoid traveling also around the moon. At any point on the earth from which the moon was visible we would then see all phases of the moon each calendar day. On the occasion of each eclipse of the sun there would occur an eclipse of the moon on the opposite side of the earth within fifteen hours of the first eclipse. Unlike the space probe proof, this is undemonstrable because no man can move stars and planets. =========================== R Sungenis This would not be the case in a Geocentric system, since the sun and the moon are both moving around the earth in the aether. Their respective distance from each other, and the angle they form with each other, is going to be the same as in your system. In other words, as the sun moves around the earth in 24 hours, the moon is also moving around the earth in 24 hours, and the sun and the moon are keeping the same angular distance from each other as in the heliocentric system. Thus, the phases of the moon’s light are going to be exactly the same as in your system. Let me illustrate with some diagrams: Diagram 5 Diagram 6 =========================== H Gibson My first argument (Mercury and Venus have yet to orbit the earth) constitutes scientific proof that geocentricity is false. =========================== R Sungenis Mercury and Venus do not orbit the earth. They orbit the sun in the Geocentric system. The sun, with the planets, revolve around the earth. =========================== H Gibson A gentleman in Pennsylvania requested my proof, for $10,000. So I forwarded my figures and a copy of the fax from which I had extrapolated them. I quote the disappointing reply: The offer “insists that empirical scientific observation,” which is impossible, not political bluffery, is the basis of the proof needed. What you are suggesting is what I will term “space navigation”. I will take the lazy way out and accept all your data and assertions about how space probes are tracked and guided to their destinations, except for one thing. You have assumed that the earth is moving around the sun and rotating on its axis. If correct, the distance variations and diurnal tracking of the probes would occur as you say they do. But exactly the same phenomena would be observed if the rest-of-the-universe, rather than the earth, were moving, that is, rotating around the earth daily and orbiting it annually (the combination motion leading to the precession of the equinoxes, etc). It is a clear case of the relative motion problem in determining what is moving and what, if anything, is not. The geocentric position is that the earth is stationary and everything else, en masse, goes around it daily! So look back to the phrase I have italicized. Everything goes around us daily but only rotates. Or does it merely appear so to the uninformed astronomer? How will rotation (spinning on its axis) get sun, moon, or stars around us daily? Either all these bodies orbit the earth daily or the earth rotates. This relative motion – which we don’t ‘get’ – seems invented to slow everything down to possible speed so we can comprehend it. Is comprehension necessary? =========================== R Sungenis You have misunderstood him (or more apparent is that he wasn’t very clear). He is not saying that the bodies spin on their axis, nor that such spinning would in any way explain Geocentrism. He is referring, rather, to the same thing I illustrated in Diagrams 5 and 6, wherein the sun and the moon rotate daily around the earth with the rest of the universe, but that both the sun and the moon have an independent orbit around the earth such that on each day of the year the sun and moon will be in a different place in the sky relative to the stars. When one adds all these days up, the sun and moon make a circular path through the stars each year, which in the Geocentric system is understood as their orbit around the earth. Incidentally, it seems no coincidence that the moon more or less follows the path of the sun through the stars, and rises and sets roughly in the same place as the sun, only about 50 minutes later each night. =========================== H Gibson I assumed the earth’s motion from the established fact that it obviously catches these probes half the year and then recedes from them at the same rate the other half. The three single direction probes lie in different directions, and their superior conjunctions occur at different times. I included the figures on the orbiting probes (one of them, Galileo, orbiting Jupiter, the rest orbiting the sun, not the earth) to show how differently they are observed from our three tracking points. =========================== R Sungenis All the probe figures you gave (Pioneer, Voyager, Galileo) can be explained using the models that I have provided above. Three tracking posts are necessary in the same way that three Global Positioning Satellites are needed to triangulate a specific spot on earth. =========================== H Gibson Professional astronomers time their orbits and measure their distances. It is all very well to say that observed phenomena would be identical if the earth stood still and everything else moved, but this is impossible not only to prove but to be. The orbiting probes orbit, as intended. But the single direction probes continue in their original directions. They get no closer to each other. No force removes them from their straight courses. They recede from the sun at constant speeds toward the same respective constellations year after year. Do I hear a reproof that I assume that constellations are relatively fixed? =========================== R Sungenis No, as I said above, the probes and the stars are all rotating around the earth. The earth is the only thing that is immobile. =========================== H Gibson Let us skip lightly past the annual superior conjunction of each zodiacal constellation, though it would seem to fit the annual orbit of the earth around the sun far more satisfactorily than the daily orbit of the sun around the earth, and concentrate on the movement of Pioneer-10. It [Pioneer-10] recedes from the sun and on average from the earth at a weekly rate of more than seven million kilometers. Suppose the earth is stationary and Pioneer-10 moves daily nearly 63,000,000,000 kilometers in orbit around the earth in addition to its undoubted radio-beammeasured distal motion. So in 1984, when Pioneer-10 was a mere five billion kilometers away, its daily orbit was only 31,500,000,000 kilometers. In 1978 it did well to orbit 16 billion kilometers daily. But give it another two dozen years and it will double its present orbital speed to about 125,000,000,000 kilometers daily – nearly eight times the speed of light! It already goes nearly four times light speed. Can they really track it? What accelerates it? I rest my case. You may phone and-or FAX me, as I do not have on-line PC service. =========================== R Sungenis In the Geocentric system we account for the high speeds necessary to keep the sun and stars rotating around the earth by noting that all the heavenly bodies are embedded in the aether. Contrary to Einstein’s reluctance to accept an aether-based universe, its existence was proven by Sagnac (1913), Michelson-Gale (1925), Dalton Miller (1933) and Herbert Ives (1943). Einstein did his best to ignore each of these scientists, and they are still ignored today. Instead he opted for Relativity. But you must realize that Einstein’s theory of Relativity was actually a scientific apology for the fact that the Michelson-Morley experiment of 1887 found that the earth was standing still in space. Einstein’s biographer says as much. He writes: Michelson and Morley...The problem which now faced science was considerable. For there seemed to be only three alternatives. The first was that the earth was standing still, which meant scuttling the whole Copernican theory and was unthinkable. (Einstein: Life and Times, p. 109-110) If one depends on Relativity (which is another way of saying that the earth is not immobile) than it will be necessary to have exhorbitant speeds for the stars, some supposedly receding faster than the speed of light, which, ironically, contradicts Einstein’s very theory. The Geocentrist realizes that the only way to account for the necessary speeds is to understand that the aether is the thing doing most of the work in getting the universe to rotate. At Planck dimensions, the aether could easily rotate in 24 hours. In fact, it would need to rotate that fast in order to create enough centrifugal force to stop the universe from collapsing in on itself due to gravitation attraction. The only way conventional science deals with this anomaly is by creating Dark Matter, which they say comprises 99% of the mass in space, an amount which is needed to stop the universe from either contracting or expanding into oblivion. But no one has seen any Dark Matter. It is invented to serve as a fudge factor, just as Einstein’s “Cosmological Constant” did years ago. By the way, I thought you might be interested in the words of someone who worked with NASA in flight mechanics. Regarding the heliocentric/geocentric debate, and in reference to my work in geocentrism, here is what he told one of my critics: As a former Robotics Analyst that worked in MOD (Mission Operations Directorate) at NASA, I can say that I’ve dialogued with folks in Propulsion and Flight Mechanics, and many agree that there would be no way to objectively prove either side (Relativity vs. Geocentrism) by science, logic, or math. I’ll leave you with that to ponder. Thank you, Mr. Gibson, for your entry to our challenge. As you can see, you have not proven that the earth goes around the sun. I hope God will give you wisdom and patience as you contemplate this issue. Please forward my congratulations to your son, Mel, on the film he is making of Jesus’ passion. Please give him my commendations and let him know that we are praying for him and for the success of his current production. God be with you. Robert Sungenis, M.A. (Ph.D. cd) President of Catholic Apologetics International 21 Mar 03 ========================== W Savina If the Sun is going around the Earth, how fast is it going in miles per hour? How far away is the Sun from the Earth ? ========================== R Sungenis That depends on what you mean. On the one hand, geocentrism requires the sun to go around the earth once per day. If the sun is 93 million miles away, this means the sun would have to travel 584 million miles per day, or 24 million miles per hour. A heliocentrist might object that such speed is impossible, since it is 1/27th the speed of light. But this objection can be answered in two ways: (1) Heliocentric science, considering that it must incorporate the sun and earth in a universe that is in a constant movement of expansion, requires some stars to be receding from earth faster than the speed of light. That is why Hubble’s “Constant” has had to be revised from time to time, since they find stars that are farther away than previously thought, but if they are so far away, then they must be traveling faster than light. (Obviously, there is something wrong with a theory which says that stars recede faster than light if the same theory says something cannot go faster than light). The upshot is this: the heliocentric system has just as much, if not more, of a problem with the required speed of heavenly bodies as the geocentric system does. (2) In the geocentric system, the LeSagean concept of gravity is usually incorporated. This concept holds that there are ultramundane corpuscles, or an aether, pervading the whole universe. The existence of aether has already been proven by numerous scientific experiments (Sagnac in 1913, Michelson-Morley in 1925, Dalton Miller in 1933, Herbert Ives in 1943, et al) but Einsteinian theorists (the very ones who tell us that some stars are receding faster than the speed of light) ignore this evidence, since it denies Relativity theory and supports an immobile earth. As it stands, the sun is embedded in the aether, as are all the heavenly bodies. This aether rotates around the earth once per day. The aether wind crossing the earth has been shown by the same experiments mentioned above. Thus, in the Geocentric model, the aether, carrying the whole universe, is the thing rotating around the earth each day, and as it rotates, it carries the stars and sun with it. The sun may have a little motion that is independent of the aether, but the aether is doing most of the movement around the earth. This means that the sun, relative to the aether, is not moving at 24 million miles per hour, but is hardly moving at all. The independent movement the sun makes relative to the aether, however, will allow it to transcribe a path through the zodiac each year. Hence, as the aether rotates once per day around the earth, the sun rotates with it, and the sun will come back to almost the same position each day, except that it will be 1/365th ahead of where it was the day before. As for the rest of the stars, they also rotate with the aether, and thus they are not moving at exorbitant speeds, rather, the aether is rotating. Since the aether is at Planck dimensions, it can withstand such speeds. ========================== W Savina When did the dinosaurs die off ? ========================== R Sungenis Because all the evidence shows that the universe and the earth are quite young, the dinosaurs existed no longer than 10,000 years ago, and most likely died after the climate changed due to the cataclysm of the Great Flood recorded in Genesis 7-9. It is a fact that dinosaur remains and tracks have been found right along side human remains and tracks. ========================== K Cole I’d like to address your response to my challenge. I will focus on your objection to my proof, which was in the first two paragraphs of your response. It specifically challenges Premise (B2) in my proof: ========================== R Sungenis You’re missing one thing. Newton’s law that smaller bodies revolve around larger bodies is true only in isolated systems in which there is one large body and one small body. (In fact, Newton had problems explaining what would happen if a third body, or even a multiple number of bodies, came between two bodies whose mutual force was originally calculated using the inverse square law). But the fact is our universe is not an isolated system. It includes innumerable galaxies. These galaxies directly effect the movement of the sun, which in turn would effect how the sun moves in relation to the earth. ==========================
  21. CASE Well, again, you don’t need to know what gravity is, just its predictable effects and relationships. A little boy doesn’t understand inertia, but he knows that trying to slow down a tank is different than slowing down a skooter. Don’t say they maintain the same orbits, when you already admit that they degrade slightly. ======================== SUNGENIS I only mentioned the “degrading” to show that the Big Bang cosmology of a 13.5 billion year old universe with a 4.5 billion year old solar system associated with heliocentrism has a big problem, since if the degradation of orbit were to be calculated out, the solar system could not be more than 100,000 years old. There is a good book you might want to read on this subject. It is Newton’s Clock: Chaos in the Solar System, by Ivars Peterson. ======================== CASE Another big error you fall into is this idea that such small bodies exert more than negligible attraction across such vast distances in the universe. Because of the momentum of earth, venus or mars have negligible effect on the orbit of the earth, and most definitely God synchronized even these negligible forces to cross at certain times to maintain balance enough for the end of the world. Even solar flares can serve a balancing effect. God creates a perfect ecosystem. ======================== SUNGENIS Yes, I agree. It is the perfect ecosystem. But then that just forces you to include the stars and everything else in the solar system into your “gravitational” formulas. Previously it seemed as if you were trying to treat the earth-moon or earth-sun as independent systems to prove your point about “smaller bodies having to rotate around larger bodies.” If you agree that no system is isolated from any other system, then you must agree that there is a system which can, by mathematics and science, position the sun between the earth and the stars (i.e., geocentrism), and have it obey all the laws you deem necessary. ======================== CASE The geocentric theory says there is one BIG exception – the sun falls towards the earth and kept in orbit by the pull of the earth. ======================== SUNGENIS No, because we don’t necessarily view gravity as a “pull.” ======================== CASE It is certainly a mutual attraction of matter. Magnetics can push if both bodies have magnetics and are close enough, and positioned. But both forces don’t extend very far. We know the weakness of the earth’s magnetic force. ======================== SUNGENIS Yes, it is “certainly a mutual attraction of matter,” but you still don’t know whether the matter is pulled together or pushed together, and no amount of mathematics is ever going to give you that answer, since the math for both is the same. Moreover, this dilemma can’t be explained by magnetism, since even your science asserts that gravity and magnetism are two entirely different things. But let me pursue this just a bit further. Do you know what causes magnetism, Mr. Case? Or do you know why magnets only attract iron and nickel but not any other metal? I dare say you don’t. No one does. It’s as mysterious as gravity. All the scientist can do is measure the force a particular magnet gives off, and he assigns the name “Gauss” to it, but he doesn’t know what makes it do what it does. ======================== CASE In all the observable positioning models listed above, you can punch in the data for why all the planets orbit the sun at one time, and they all fit perfectly to the formula as far as proportion of mass, size, speed and distance. But as soon as you make an exception for any single planet with the very same data and punch it in as though ONLY the sun revolves around it....it becomes such an obvious absurdity to the demonstrable laws of mass, momentum and gravitational pull, that we know certainly that the sun cannot revolve around any single planet within that same system where that same formula is applied. Since the force of gravity increases in some proportion to its mass – picture a giant elephant 10 stories high and a human being holding it by a chain. If the elephant tries to walk in a straight line perpendicular to the man, it will be as if the man never existed. However, if the elephant stood still and the man tied with the chain to the elephant tried to walk perpendicular to the chain, the man would only continue in a circle around the 250 foot elephant with a taut chain. The first consideration is so absurd that actual mathematical figures need not be punched into the associated formulas. Just the drastic disproportion of masses/forces/sizes and fundamental principles of “more is greater than less” tells us it is a matter of utter impossibility in the solar system to apply the same physical laws to all the planets and sun, and then use the same orbital quantities to make the very same sun orbit the earth of comparable mass and size. ======================== SUNGENIS Again, Mr. Case, what you are stating above is precisely the argument I’m using against you. You seem to think that the sun and its planets are in some kind of a universe all by themselves, away from the stars and all the other cosmic forces. But the fact is that there is a whole universe of stars out there which directly influence the forces we see in our solar system. For example, just agreeing with you for the sake of argument that the sun is 330,000 times the size of the earth, in that very system of measurement you are using, there are stars which are 330,000 times the mass of the sun, and sometimes larger. Betelgeuse, for example, is supposed to have a radius of the orbit of Neptune (almost 4 billion miles in diameter). Do you think Betelgeuse, being that massive, might exert some force upon our solar system? Yes, I think you would agree. Now multiply that force by the billions upon billions of stars you see in the sky, stars that circle the earth each night. Now, I can imagine that the force of these stars, working with or counteracting each other, is what helps keep the sun in its path. (Even you would have to admit this is the case, since in your system you already believe that the sun revolves around the Milky Way at 500,000 miles per second, and is held in place by the stars of the Milky Way.) This is why we insist that when Genesis 1 says that God “placed the sun, moon and stars in the FIRMAMENT” (the firmament being a physical object) it was done so in a way that all the billions upon billions of forces from each of these bodies were taken into account so that the system would work. Thus, even in your system, it is not an “absurdity” to say that the sun could rotate around the earth, for obviously it is not the earth alone that is involved in the forces that make the sun move. ======================== CASE The stars don’t have any effect because of their distance. If they did have such a power, our sun would show it because it is so close. Funny how you doubt the sun’s size and so easily accept “Betelgeuse”. I still don’t agree because the distance by formula shows the pull is negligible. ======================== SUNGENIS You say “The stars don’t have any effect because of their distance.” Can you prove that to us, Mr. Case? Isn’t your system the one that has the sun rotating around the Milky Way galaxy at 500,000 mph? So what in your system is keeping the sun in its orbit around these stars? Is it not, according to your previous formula, the gravitational attraction of the “larger” Milky Way on the “smaller” sun? So apparently, even in your system, the stars are “close enough” to effect the movement of the sun, yet you claim that the stars are not close enough in my system. How so? As for your statement “Funny how you doubt the sun’s size and so easily accept Betelgeuse,” you will notice that I said “in that very system of measurement you are using.” I didn’t say I accepted the size they give to Betelgeuse, I said that, in your system (the Big Bang, heliocentric system) Betelgeuse is purported to be as large as the radius of Neptune. Incidentally, the currently accepted size of Betelgeuse is dependent on whether Newton’s third law of motion has no discrepancies. There are studies being done presently to determine whether his third law has any discrepancies, as is in fact valid. Already the Cavendish Torsion balance has discovered a .37% discrepancy. ======================== CASE The sun is in a relatively tight and well-formed circular path (or the earth is). There is NO possible way that a force whose center is the earth could pull the sun’s momentum. To try to gratuitously invent some “unknown force is doing it” is one thing, but it actually VIOLATES known demonstrable laws of the physical world. It doesn’t matter if the sun in 2,990,000 times as massive, the proportions of nature make it impossible for the earth to attract the sun. And if the earth HAD THAT STRENGTH, the moons and other planets would immediately suck right to it. It is absurd to say that force is the earth. ======================== SUNGENIS I answered this above. The force is not the earth. The force is the star system surrounding the sun and earth. ======================== CASE Just take the moon and sun into comparable consideration. Consider both the moon and sun revolve around the earth. The fact that they make an arc around the earth means that the earth has the force to pull both of them away from going in a straight line that their momentums insist upon going, and towards itself [the earth]. There is no question that the moon is smaller than the earth and closer and that it revolves around the earth. It is an UTTER IMPOSSIBILITY to apply the same physical laws such as to suggest that the same earth force also takes an object 330,000 times its mass, at a phenomenally LARGER distance away, at a phenomenally LARGER velocity and suggest the strength of the earth can pull the sun’s momentum out of a straight path. If the earth had such a pull, all the planets of comparable size would vie to pull the same sun, e.g. Venus would try to pull the sun towards itself, etc. One would have to resort to saying it is a constant miracle and intervention of God rather than having anything to do with physical laws. (But that would be relegating it to a mystery of faith, and we know the Church has allowed the heliocentric model. To say otherwise would be to accuse Christ’s Church of allowing heresy for centuries). Or resort to gratuitously asserting there is “some other” force in nature associated ONLY with the earth that gives it such gravitational power over the sun. But if it had such a phenomenal force, it would likewise have its effect on the other planets.....and it doesn’t. At every step it is an absurdity to think the sun goes around the earth, or any other planet. The heliocentric solar system is the only alternative to this absurdity, not just by syllogistic default, but by positively fitting the established proven physical laws of momentum, velocity, distance and gravity of nature – consistently – among all the other bodies and their observable facts. ======================== SUNGENIS Again, all your assumptions are based on an unproven and unexplained theory of gravity, as well as an ignoring of all the other forces occurring in the universe that act upon our solar system. Unfortunately, this is the same problem into which modern science has gotten itself. Due to their sole reliance on mathematics to explain forces (as they do in the Newtonian and Einsteinian systems), they don’t have a physical explanation for gravity, or any other force they encounter. As a result, their mathematical calculations not only do not represent reality, but they can only help when applied in isolated systems of one, two or three bodies. When a fourth, fifth or one hundred billionth body is added, they have no clue how it all works out. But we know how it does. The only way it can work out is when an infinite intelligence, God, places all the objects of the universe in their necessary locations in order that the forces caused by each one will completely balance out and result in the solar system we have today for earth. In that system, there is no reason, considering the principles I laid out for you in this essay, why the sun cannot be revolving around the earth. Thank you for your challenge. Robert Sungenis ======================== CASE In this format, I have repeated myself enough. What I have said above suffices for your remaining paragraph. You indicated at the outset of our exchanges that an explanation “must be direct, observable, physical, natural, repeatable, unambiguous and comprehensive.” You explain properly there what “demonstration” consists of, but then you respond to the challenge by denying “demonstration”! All the physical laws I mention involving movement, mass, inertia and orbital calculation can be demonstrated as physical laws – then you gratuitously call them theories and dismiss them. The very fact that they can PREDICT an outcome beforehand shows they can demonstrate the laws to be valid. A sun of such mass and speed revolving around a tiny earth not only shows to be inaccurate with the proven physical laws, but GROSSLY violates those laws to an insane proportion, especially as you try to apply the same force attributed to the earth to the surrounding planets in free-moving, empty space. ======================== SUNGENIS All your objections have been answered in my preceding paragraphs. ======================== CASE I see that your fundamental error on this subject is that you do indeed consider it intrinsically connected to the Faith, and this is the reason why you give precedence to literal interpretation over demonstration, and come to doubt the most common experience of physical laws and the fact that they have been proven by demonstration to predict outcomes. ======================== SUNGENIS Quite the contrary. I give precedence to literal interpretation because that’s what the Catholic Church has always done, unless irrefutable and proven evidence forbids such an interpretation. As for “demonstration,” I’ve already shown you that mathematical formulas don’t demonstrate which model is correct, since both models use the same math. ======================== CASE You are going to have to understand the difference between something that is a danger to the Faith intrinsically as compared to extrinsically. That which is extrinsically a danger can cease to be a danger once the extrinsic circumstances change. That is why the Church condemned anyone to teach it at first, and then the extrinsic dangers faded away. The Church obviously has allowed Catholics to teach and believe it for centuries. The Galileo affair is THE most prominent case involving “the visible universe” and Scripture, and we see in 1893 when Pope Leo XIII wrote his encyclical “Providentissimus Deus”, precisely on “the study of Sacred Scripture”, he particularly had this in mind. How could he have not foremost had it in mind?” The sacred writers, or to speak more accurately, the Holy Ghost “Who spoke by them, did not intend to teach men these things (that is to say, the essential nature of the things of the visible universe), things in no way profitable unto salvation.” Hence they did not seek to penetrate the secrets of nature, but rather described and dealt with things in more or less figurative language, or in terms which were commonly used at the time and which in many instances are in daily use at this day, even by the most eminent men of science. Ordinary speech primarily and properly describes what comes under the senses; and somewhat in the same way the sacred writers –– as the Angelic Doctor also reminds us –– “went by what sensibly appeared,” or put down what God, speaking to men, signified, in the way men could understand and were accustomed to. The unshrinking defense of the Holy Scripture, however, does not require that we should equally uphold all the opinions which each of the Fathers or the more recent interpreters have put forth in explaining it; for it may be that, in commenting on passages where physical matters occur, they have sometimes expressed the ideas of their own times, and thus made statements which in these days have been abandoned as incorrect.” Would anyone say that, with the notoriety of the historic Galileo controversy, that this pope had something else in mind and failed to note to his readers that it doesn’t pertain to the Galileo affair? Who would know what then he WAS referring to that “sensibly appeared” to be one way, but really wasn’t, in the physical universe? In 1893, when the whole Catholic world believed and taught Heliocentrism and dismissed Geocentrism, it is absurd to think the pope simply failed to take the occasion to correct the error to protect divine revelation! How could the divine institution of Christ’s Church allow centuries, and even more generations of Catholics living and dying, to come and go, and say nothing in condemnation? Why on the other hand would the Church repeatedly condemn Liberalism decade after decade, and other errors, but never repeat a condemnation since the time of Galileo? Simply because the extrinsic danger to the faith was no more. But now we have a handful of laymen at the end of the 20th century, smarter than all the popes and are ringing the bell to save people from danger? The implication of accusation against the Church cannot be avoided. ======================== SUNGENIS First, you cannot prove that Leo XIII was speaking specifically about the heliocentric theory you are adopting. It would be hard to prove since the “Angelic Doctor” to whom Leo refers, namely, Thomas Aquinas, believed in geocentrism, not heliocentrism. So it is rather difficult to conclude that Leo is using as proof the very man who did not apply “what sensibly appeared” to be geocentrism? Second, let’s suppose, for the sake of argument that Leo is speaking about cosmology (heliocentrism or geocentrism). If that is the case, he is saying nothing different than what Robert Bellarmine had said regarding this issue. As we noted, Bellarmine told Galileo that: “If there were a true demonstration that the sun was in the center of the universe...and the earth circled the sun, then it would be necessary to proceed with caution in explaining the Scripture which seemed contrary, and we would rather have to say that we did not understand them than to say that something was false which has been demonstrated.” So this objection was already addressed by the Church. In other words, IF science had proven its case regarding heliocentrism, the Church was ready to reinterpret Scripture. But you will notice above that Leo DID NOT SAY that science had proven its case but only that “IT MAY BE THAT” such is the case. The words “MAY BE” are vitally important, since they mean that Leo is not committing the Church to saying that the Fathers DID merely “express the ideas of their own times,” but only that if science proved the literal interpretation of a passage to be inappropriate, THEN, and only then, did the Fathers “express the ideas of their own times.” Third, Leo’s statement “which in these days have been abandoned as incorrect” does not identify who it is that did the “abandoning.” It is just a passive statement that, in his day, there was a consensus that a certain view of cosmology has been abandoned. He doesn’t say the Church has abandoned it. That leaves a consensus of scientists as those who have “abandoned” it. But is Leo saying that these scientists are necessarily correct? No, not by any means. He is only saying what Bellarmine said, that is, if the scientists are correct, then the Fathers were “expressing the ideas of their own times.” But you and I both know that what the consensus of science knows today it may be modified or overturned tomorrow. For example, just a few months ago, two scientists from Australia made headlines, and their papers were published in the esteemed scientific journal, Nature, showing that their experiments have overturned Einstein’s theory of Relativity. Many other studies have been done with the same result. If anyone wants a good history of how science keeps changing its views, read Stephen’s Hawking’s book, A Brief History of Time. There are a number of books available which show the same history. Studies of the inside of the atom have made science appear like children playing in shooting gallery. As of this day, they still don’t know the physical model of the atom. They even have a name for this. It’s called the Heisenberg Uncertainty Principle. In fact, what science tells us today, if we are honest with the data, is that not only is the heliocentric theory unproven, but much evidence shows that geocentrism is the more correct model. For example, astrophysicist V. P. Varshi found in 1975 that all 348 Quasars were positioned in successive concentric circles with the earth as the center. William Tifft showed that the red-shifts of galaxies occurred in certain preferred values, and William Napier showed that this was precisely 37.5 km/sec with the earth as the center of the values. A study was done in the 1970s at Cal Tech which, after adding all the known coordinates in space, found that they all canceled each other out and left earth in the center. The team said they were “horrified” by the results, because they supported geocentrism. Speaking of “ecosystems,” what they also found was that the periodicity of extra-galactic redshifts (37.5 km, or 1:1.23) was the same ratio that appeared in the spacing of the planets in our solar system, and in a study done independently by Brazilian, Italian, French and Croatian scientists, the same ratio in the Bohr model of the atom. I could give you much more scientific information, but the point is made that there is a lot of scientific evidence to support the geocentric position. Much of this information was not known in the time of Leo. Leo was on the upswing of Lyell, Darwin, James, and many other scientists who were making it appear that science was disproving the Bible. In fact, being in the midst of the Evolution crisis, Leo’s statement in Providentissimus Deus regarding the Fathers “expressing the ideas of their own times,” was probably more pointed toward Evolution, since it was accepted by the scientific community that Lyell’s long-age geologic column was a fact. If that theory turned out to be true, Leo wanted to prepare the world for the fact that when the Fathers spoke of “days” in Genesis 1 they may have meant long ages. Of course, we know today that Lyell’s theory is just that, a theory. Sedimentology experiments and Mt. St. Helens have proven that strata can form in a matter of days. In addition, in 1982 the world’s leading evolutionists, Gould and Eldridge, admitted that the intermediate fossils scientists hoped to find in Lyell’s strata simply didn’t exist. I’m sure I don’t have to tell you of all the scientific evidence which we have found since the time of Leo which directly contravenes the theory of Evolution. Thus, although you make Leo’s statement in Providentissimus Deus to be a concession on the part of the Church, it may not be that at all. Leo chose his words very carefully, and as such, his statement is conditional, not absolute. ======================== CASE Lest anyone say that I am mistakenly accusing you are treating this subject as if it were “of faith”, let me point out that you have specifically said Heliocentrism is a danger all around: “it directly affects how we view God, Scripture, the Church, and Modern Man.” ======================== SUNGENIS Yes, it most definitely does affect how we view God, Scripture, the Church and Modern Man. I stand with St. Robert Bellarmine. He said that asserting an unproven scientific theory as fact does “injury to our holy faith and renders the Holy Scriptures false.” I’ll stand with the three popes who condemned any person who said it was incontrovertible fact that the earth moves. I’ll stand with Solomon and the Fathers who said that the sun moves and the earth does not. And I’ll stand by the scientific evidence which does the same. ======================== CASE More evidence that you treat this as pertaining to faith is your comparison: "If someone wants to argue that the Catholic Church takes Matthew 26:26 literally because the Tradition of the Church as far back as the early Fathers binds us to do so; well, the same can be said about Geocentrism, since all of the Fathers, without exception, were Geocentrists, even in the face of several Greek astronomers (Aristarchus of Samos; Heraclides of Pontus) who were already advocating Heliocentrism one thousand years before Copernicus.” As they say, you are comparing apples and oranges. The Church Fathers only “comprehended” what they read about the sun, like any other Christian, by the default literal interpretation AND their own senses, but they did not consider it a religious belief of divine revelation. Matt 26:26 was divine revelation that was not of the senses. One is of Faith, the other is not. ======================== SUNGENIS If you claim such, perhaps you can show us where the Fathers state that they were merely “comprehending” and did not consider it a religious belief of divine revelation. I beg to differ with you. In the face of Greek opposition, the Fathers understood geocentrism as precisely a point of divine revelation, since that is what Scripture said. The Fathers knew the alternatives (heliocentrism) and they knew that such explanations were entirely plausible, but they rejected those plausibilities, just as Robert Bellarmine did 1000 years later, since no one had proven that the plausible was indeed the truth. If there was no competition from the Greeks for an alternate cosmology, you would have a point. But if you read the writings of the Fathers on this subject you will find that their belief in Scripture teaching geocentrism was used as a polemic against the Greeks who had NO divine revelation to guide them. ======================== CASE It is merely pious faith, and safe, to hold to the literal interpretation in lieu of demonstration & authority to the contrary. It is Protestant to hold to the literal interpretation against demonstration and the authority of Christ’s Church which has clearly allowed it to be taught for centuries without condemnation. ======================== SUNGENIS As I have shown quite easily, Mr. Case, you have only “demonstrated” that your theory uses the same math as mine. You haven’t “demonstrated” heliocentrism as a proven fact, and that was the essence of this challenge. Second, what “authority of Christ’s Church” are you referring to? It certainly can’t be Leo, for he gave no mandate against geocentrism. As we saw, he merely said “IT MAY BE THAT” the Father’s spoke in a certain way, but that’s the same thing Bellarmine said. In fact, you can search far and wide in the Catholic Church’s documents and you will find nothing that specifically countermands the decrees of Pope Urban VIII and Alexander VII on the geocentric issue. All you have is that books about the Copernican theory were secretly removed from the Index in 1757 and 1835 when the Index, for all books on all subjects, no longer had the same requirements it had in 1616, but that certainly is not a statement asserting the veracity of the Copernican theory. In fact, in 1822, when Pius VII allowed Canon Settele to teach the Copernican system, his based his permission not on heliocentrism as fact but merely as the “general OPINION of modern astronomers.” As far as authority goes, the dilemma is more on your side of the fence, since you are faced with a papal bull by Alexander VII which specifically denied the “motion of the earth,” whereas Pius VII merely allowed Settele the option to teach the “opinion” of modern astronomers. Allowing someone to inform other people about the “opinions” of someone else is hardly an official endorsement of the opinion, but a papal bull, as many argue, is infallible. ======================== CASE Literal interpretation is a rule of thumb for the layman studying Scripture when not being sure of what the Church allows. Once we know what is allowed, the literal interpretation is no longer a rule if the Church obviously allows to the contrary. Think about it, where was heliocentrism mentioned when the Church repeatedly condemned the myriad errors of Liberals in the 19th century? Where was St. Pope Pius X to mention it when enumerating the list of “modern” errors? ======================== SUNGENIS Other than Pius VII allowing Settele to write about the “opinions” of modern astronomers, what official evidence do you have that “the Church obviously allows to the contrary.” I can tell you this. There is no official statement from the Church that specifically allows one to interpret the historical statements in the Bible other than literally. If you have one, I’d like to see it. Even the Catholic Catechism is very cautious about this. For example, in para. 337 it states: “God himself created the visible world in all its richness, diversity, and order. Scripture presents the work of the Creator symbolically as a succession of six days of divine ‘work,’ concluded by the ‘rest’ of the seventh day.” Since the word “symbolically” is used, some have concluded that the Catechism is teaching that Genesis 1 is merely symbolic. But that is not what the Catechism says. The only words that are put in quotes in para. 337 are “work” and “rest,” since they are the only words we know are symbolic, for God does not literally “rest” and “work.” As for interpreting literally, rather than your self–imposed rule, the Church’s mandate from Pope Leo in Providentissimus Deus is the following: “…not to depart from the literal and obvious sense, except where reason makes it untenable or necessity requires.” So far in your challenge, Mr. Case, you have shown little in the way of making a geocentric interpretation of Scripture “untenable,” nor have you shown anything that makes heliocentrism a “necessity.” On that basis, then, Leo’s mandate requires you to interpret Scripture literally until you do find such irrefutable evidence. Notice also, as opposed to the statement you cited earlier from Leo, the present statement does not include the words “it may be that.” Rather, Leo makes it clear that we are NOT, with no conditions, to depart from the literal and obvious sense, unless it is untenable. Do you really think, Mr. Case, you have enough knowledge of science to make geocentrism “untenable” (i.e., without any plausibility at all)? Everything you have offered in the way of science, math and logic has an alternate interpretation and explanation. ========================== CASE Now that we have been back and forth a couple of times over my original submission, things are certainly becoming clearer to me why you maintain what you do. However, I thought I would now take the opportunity to consolidate the subject matter, as it has become burdensome to follow multiple points in the conversation-per-paragraph format, especially for those who wish to follow along. ========================== SUNGENIS The only problem with your “consolidation” is that you have conveniently eliminated some of the more important challenges I have given you to your own view of things. For the sake of our audience, I will bring up a few of them again in this post. As for the others, you can look back at the previous posts. ========================== CASE 1. The Playing Field: You have advertised that you have "decided to make a level playing field" by offering a large sum of money. I think you ought to alter that description. The money is certainly an added motivation for accepting the challenge. However, the actual "playing field" can never be level when the one challenged is also the judge. This is hardly "level"; it merely confirms your seriousness by giving yourself something to lose. I certainly accept the conditions, I just think you should omit the faulty description for the public. ========================== SUNGENIS You entered this challenge fully accepting the “description” that I was the final judge of the issue. If you didn’t think it was acceptable at that time, you could have declined to enter the challenge on that basis. Crying foul now says more about your insincerity than mine. ========================== CASE 2. The public challenge states, By "proof" we mean that your explanations must be direct, observable, physical, natural, repeatable, unambiguous and comprehensive. We don't want hearsay, popular opinion, "expert" testimony, majority vote, personal conviction, organizational rulings, superficial analogies, appeals to "simplicity," "apologies" to Galileo, or any other indirect means of persuasion which do not qualify as scientific proof. I have seen so far that you have violated your own rules by presenting things plainly against your stated idea of what "proof" actually consists of. I will get into details and examples further on. One such glimpse is your statement on the front page of your Web site mentioning the, "socalled "laws" of physics" and how they are "not laws at all", but "unproven theories." ========================== SUNGENIS The “Challenge” said YOU were to provide proof for the heliocentric cosmology, not me. If you claim, as most of your persuasion do, that you can prove heliocentrism based on scientific evidence, then you are REQUIRED to provide the proof. I, on the other hand, simply state that Geocentrism, because it can provide all the necessary mathematical and scientific requirements, is a natural rival alternative to your system. I maintain that the one that is true cannot be proven from the scientific evidence, and that is why I offer no proof. I only offer alternatives using the same Newtonian mechanics that you use for your system. The only reason I appeal to scientific evidence is to show the reader that, despite claims to the contrary, the Geocentric system can be shown to function well under such scientific rubrics, and thus science cannot be used to dismiss Geocentrism. In turn, I simply direct the reader to Scripture, the Fathers, and the authoritative statements of the Catholic Church for the authority they need to decide which system (heliocentrism or geocentrism) is indeed correct. ========================== CASE 3. Concessions you made in the following two recent excerpts... - "Mr. Case, you have shown little in the way of making a geocentric interpretation of Scripture "untenable". - "geocentrism is the more correct model" This is a clear admission that you think I have shown "some" things in the way of making geocentrism untenable, and that you think my position is "correct" albeit LESS correct than yours. I think this speaks for itself without further comment. ========================== SUNGENIS The only thing the above statement speaks of is your presumption. I in no way meant, and certainly did not say, that you have made “geocentrism untenable.” When I say “more correct” I am drawing the reader’s attention to two things: (1) of the two systems, the only one supported by the three witnesses I described above (Scripture, Fathers, Popes) is the geocentric model; and (2) the geocentric model does not depend on the unproven theories of Relativity and Evolution and all the ramifications they entail. The geocentric system is a simple, straight forward model that makes use of all the known components in the science of physics. ========================== CASE 4. You stated that I believe, "Earth to be moving in an a-centric universe". Actually "acentric", but I have never used that concept in all that I have written. Everything material that exists certainly has a center, but that is irrelevant in considering the science of whether the Earth revolves around the sun, or vice versa. ========================== SUNGENIS You believe there is a center to the universe, Mr. Case? Then where is it, and how do you determine it? Making assertions is easy, proving them is another story altogether. The fact is, if you believe that everything in the universe is in motion (which is not an option, but is required by the heliocentric system), then you have absolutely no way of determining the center of anything. Geocentrism, on the other hand, makes it easy for us. The center is the earth, as even recent astronomical evidence has shown us (e.g., the studies of astrophysicists Varshni, Cal Tech, Napier, Burbidge, Karlsson, Neto, Agnese Festa, Nottale, Rubcic, et al, and to which you have provided no response). What must be understood is that, by Einstein’s own admission, the theory of Relativity (which purports that there is no center to the universe, for that’s what “relativity” means) was formulated to answer the 1888 Michelson-Morley experiment that demonstrated that the earth was standing still in space, and thus serving as the natural center of the universe (and to which you did not respond). ========================== CASE 5. During the course of this exchange you attempt to ascribe beliefs to my position that I have never professed, and then fault me for "mere assumptions rather than cold hard facts". Things such as the theory of "dark matter" and "the universe collapsing". They might be held by some heliocentrists but they are not part and parcel of heliocentrism. There is no direct relationship to the subject of our solar system, as to whether the Earth or sun moves. I do not hold those things. I have only presented DEMONSTRABLE laws of physics which ironically you have swept under the carpet as mere "theories" by countering with hypothetical assumptions of your own that are NOT demonstrable. More details further on. ========================== SUNGENIS First, if you don’t want to adopt the prevailing theories of heliocentric science, that is your prerogative, but the consequence of that is that you have to come up with your own theory of how the universe is put together. I haven’t seen you offer one. You cannot deal with this issue in a vacuum, as if our solar system is isolated from the rest of the universe. Everything is connected to everything else. You cannot avoid the fact that, in your system, the sun must be moving around the galaxy, and the galaxy must be moving around still larger clusters, and so on. That being the case, you must answer the nature of the red shift, Olber’s paradox, 3 degree Kelvin radiation, recession of galaxies, instantaneous effects of gravity over long distances, and about a dozen other difficulties. Without an explanation for these things, you don’t leave room for a heliocentric solar system, except in theory. But if it’s mere theory, then you’ve lost the “Challenge,” since it requires proof of heliocentrism. ========================== CASE Apart from demonstration as proof, the issue of the Church is a distinct issue that should be addressed in its own space. In fact, for Catholics, it should be the primary issue before going on to details of secular science. You have yourself given considerable time to it in this exchange. Therefore, I am going to speak of these matters first before addressing secular science. Intrinsic or Extrinsic Danger to Faith? I have mentioned this distinction but you have skirted around it. That which is INTRINSICALLY dangerous to the Faith can never be otherwise, and can never allow even the possibility that it could be otherwise. For example, the truth that Our Lady is in heaven, body & soul, is a truth that can never be otherwise. This means that one could never suggest even the "possibility" that we can find any remains of her body on Earth. To allow for the "possibility" is to doubt it, which is heresy, and can never be otherwise. But while a "denial" of a truth like that is a heresy, there are lesser condemnations for things which are merely "dangerous" to the faith, and these are heretical. The practice of the Church ascribed "heretical notes" to some things merely for being "rash", "scandalous" or "offensive to pious ears". Such discipline is necessary because the Church doesn't always make an immediate and final decision on whether the danger is extrinsic or intrinsic. The matter must be looked into. Some things are EXTRINSIC dangers to the faith, depending on what the Church decides. When it is truly a case of an extrinsic danger to the faith, the danger depends on whether extrinsic circumstances can change. Knowledge can overcome rashness, scandal and the offense to the ears of the pious. One excellent example is the theory of the "Antipodes" which parallels that of the controversy about the Earth and sun. Here is an excerpt from the 1913 Catholic Encyclopedia: Speculations concerning the rotundity of the Earth and the possible existence of human beings "with their feet turned towards ours" were of interest to the Fathers of the Early Church only in so far as they seemed to encroach upon the fundamental Christian dogma of the unity of the human race, and the consequent universality of original sin and redemption. This is clearly seen from the following passage of St. Augustine (De Civitate Dei, xvi, 9): As to the fable that there are Antipodes, that is to say, men on the opposite side of the earth, where the sun rises when it sets on us, men who walk with their feet opposite ours, there is no reason for believing it. Those who affirm it do not claim to possess any actual information; they merely conjecture that, since the earth is suspended within the concavity of the heavens, and there is as much room on the one side of it as on the other, therefore the part which is beneath cannot be void of human inhabitants. They fail to notice that, even should it be believed or demonstrated that the world is round or spherical in form, it does not follow that the part of the earth opposite to us is not completely covered with water, or that any conjectured dry land there should be inhabited by men. For Scripture, which confirms the truth of its historical statements by the accomplishment of its prophecies, teaches not falsehood; and it is too absurd to say that some men might have set sail from this side and, traversing the immense expanse of ocean, have propagated there a race of human beings descended from that one first man. This opinion of St. Augustine was commonly held until the progress of science, whilst confirming his main contention that the human race is one, dissipated the scruples arising from a defective knowledge of geography. Had this "fable" been prominently promoted to the public as a fact by a well-to-do Catholic, the Church could likewise have condemned it as heretical. It would have certainly caused disruption for being rash, scandalous or offensive to pious ears when it was accompanied by no real proof and knowledge of geography. Though many could have originally suspected it was intrinsically a matter of faith, we know today that it would have been only a danger extrinsically and not in itself. Therefore, had the Church condemned it as heretical, the Church would be correct insofar as there was a "danger", yet though the Church could have been wrong as to the "fact" of the Earth being round, such mistaken fact is not intrinsically connected to the faith, and does no harm to infallibility (for the sake of argument, if infallibility could even be brought into it at all). ========================== SUNGENIS The spherical shape of the earth was not an issue in the 16th century Galileo crisis, and neither popes nor the Sacred Congregations ever had to decide on the shape of the earth, and thus your example is not germane to this subject. The fact remains that, in dealing with the specific subject of whether the earth goes around the sun, three popes gave some of the most dogmatic teachings we have ever had on a matter of scientific interest. When they gave their decrees, they stated quite plainly that heliocentrism was an “heretical” concept, and thus, they, not I, made it a matter of faith. And despite Paul VII’s allowance for heliocentric “opinions” to be taken off the Index, none of those three popes’ edits have ever been abrogated or nullified. Until a pope or Council in the future states that the decrees by Paul V, Urban VIII or Alexander VII are null and void, then we are stuck with what they said. Irrespective of the Index, which history shows changed over time, the fact remains that these popes said heliocentrism was “heretical” and “contrary to Scripture.” Those statements stand alone. Unless they are nullified by an even higher authority, then we are bound by them. ========================== CASE Another prominent example of an extrinsic danger is the sin of "usury". The Church has repeatedly condemned such as unjust and sinful, but as we see history progress, its condemnation became less and less as the economy changed substantially to favor industry and investment. It was a sin of injustice, but the sin was tied to the changeable economy of secular society. The Church flatly condemned the practice in the context of the economic system then presently in place, without speculating that a substantial change in the economy would take place in the future. But later, as the economy of man made it more and more easy to invest money for a return, the sin of usury became less and less prominent and today is almost unheard of. Usury was an extrinsic evil that could once again become prominent were the economy of man to change once again to a point where investments were no longer commonplace. We don't today look to former condemnations of usury when it is clear what the Church has since permitted charging interest. ========================== SUNGENIS The issue of Usury is not germane either, since later popes, when confronted with prior issues regarding usury, gave definitive and formal written judgments to relax certain of its provisions. Examples of further papal decisions on usury were made by Pius VIII (1830) and Gregory XVI (1838). (Denzinger 1609, 1610). There were no such official reversals or modifications of the edits of Paul V, Urban VIII and Alexander VII regarding Geocentrism, save for the right to publish the “opinions” of various astronomers by Pius VII. ========================== CASE Now, the very fact that St. Robert Bellarmine allowed for the possibility of demonstration for the heliocentric theory, shows that the danger he was concerned about was most certainly not an "intrinsic" danger, especially because we have heard nothing against St. Robert's allowance, nor against the approval of Copernicus' book by a pope. When the Church condemned Galileo and this error, She was condemning the dangers against the faith for extrinsic reasons in the context of its rashness, lack of proof, how it would disturb the public, and in context of the serious heretical atmosphere of the Protestant revolts. ========================== SUNGENIS First allow me to address St. Robert. This is what he, as head of the Sacred Congregation, wrote to Galileo: We, Robert Cardinal Bellarmine, having heard that Signor Galileo was calumniated and charged with having abjured in our hand, and also of being punished by salutary penance, and being requested to give the truth, state that the aforesaid Signor Galileo has not abjured in our hand not in the hand of any other person in Rome, still less in any other place, so far as we know, and of his opinions and teachings, nor has he received salutary penance nor any other kind; but only was he informed of the declaration made by his Holiness and published by the Sacred Congregation of the Index, in which it is stated that the doctrine attributed to Copernicus – that the earth moves around the sun and that the sun stands in the center of the world without moving from the east to the west, is contrary to the Holy Scriptures and therefore cannot be defended nor held [Latin: non si possa difendere ne tenere]. And in witness of this we have written and signed these presents with our own hand, this 26th day of May, 1616. Robert Cardinal Bellarmine. (Cited in Antonio Favoro’s Galileo e l’Inquisizione, Documenti de Processo Galileiano, per la prima volta integralmente pubblicati, Florence, 1907). Notice that Bellarmine refers to “the declaration made by his Holiness” (i.e., the Pope) that Copernicanism is “contrary to the Holy Scriptures and therefore cannot be defended nor held.” Thus, I don’t know what you are referring to when you appeal to “St. Robert's allowance.” Perhaps you are misconstruing Bellarmine’s statement to Galileo that he [bellarmine] had received no proof from Galileo of an alternate cosmology. Moreover, Bellarmine’s wording in the above paragraph does not speak to merely “extrinsic” issues as you claim. Telling someone that his views are “contrary to the Holy Scriptures” goes right to the bedrock of the INTRINSIC nature of doctrines that are harmful to the faith. Is there anything more intrinsic than being judged as “contrary to Scripture” by the highest office in the land? If there is, please tell me what it is. If it’s scientific “proof” you claim will rescue you from these papal edits, then please, show us the proof you have. All you have shown so far are mathematical equations, but they prove nothing except that you can add and subtract. ========================== CASE God's order requires us to KNOW before presenting something publicly as a fact. You can see by the quote above that St. Augustine allowed for demonstration also, but specifically made mention that those who promoted it did not "possess any actual information". This was also the case with Galileo and why St. Robert rejected it because there was nothing to demonstrate it. However, when something is intrinsically against the faith, you simply cannot suggest the "possibility" of demonstration against it. ========================== SUNGENIS Since it stood in Bellarmine’s mind that Galileo had presented NO proof, then Bellarmine’s offer to Galileo was merely gratuitous. This is proven by the fact that the above declaration I quoted from the Sacred Congregation makes no mention that Galileo would ever find any proof. The above declaration simply says that Galileo’s proposal was “contrary to the Holy Scriptures,” period. If Bellarmine, or the Pope who commissioned him, had ever seriously considered that Galileo, or anyone else, could have found proof for Copernicanism, they surely couldn’t have written the above declaration as matter of factly as they did. You might judge their words as quite audacious and ill-informed, but apparently they didn’t think so. Bellarmine was no fool. Surely he was aware that if someone DID have proof for Copernicanism, now or in the future, he could not say the things he said in the above quote. But the fact is that he doesn’t even suggest the possibility of disproving an immovable earth in the above declaration. Read the document very carefully. There is absolutely no equivocation in his, or the Pope’s, words. That’s what you have to deal with. If you can find us one pope who officially and specifically abrogated the above declaration, showing us that he admits and acknowledges a formal error in Paul V, Urban VIII and Alexander VII and dogmatically reverses their decisions with a papal bull of his own, then you’ve won this debate. But until then I’m afraid you’re stuck with what Bellarmine wrote above. ========================== CASE The most significant factor of all, for those with a proper Catholic sense, is that the Church obviously has permitted the heliocentric theory to be taught to Catholics for centuries. This means that the extrinsic dangers ceased to exist as science advanced, just as with the theory of the Antipodes. Catholics have universally been taught it with not the slightest objection from the Church, while other relatively obscure errors were repeatedly condemned by the Church associated with Liberalism and Modernism. Heliocentrism was taught widely to all Catholics in the schools yet there were no historical condemnations by Popes, Saints or theologians stating that the centuries-old disciplines were in effect. The situation was like that of usury. The fact that the books promoting Copernicus' theory were long ago deliberately removed from the Index is plainly significant. I object to your characterizations that they were "secretly removed" from the Index. That they were gone was publicly discernible; nothing "secret" about it. The Church decided to do it for a reason, which is obvious....the danger no longer existed. As St. Thomas said, "it would be blasphemy to say that the Church does anything in vain" (Supp. Q. 25. A.1) ========================== SUNGENIS You say “that the Church obviously has permitted the heliocentric theory to be taught to Catholics for centuries”? That is a misrepresentation. I know of no official papal or conciliar statement which says, in effect, “we permit the teaching of heliocentrism.” I know of no official papal or conciliar document that denies that the earth is immovable. The only thing certain prelates have done is given a passive acknowledgment of the competing theory of heliocentrism. But passive acknowledgement doesn’t prove anything, nor does it annul previous papal edits. The Church today passively acknowledges and permits almost all of its seminaries and universities to teach that Scripture is full of errors of historical fact, but that doesn’t mean that there are errors in Scripture, nor that the Church has ever actively and dogmatically taught that there are errors in Scripture. The Church allows the same institutions to hold and teach evolution, but that doesn’t make evolution either ecclesiastical or Scripturally valid. As for your quote from Aquinas ("it would be blasphemy to say that the Church does anything in vain"), you might do well to apply that to the words of Paul V who said that Copernicanism was “absurd, false in theology, and heretical, because absolutely contrary to Holy Scripture,” or Urban VIII’s words stating “the error and heresy of the movement of the earth,” or the papal bull of Alexander VII which condemned “all books teaching the movement of the earth and the stability of the sun.” When you can come up with a papal bull of like-authority to that, which states specifically that Copernicanism is no longer an “heretical” concept, then you have something to talk about. Until then, all you have is one pope who allowed “opinions” to be published (Paul VII), but who in no official statement repudiated or annulled the solemn edits of his predecessors. ========================== CASE The sensus Catholicus knows the implications – to say the former disciplines were still in effect despite the clear practice of the Church for centuries is to plainly condemn the Church for not protecting the Catholic faithful as they were universally being taught the heliocentric model! ========================== SUNGENIS Oh, you mean like they are protecting the flock from reading the footnotes of the New American Bible that they endorse which contains page after page of accusations that Scripture is full of historical errors? Or protecting the flock from the wayward ideas of Teilhard de Chardin, Karl Rahner, Edward Schillibeeckx, and every other liberal theologian who denies basic tenets of the faith? Or who allows immoral prelates like Mahony, Weakland and many others to turn our dioceses into Sodoms and Gomorrahs? ========================== CASE Christ's Church, a perfect supernatural society, simply cannot fail to protect generations of Catholics who have lived and died being taught and believing the heliocentric model IF it were harmful to the faith. That is most strenuously an impious notion to place on the Church of Christ. ========================== SUNGENIS The only thing “impious” here is your stubborn refusal to accept the official edicts of three popes on the matter we are discussing, and instead you choose to declare their official teachings as errors. By what authority do you claim to do such? If you claim that authority rests in the Church of today, then show us where any pope or Council has made an official annulment of the edits of the three popes in question. If you don’t have such a statement, then you don’t have any authority. Somehow, the possibility that the three popes God put in place in the seventeenth and eighteenth century who were also trying to “protect generations of Catholics” simply doesn’t register with you. You would rather take the word of the atheistic and agnostic scientists as your authority of choice, since you certainly don’t have any authoritative statement from the Catholic Church dismissing Geocentrism and validating Heliocentrism. ========================== CASE Pope Leo XIII was on the very doorstep of the issue and did not mention a condemnation. ========================== SUNGENIS That supports my side, not yours. The fact that Leo XIII did not reverse or annul the decisions of Paul V, Urban VIII or Alexander VII shows that he wanted nothing to do with committing the Catholic Church to the speculations of science. ========================== CASE The Catholic Encyclopedia of 1913 in many places mentions heliocentrism as a commonplace belief. ========================== SUNGENIS If I had a dollar for every “commonplace belief” that man has held, but was eventually overturned by later “commonplace beliefs,” I’d be a rich man. What you need to support your contentions are not the CE reflections on what was common, but a reference in the CE which documents an official and specific reversal of the edicts of Paul V, Urban VIII and Alexander VII. ========================== CASE Pope Pius XII mentions the stand of Copernicus to the Pontifical Academy of Science in the 1950's as if it were a recognized historical breakthrough. Yet YOU are doing the job that centuries of popes failed in – to protect Catholics from this danger? The implications are blasphemous. Sins of omission are actual sins. People lose sight of this today. ========================== SUNGENIS If you are referring to the 1951 speech to the PAS, yes, and in the same piece he treats evolution as an historical and scientific fact. I have written a paper on this speech if you would like to see it. So if it treats evolution as a fact, and yet his 1950 encyclical says evolution is not a fact, and those who make it so are way out of bounds, what does that tell us. It tells us the same thing that happens today when the Pope gives a PAS address. Most of what the Pope says in the speech is written by the PAS. Be that as it may, since when do the private opinions of anyone in the hierarchy serve as dogmatic beliefs for the faithful to hold? Can you cite an official and binding teaching of Pius XII which tells us that Geocentrism is wrong and that Heliocentrism is correct. If not, then the only thing “blasphemous” here is that you would charge someone as being “blasphemous” who bases his beliefs on three prior official papal edicts that have never been officially annulled. What is “blasphemous” is that you put such trust in the religion of Scientism such that anyone who challenges your viewpoint is committing the sin of “omission.” It wouldn’t be so bad if you had done a half-way decent job in marshaling some credible scientific proofs for your claims, but not only have you not done so, your “scientific proofs” to our Geocentric challenge have been some of the most vacuous of all the submissions on our website. ==========================
  22. GEO Doesn’t really make a difference. He had to approve the Inquisition’s findings, whether he was present or not present at the actual deliberations. Besides, Paul V is not your major problem, Urban VIII is, for he was both at the Inquisition [then as a cardinal] and approved of their decision, and refused, during his subsequent pontificate, to remove the ban, even until Galileo’s death. ======================== What science is (& is not) about AC Explain your statement “If gravity took 8.5 minutes to travel the 93 million miles, well, we wouldn’t be here to talk about it.” I don’t understand why the time involved for gravity to move from the sun to the earth would make a difference. ======================== GEO It makes a difference because the instantaneous gravitational reciprocity between the sun and the earth has been proven, that is, the earth reacts immediately to the sun’s gravity. There is no appreciable time interval, whereas light has been shown to travel the same distance in 8.5 minutes. Therefore, there is a contradiction in your theory of Relativity, if, as is well known, you believe that the fastest anything in the universe can travel is the speed of light. ======================== AC You also have complained several times that Newtonian physics does not explain what gravity “is”. This shows that your misunderstanding of science is so great that you do not even know what science IS. Science is the study of nature. The purpose is only to observe and to make sense of the observations. Science does not seek a higher understanding of things. The question of what gravity “is” is a philosophical question, not a scientific question. Science just tries to observe what gravity does. Scientists are happy with the explanation that gravity is the attraction between objects, because that is what they observe. To them, gravity is as gravity does. ======================== GEO Perhaps the “science” you’ve adopted “does not seek a higher understanding” and reserves gravity to a “philosophical question” and merely “observes what gravity does,” but not the science with which I am familiar. I think you make such conclusions because you simply don’t have an answer for the nature of gravity, and to avoid the embarrassment you are seeking to change the definition of science. But even granting for the sake of argument that your definition is correct, why are scientists still trying to find a physical cause for gravity? They’ve been working on the Graviton theory for about 50 years or more in order to have a physical cause for gravity (but with no success). Moreover, contrary to what you claim, science CANNOT “make sense of the observations” unless science knows what is behind the observations. If we were to take your proposition to its logical conclusion, then I would win the heliocentric/geocentric debate hands down, since what we OBSERVE is the sun going around the earth. But I’m sure you would be the first one to tell me that you don’t go strictly by what you observe, because appearances may be deceiving. Isn’t that what this whole debate is about? ======================== AC Does the aether provide an absolute frame of reference? If Relativity is invalid, then only one reference frame is the correct one against which all positions and velocities can be measured. It appears as aether, as pre-Relativity physicists thought, should be this “absolute frame of reference”. However, this presents a problem for your theory. Since aether is the absolute reference frame, if the stars are stationary with respect to the aether, then they are absolutely stationary. Furthermore, an absolute frame of reference cannot be said to move with respect to itself, so a moving aether would be self-contradictory in this case. If the aether seems to be rotating around the earth, then the earth must be rotating by definition. ======================== GEO No, the earth is the absolute frame of reference. If, as I am claiming, the earth doesn’t move but the aether does, then obviously the aether is not absolute. It is only the medium which holds things in their places. According to Sagnac, Michelson-Gale, Miller and Ives, either the aether is moving around the earth or the earth is moving through the aether. They couldn’t tell us which one was correct, but they DID tell us that aether existed. Thus, we’re going to have to pick one as the absolute and one as moving. Unfortunately for science, they cannot tell us. That is where divine revelation comes in. It tells us that the earth is the absolute frame of reference, because it doesn’t move. ======================== AC But if the earth instead is the absolute frame of reference, then the stars and planets are moving, and stars infinitely far away are moving at infinite speeds. Our own sun is moving at 3.875 million miles per hour. This seems ridiculous, to my ears, and the aether, which permeates the entire universe, should obviously be the absolute reference frame. Of course, you can solve this problem with the second postulate of Special Relativity, which states that any frame of reference moving at a constant velocity can be a valid reference frame. Why not? You already use the Lense-Thirring theorem from General Relativity. ======================== GEO I only use Lense-Thirring to show you that your own Relativity theory disproves Relativity’s claim that the earth is moving against the fixed stars. As for your Special Relativity theory explaining the anomalies by claiming that any frame of reference can serve as a valid frame of reference, I’m not going to dispute that. Your theory must incorporate that definition. But whether it is right or not is another story altogether. As I said in some previous exchanges, an equation such as minus the square root of minus one equals one is the same as the square root of one equals one, but which of those two is correct? Only the second one is, because the first one is an imaginary number that does not exist. Analogously, Relativity may give you a mathematical model of what you observe, but you don’t know whether it is really representing reality or not. And if Relativity is wrong, then the earth is not moving, as Einstein admitted to himself several times. It’s as simple as that. Unfortunately for Einstein, he thought the Michelson-Morley experiment gave a null result, and thus he developed his whole theory of Relativity on a false premise (or maybe he did it intentionally, and thus it is not surprising to find no reference to either Sagnac or Michelson-Gale in Einstein’s papers, as well as a concerted effort by Einstein and Shankland to discredit Miller’s extensive work on aether). As for the stars moving at tremendous speeds, I’ve already explained this in previous exchanges. It is the aether which is moving. The stars rotate around us because the aether rotates around us. If the aether is in Planck dimensions, then the math shows that it would have to rotate at that speed to keep its consistency. In fact, the aether’s rotation explains one of the biggest anomalies in science today. I’m sure you are familiar with the conventional wisdom that in order for the universe to be oscillating there must be a certain amount of matter in the universe. But science realizes that there is less than 1% of the needed matter to support the Big-Bang theory. So, they’ve invented the matter. They claim that the universe is filled with 99% of matter we can’t see, and they’ve assigned a name to it called Dark Matter. Unless they have this matter, then the universe should not be expanding but should have collapsed in on itself many years ago. Dark Matter sounds impressive, but anyone with a decent education knows this is just an escape from reality. When science is reduced to explaining anomalies by waving a magic wand to produce the needed material, then we know that something is seriously wrong with its whole concept of the universe. In the aether universe, the material is there already. It doesn’t collapse in on itself because the centrifugal force of a rotating universe counteracts the inward gravitational force. ======================== AC Why does the force of the aether depend on the masses of objects? The displacement of the aether, responsible for the force, would be equal to the VOLUMES of the objects, not the masses. How does the aether know how dense an object is? ======================== GEO Because aether permeates the object itself as well as the perimeter of the object. The difference between the two causes the specific tension that the object makes in the aether. Laws of physics & geocentricity AC This argument against geocentrism is a bit long, but please suffer through it just once. In order for the earth to be the center of the cosmos, you must first explain away Kepler’s laws of planetary motion. Since those laws are wholly dictated by gravity, gravity’s effects must also be discounted. The real challenge here will not be discounting any specific theory of gravity (Kepler himself died before Newton was born) but the EFFECTS of gravity. ======================== GEO The Geocentric system discounts neither Kepler’s laws nor the law’s of gravity. ======================== AC Johannes Kepler discovered 3 laws which, together, explained the motion, relative distances, and predictive positions of the planets. For his theory to be sound, Newton’s later calculation of the gravitational force had to agree with his observations. It turns out that Kepler’s laws describe the motion of the planets beautifully (even predicting Neptune’s existence before it was discovered). Here’s the problem then… In a geocentric universe, Kepler’s Laws do not work, cannot work, but in our present solar system – THEY DO WORK. ======================== GEO No, all of Kepler’s laws and all of Newton's laws work in the Geocentric system. If they did not work, there would not be a Geocentric system. ======================== AC Why can’t these laws work in a geocentric universe? Because they rely on the gravitational force, and that force wreaks havoc with any geocentric model of the cosmos. (Again, Kepler had no name for this force, but recognized its existence and dealt instead with its effects on the planets.) ======================== GEO I don’t know where you’re getting the idea that the Geocentric system does not believe in gravity. There has never been anyone who has ever entertained that idea. ======================== AC Newton never discovered a physical law explaining gravity. He simply found a way to describe its effects without ever understanding what it was. Those effects are very real though, and cannot be discounted as subjective. Objects fall (on earth) at roughly 32 feet per second squared. When Einstein and others finally dethroned Newtonian physics they at last supplied an explanation for gravity, but whatever explanation you may accept, objects still fall here at 32 ft. per second per second. ======================== GEO They do the same in a Geocentric system. ======================== AC Unless you are prepared to disprove gravity, please address the following issue: The force we call gravity, regardless of our explanation or label for it, is driven by an object’s mass. ======================== GEO You don’t know that. If, as you admitted above, neither you nor Newton have an explanation for the nature of gravity, only its “effects,” then how can you say that gravity is caused by an “object’s mass”? Obviously, that is a contradiction. You have no idea what causes gravity. All you know is that objects attract one another. You don’t know if the cause is in the object itself or in the environment the object is placed, or whether God just sprinkled pixie dust on them. Thus, if this issue about gravity is the basis of your argument, you’ve already defeated yourself. ======================== AC The sun’s mass, being many times greater than the combined masses of all nine planets, their moons, the asteroids and surrounding dust, has a gravitational field proportionally larger than the rest of our solar system. Taken by itself, gravity demands that the sun be at the center of our solar system. Start off with the sun orbiting the earth, and gravity quickly alters the system as the planets radically change their orbits (due to the sun’s enormous pull on them). Any system without the sun as the focus would be inherently unstable. The sun would soon take center stage. ======================== GEO No, because you’ve totally discounted the stars as the third and necessary component in this picture. The gravity caused by the billions of stars counterbalance the gravity of the sun, and therefore, the sun, even as large as it might be, does not need to be the foci of the ellipse. You should read the other exchanges to find out more about this. Your contentions variations of what has already been put forth by others and explained. ======================== AC If you discount this as a false problem, claiming that the orbits of the sun and the planets are fixed so that gravity does not affect them (as in the classic view where sun, planets, and stars are held in place by crystal spheres) then you would completely destroy my argument. . . except for three tiny things that you MUST explain away. . . ======================== GEO No, but the Geocentric theory adopts the aether as the medium in which the stars, the sun and the earth are placed. There is no need for crystal spheres. The tension caused in the aether by the bodies mass creates the gravitational effects we observe, since the aether is always going to try to reach equilibrium. And according to Ernst Mach and Lense-Thirring, supported by Einstein himself, the gravitational effects of a rotating star system around a stationary earth are exactly the same as the gravitational effects of a rotating earth in a stationary star system. ======================== AC …Kepler’s laws of planetary motion. Yes, you are always free to say “Laws, what laws? I do not recognize Kepler’s laws.” But that would be a great disservice to reason, because Kepler’s laws WORK. They DO predict planetary motion, and at the same time make a geocentric cosmos impossible. Here’s why: 1) Kepler’s 1st Law of Planetary Motion: A planet moves in an ellipse (with the Sun at one of the two foci). This law deals with a problem Kepler had when trying to describe the motions of the planets. Only an elliptical orbit can correctly describe these observed motions, and then only if the SUN is given a very special position – resting at one of the two foci (that is, imagine a circle with a dot at its center, now stretch the circle into an ellipse so that the dot stretches into a line. The two foci of an ellipse are the two end points of that imaginary line). If you try placing the earth at either focus, you will find predicting planetary motion (using Kepler’s 1st law) an utterly hopeless task. This law is inconsistent with a geocentric view. ======================== GEO Already explained above. ======================== AC Kepler’s 2nd Law of Planetary Motion: A Planet sweeps out equal areas in equal times. This law, restated, tells us that a planet in orbit changes speed in a very predictable way. Kepler derived this law through planetary observation. As a planet’s orbit around the sun brings it closer to the sun, as MUST happen with an elliptical orbit, gravity pulls harder on the planet – making it speed up. Later, as the planet moves away from the sun’s gravitational field, the sun’s gravity tugs from behind and slows the planet down. All this happens at a very specific, and measurable, rate. This law DOES describe the speed changes of the planets, in perfect harmony with what we now know of gravity. Why would planets in fixed orbits around a geocentric earth ever change speed? And even if they did, why would their rates of change be in exact accordance with the effects of a gravitational force that emanates from the sun (and only IF that sun is located at a focal point of an elliptical orbit)? ======================== GEO Again, the effects you have in the heliocentric system are the same in the Geocentric system. The only difference is that in our system the stars are rotating around a stationary earth while in your system the earth rotates against a stationary star system. ======================== AC Kepler’s 3rd Law of Planetary Motion: It states that the squares of the periods of the planets (the time for them to complete one orbit) are proportional to the cubes of their average distances from the sun. Again, restated, this law states that the farther a planet is from the sun (or whatever it’s orbiting) the more slowly it moves. But it says something else too – that if you know the distance between any planet and the object it orbits, you can easily calculate how many days are in that planet’s year. The earth takes one earth year (365 days) to orbit the sun. In a geocentric model, the sun takes that same 365 days to orbit the earth. Fine. ======================== GEO No, it’s a little more complicated than that. The sun revolves around the earth once per day. The sun also moves around the earth in a precession once per year, as well as a change in plane once per year, in order to create the seasons. ======================== AC But then, in a geocentric cosmos wouldn’t all the planet’s have years based on their distance from the EARTH, not the sun? They would have to, wouldn’t they, if Kepler’s laws were valid? ======================== GEO No, the planets revolve around the sun in the Geocentric system just as they do in the Heliocentric system. It takes the same time for both. The only difference is that in the Geocentric system, the sun and its planets revolve around the earth once per day. ======================== AC Problem is. . . the math doesn’t work out that way. It’s easy to tell how long a year is for any planet. Just wait for it to get back to the same place again, or count the seasonal changes of the Martian polar caps as they shrink and re-grow each Martian year. There are many ways to tell. All the planets in the solar system agree with Kepler’s 3rd law. They all take a specific number of days to make one orbit, in exact accordance with the 3rd law, just as they should if they are orbiting the sun (elliptical orbits mind you, with the sun at one focus). These calculations are based on the planets’ distances from the sun, and the math all works out. ==================== GEO As I said above, the math is the same for both systems. The Martian year is the same in both systems. That’s why this is a controversy, because both systems work. ==================== AC None of the planets would have years lasting the lengths they do if our universe was geocentric. The earth is just about 93 million miles from the sun. That’s not much compared to the infinite beauty of the night sky, but it’s plenty to prove that Kepler’s 3rd law wouldn’t work if the planets orbited the earth. The numbers just don’t add up (unless of course, you turn back to the sun-centric view and Kepler’s three elegant laws). In the end, you will find it impossible to explain the length of the year for all nine planets without dismissing Kepler’s laws, and the gravitational foundation they were built on. And even if you do dismiss gravity (despite its constant presence in your own life) you would then have to explain to me how Kepler’s laws always work out perfectly in every case (since those laws depend on gravity’s validity in the first place, including its demand for a sun-centered solar system). Put another way, Kepler’s laws of planetary motion MUST be invalid in a geocentric cosmos. They depend completely on the sun’s gravitational pull from a sun-centric position. And yet ALL observations of the planets (since Kepler’s discovery about 400 years ago) support Kepler, including the length of every planet’s year. ======================== GEO I think you’ll have to go back to the drawing board. You’ve made a lot of assumptions. Those who believe there is empirical proof that earth rotates &-or orbits are asked to give it: an open discussion 168 Model commonly accepted model for geocentricity AC I offer the following brief RealVideo clip: Kepler Finds Earth’s Orbit Around the Sun. It is a clip from a program titled The Mechanical Universe – Kepler’s Three Laws, and is copyright 1985 by the California Institute of Technology. Direct proof: direct – yes, the results of the computations apparently lead directly to the conclusion that the Earth must orbit the sun, and not the other way around. Observable – yes, Kepler reached his conclusion by observation of the orbit of Mars, and the relative positions of the Earth and the Sun. Physical – yes, the conclusion was reached by direct observation of the physical orbit of Mars. Natural – yes, the conclusion was reached by direct observation of the natural physical orbit of Mars. Repeatable – yes, the experiment would be repeatable today following the same procedure. Unambiguous – yes, I believe the video shows the results in an unambiguous way that simply cannot be explained in a way compatible with geocentrism. Comprehensive – well you got me there, if you want an out, as I am not actually providing any proof to you, but rather a graphic demonstration that is the apparent result of the proof. To qualify for comprehensive proof, I would have to provide you with the actual computations and logic that Kepler used, in a way you could understand and possibly refute, and I am admittedly not doing that. It is quite likely that neither you nor I could fathom the computations or complex geometry in any case. I feel that this clip provides very strong evidence that the proof you require does exist, and perhaps someone who is an astrophysicist will be able to simplify the explanation enough to confirm the proof to you, though you may still be able to come up with some fantastic scenario that you feel is able to make it conform to geocentrism. ======================== GEO Your diagrams don’t at all prove your contention. First, you might want to read up on the relationship between Kepler and Tycho Brahe. You will find that it was Brahe who made all the meticulous chartings of the planets, not Kepler. Kepler actually confiscated Brahe’s work after Brahe died. While Brahe was alive, he was using those very calculations to show that Geocentrism was the correct model. Before he died, Brahe insisted that Kepler use the calculations to continue the Geocentric position. Kepler, however, was influenced by other things. Since he was heavily into the occult, as was his mother, he was searching for “musical harmony” among the heavenly bodies. It was his opinion that a solar centered system coincided with that “musical harmony” as opposed to a geo-centered system. As such, Kepler’s choice had little to do with evidence disproving the Geocentrism. The fact still remains that both Heliocentrism and Geocentrism work, mathematically and physically. Kepler’s findings do nothing to disprove that fact. I have already dealt with Kepler’s claims in previous exchanges. I suggest that you consult them. The first is at www.catholicintl.com/epologetics/geo25.html . You wrote: Now the following can be seen above: Observations are made on the day that Mars returns to the exact same point in its orbit (here when it crosses the yellow line at the right. On those dates both the Sun and Mars are at the same fixed relative points, with the same distance between them. The Earth’s position changes relative to both the Sun and Mars, tracing out an orbit around the Sun (proving heliocentrism). This is nothing more than the retrograde motion of Mars as seen from earth. If you go to our posting on this subject you will see how retrograde motion is explained in the Geocentric system, which is at www.catholicintl.com/epologetics/geo18.html. You then write: Other points worth noting. Venus and Mercury are never observed very far from the Sun. Venus is always observed within 46. and Mercury within 28., so they are only observed in the early morning or early evening (they basically rise and set with the sun). They are never seen on the opposite side of the earth from the sun. Venus and Mercury exhibit the full range of phases, like the moon, but the other planets do not. The reason for this is that Venus and Mercury have orbits closer to the Sun than the Earth (inferior orbits), the other planets have orbits farther away than the Earth (superior orbits). Venus and Mercury both exhibit a full / gibbous phase when observed near the Sun, but farthest from the Earth. If they orbited the Earth (geocentric), we would never see a full phase when they are near the Sun. This proves that Venus and Mercury orbit the Sun, and not the Earth. The problem with this is that you are working with an incorrect model of the Geocentric system. In the Tychonic system, which we are using, all the planets orbit the sun, not the earth. It is the sun that orbits the earth, and carries the planets along with it. Thus, the diagram you showed which has Venus orbiting the earth is wrong, and that is why the model doesn’t work for you. As for the Geocentric explanation of the phases of Venus, it can be explained both from a Ptolemaic system and a Tychonic system. First the Ptolemaic system. Galileo had claimed that the Ptolemaic system could not account for the phases of Venus or Mercury. Actually, Galileo was partially correct. If one uses circular orbits for Venus and Mercury, the phases of the two planets do not appear. But if one uses elliptical orbits, as even Kepler did for the heliocentric system, then the phases of Venus and Mercury can be easily accounted for in the Ptolemaic system. Even the Jesuit astronomers who were challenging Galileo’s findings knew this to be the case, and thus they submitted their evidence to the Inquisition, which also concluded that Galileo had no proof that only the heliocentric system worked. In fact, the only thing that Galileo’s findings showed was that the epicycles in the Ptolemaic system were much larger than had previously been suspected. As for the Tychonic model of Geocentrism, if one uses the same elliptical orbits of Kepler, the result is that two epicycles in the Ptolemaic system will translate into one ellipse, per planet, in the Tychonic system. Thus, around the sun, Mercury and Venus would each have a perigee and an apogee, and each locus of points along that polarity would show the respective phases of Mercury and Venus, as viewed from earth. ======================== AC I presumed that you were defending a pure Geocentric system, in which all planets orbited a stationary Earth as proposed by Aristotle and Ptolemy. However my presumption was incorrect. As he states above, he is defending a system in which all planets but the Earth do orbit the Sun. Tycho Brahe proposed such a system. Here are links to pages that demonstrate the various systems. The Universe of Aristotle and Ptolemy. An impressive online animated simulation of three systems: Ptolemy (Geocentric), Brahe (Geocentric), and Copernicus (Heliocentric). Go to www.jove.geol.niu.edu/faculty/stoddard/JAVA/ptolemy.html and then “Conduct another experiment” Again, at the following web site you can, run a simulation: http://www.astroarts.com/simulation/cometary-orbit.php 1. Enter 55P in the “Object Name” box. 2. Press “Search” button. 3. Press “Show Orbit” button. 4. Run the simulation as before, but in the “Center” drop down menu, select “Earth” rather than “Sun”. Note that the Earth appears to remain stationary, and the Sun and all the other planets appear to revolve around the Earth. Note you can toggle the “Center” menu to any planet, and you do not change any of the mechanics of the simulation, you only change the presentation of the graphic, and artificially center everything on the selected planet. This is basically a matter of perception. You can center everything on the Earth, or Mars, or Jupiter for that matter, and the whole universe is presented as revolving around the selected center, but nothing else changes (in particular, no epicycles are needed to introduce retrograde motion). So I think this simulation shows rather effectively, whether the Earth is in the center, or the Sun is at the center, can be purely a matter of perception, and not physics. So it is quite possible for one person to perceive the Earth as orbiting the Sun, and someone else to perceive the whole universe as orbiting the Earth, and yet, hard as it may be to believe, both can be quite “right” at the same time, and neither can “prove” the other wrong, because everything else is absolutely identical for both of them. R Sungenis vs J L Case CASE The strength of our senses & logic (logic being the strongest of the two, for sense data is useless without our logic) in witnessing a demonstration are mentioned by St. Robert Bellarmine: “we would rather have to say that we did not understand [passages of Scripture] than to say that something was false which has been demonstrated”. Demonstration gives us certain knowledge. We KNOW what we sense, and our logic forms the conclusion that we KNOW things are true because of the facts. Such is so strong that we must give precedence to it when even the literal comprehension of scripture is contrary. This doesn’t put science above faith. Faith supercedes science. Faith is not contrary to reason but above it. St. Robert is talking about when the demonstration is actual CONTRARY to the literal comprehension of a passage of Scripture. ======================== SUNGENIS All granted, but St. Robert was not saying this as if science had demonstrated heliocentrism, but that it had NOT demonstrated it. The context of the paragraph (which you did not cite) says that very thing. ======================== CASE I did not cite it because it was not to my point. The point which you conceded by your saying, “all granted”, is that Catholicism allows for the possibility that demonstration CAN prove that what is literal in Scripture can be instead taken non-literally merely by knowledge of natural laws of science. But it can only do so where that literal interpretation is not intrinsically connected to faith. Whether the sun, or the earth, moves is not intrinsically connected to faith. If it WERE intrinsically connected with faith, St. Robert would have sinned against faith by admitting the POSSIBILITY of demonstration. Those people who mistakenly handle this subject as though it were “of faith”, will naturally attack “demonstration” in order to put literal interpretation of Scripture above demonstration. But as we see from St. Robert, natural demonstration is indeed ABOVE the literal interpretation of Scripture when the point is not “of faith”. And the Church has long said that this subject is not intrinsically connected to faith. ======================== SUNGENIS I don’t know anywhere the Catholic Church has stated that this subject is not connected to faith. If you have such an official statement from the Church, then you ought to produce it. One of the last official decrees from the Church we have is the decree of Alexander VII, which he wrote in a papal bull (Speculatores domus Israel), which was attached to the Index of Forbidden Books, stating that he condemned “all books which affirm the motion of the earth,” binding the consciences of the faithful. In fact, one Catholic author, William W. Roberts, in his tome The Pontifical Decrees against the Doctrine of the Earth’s Movement (1885), which was backed by many prelates of his day, argues that the Church committed its infallibility to those papal condemnations. There has been no pope or council since who has stated that Alexander VII was wrong, or has rescinded his judgment against the motion of the earth, or stated that Copernicanism is a scientific fact. Unless there is one, then the decree stands as is, regardless whether most Catholics recognize it today or not. As for Bellarmine, he is very relevant to this issue, since as head of the Sacred Congregation, he wrote in his letter to Galileo: “...speaking hypothetically, and not absolutely, as I have always believed Copernicus spoke. for to say that, assuming the earth moves and the sun stands still, all the appearances are saved better than with eccentrics and epicycles, is to speak well; there is no danger in this, and it is sufficient for mathematicians. But to want to affirm that the sun really is fixed in the center of the heavens and only revolves around itself without traveling from east to west, and that the earth is situated in the third sphere and revolves with great speed around the sun, IS A VERY DANGEROUS THING, not only by irritating all the philosophers and scholastic theologians, but also by injuring our holy faith and rendering the Holy Scriptures false.” So you see, that Bellarmine allowed one to say that Copernicanism “appeared” to work, but he did not allow one to say that Copernicanism went beyond appearance into FACT. As for the “demonstration,” Bellarmine further stated to Galileo: “I say that if there were a true demonstration that the sun was in the center of the universe and the earth in the third sphere, and that the sun did not travel around the earth but the earth circled the sun, then it would be necessary to proceed with great caution in explaining the passages of Scripture which seemed contrary...BUT I DO NOT BELIEVE THAT THERE IS ANY SUCH DEMONSTRATION; NONE HAS BEEN SHOWN TO ME. It is not the same thing to show that the APPEARANCES are saved by assuming that the sun is at the center and the earth is in the heavens, as it is to demonstrate that the sun REALLY is in the center and the earth is in the heavens.” This is the same argument I am giving you. If you can demonstrate that heliocentrism is the ACTUAL model of our solar system and not merely something that saves the APPEARANCES of what we see in the sky, then you have won this argument, and I will retract everything I’ve ever said about this subject. But you haven’t done so. ======================== CASE I have researched and found that visual “positioning” models have been made for BOTH the heliocentric model AND the geocentric model. Both models appear to show the same relative positioning depending upon point of view. Both therefore can predict the positioning of the planets in relation to each other for a future date. Of course, all this does so far is say that we have no weight to believe in one model over the other, EXCEPT for the fact that when we bring Scripture into the picture, we must (aside from actual Church allowance to the contrary) believe that the earth is motionless: both “positioning” models on a scale would be equal, but when the Scripture is added to the geocentric scale, the weight of that position is an unmistakable (again, when Church permission is excluded from consideration here). However, a visual “positioning” model is not a true model. It takes no account of the physical laws of God’s created nature which is so orderly and predictable that we can study it and develop formulae of ratios and proportions such as to predict how things will react according to those laws of nature. This is science – studying the order of God’s creation. The “positioning” model does not take into consideration proportions of mass, velocities, momentum, gravitational attraction, magnetics or friction. Mere positioning is not a true demonstration, but it is a start. It is a demonstrable fact of God’s creation that objects will continue at a constant rate of velocity in a straight line unless acted upon by another force. The object’s momentum is a product of its velocity and mass. The greater either one of those two, the greater the momentum. Depending upon the angle of an outside force and the strength of that outside force will depend upon acceleration/deceleration, the angle it makes to diverge from its straight path. This is so orderly that upon study man has devised formulas that work to predict the reactions and motions by applying the mathematical quantities. There is such a relationship that the Force'Mass x Velocity. Knowing any two of them can automatically give us certainty of the third value. (Velocity'Distance x Time) and (Mass'Density x Volume). ======================== SUNGENIS Granted, but the mathematics is only a numerical representation of what is supposed to be transpiring, but it is not a PHYSICAL description of what is transpiring. Equations do not necessarily represent reality. Equations only put in proportion one set of values in relation to another. Here’s a crude analogy. If, for example, I were in a sealed room, and you were outside the room, and I made a banging noise in the room every five seconds that you could hear very faintly but could not see, then mathematically you could say that I made 12 noises per minute, which would be accurate. But the math doesn’t tell you what is making the noise. The bang I am making could be anything from hammering a piece of metal to playing a base drum to setting off a small firecracker. This analogy is important since, for all Newton’s formulas, he couldn’t tell us WHY gravity existed, or what made it act the way it did. All he did was tell us, at least with the measurements available then, how gravity affected different objects in mathematical proportions. ======================== CASE You are denying the very essence of “demonstration”. Demonstration PREDICTS the future through formula. If a formula does not represent reality, then it won’t PREDICT and therefore is not a valid formula. By I am not talking about possibilities of invalid formulae and I don’t know why you are bringing it up here. I am talking of VALID formulae. The formulae which I cited are valid because they can PREDICT the future of movement and directions. This is definitively “demonstration”. And the predictions are true and can be repeated. Otherwise you would have to say it involved a divine prophecy or working of the Devil, on-tap. But it is based on God’s laws of nature and man has engineered intricate and immense things with these, and they come out as planned through the formulae. As long as the EFFECTS of gravity on velocity and direction can be PREDICTED at will, man does not need to know why it exists. You are starting off with faulty premises from the start. ======================== SUNGENIS I’m sorry to have to disappoint you, but your reasoning simply does not answer the question at issue. The question at issue is not which model’s math works (heliocentrism or geocentrism), since it is a fact that they BOTH work. I wouldn’t be here arguing the issue if the geocentric math didn’t work. If the math of each was different, then you would have a valid point. Both must have the precise mathematical result to explain what they see in the sky. Let me give you another illustration to prove the point. The Newtonian system of gravity uses the inverse square law to calculate the effect of gravity between two objects. The Newtonian system, for lack of a better explanation, assumes that the force of gravity emanates from the objects themselves. Conversely, the Le Sagean system uses the same inverse square law when it calculates the force between the objects, but it maintains that gravity is not inherent in the object itself, but is caused when the objects are placed in the sea of ultramundane corpuscles that pervade the universe. When objects are placed near each other, there is an imbalance of corpuscles between them. This causes the corpuscles on the far side of each object to push against the object, and thus move them closer together. If you are not familiar with this model, I suggest you pick up a copy of the book Pushing Gravity, which just came out. The point of this illustration is to show you that there are two entirely different models to explain the physical nature of gravity, and both use the same mathematical formula (the inverse square law) and come up with the same mathematical results and predictions. But in the case you and I are working on, we aren’t as interested in the math, since we know both our systems must have the same mathematical results. Rather, we are more interested in the MODEL behind the math. But as you can see, the math does not prove the model. The math only gives us a proportional relation of how one object is being effected by another. When you add to this Einstein’s theory of gravity, you have three working models from which to choose. ======================== CASE Now we know the positioning of the planets in relation to themselves and to the sun. That is why we come up with two positioning models. Actually, a separate relative positioning model can be made for EACH orbiting object in the solar system. 1. Sun center – all planets revolve around it.2. Mercury center – sun revolves around it while planets revolve around the sun. 3. Venus center – sun revolves around it while planets revolve around the sun. 4. Earth center – sun revolves around it while planets revolve around the sun. 5. Mars center – sun revolves around it while planets revolve around the sun. Etc. There is one created universe with the same physical laws of nature, and they must be consistent within creation. A theory which contradicts a physical law is demonstrated to be a false theory, and thus not a theory anymore, but a falsity. Now, all matter has a gravitational pull on other matter. The greater the mass, the greater the gravitational pull. Gravitational pull is undiscernible when the mass is very small. The only time we as humans experience it noticeably is because we are on a planet that is so massive that it attracts all objects. ======================== SUNGENIS Not necessarily so. This is precisely why I gave the above analogy. If neither Newton nor anyone else has been able to tell us what gravity IS, how do you know it is a “pull”? All you know is that objects move in the direction of the center of the earth, but you don’t know if something is pulling the object there or pushing the object there. ======================== CASE I already showed your last premise is an error. The analogy is useless here. We don’t need to know what gravity IS. We know all matter has gravity in increased proportion to increasing mass. We only need to be able to PREDICT, and when it comes to be repeatedly, the formula is correct. Experiments on earth can predict the effects of gravity, that is how we know. This is demonstration. ======================== SUNGENIS It is a demonstration only of the EFFECTS of gravity, but it is not a demonstration of the physical model of gravity. And since, as I stated above, you and I are only interested in which physical model is correct (heliocentrism or geocentrism), then the math is moot at this point. In fact, even if we were to use math in some indirect way to help prove our case, I have more on my side than you do on yours. For example, NASA sends all its rockets and satellites up using a fixed–earth model of the solar system. All their math is based on a fixedearth model. Granted, they can convert the math into a heliocentric model, but it is quite cumbersome, and thus they don’t make a practice of it. As it stands, of the two, the geocentric is the preferred. ======================== CASE The moon is smaller than the earth and the earth has more mass. This is why the moon, being smaller, revolves around the earth and not vice versa. ======================== SUNGENIS What “law” of physics states that a smaller object must rotate around a larger object? Read below. ======================== CASE The law of gravity and the interaction of two masses for orbits. I will concede it is “possible” because I cannot say I have studied all there is, but the probability is so very low because it would require such ideal circumstances that nature does not provide. The common occurrence is small around large because the probability is very high for a temporary orbit to be formed. (We cannot say permanent orbit.) My main point really is that large does not revolve around the smaller. ======================== SUNGENIS As long as you admit that it is possible, regardless of the fact that you assume the obverse to be the norm, then you really have no way of using your premise to deny geocentrism. In fact, my whole contention is that the earth is unique among all the heavenly bodies, for it is the place where man was given dominion and the place where the Incarnation would take place. If the earth is indeed the center of the universe, then obviously, any heavenly body outside of the earth must be moving against a central, and thus, stationary earth. I wouldn’t expect this for any other heavenly object, since none of them can claim to be the center of the universe. (Obviously, I am rejecting the modern idea that there is no center to the universe). But to give a physical explanation (since we are focusing on physical models in this discussion) of the earth–stationary/sun–revolving system, I pointed out in my last post that the star system which encircles the earth creates its own force. This force acts upon the sun in such a way that causes the sun to remain in its orbit around the earth. Since the combined force of the stars is going to be greater than the sun, then naturally, the sun is the inferior object in this relationship. The sun is going to do what it is forced to do by the stars. As it stands, the sun will be forced to remain embedded at a distance of 93 million miles from earth, and rotate around the earth each day, along with the rest of the stars. In this scenario, even your insistence of the “larger body controlling the smaller body” is satisfied, since the stars are controlling the movement of the sun, and the earth is merely the pivot point for this relationship. ======================== CASE The moon has a velocity parallel (tangential) to the earth, but because of the earth’s gravitational pull it takes it off of its straight path making it fall at an angle. While it decelerates its parallel speed it correspondingly accelerates towards the earth. There is such a perfect equilibrium that it falls always missing and going around the earth. This is an orbit. Not so easy naturally to attain such a equilibrium but in God’s designs it was founded. There is nothing miraculous about it. ======================== SUNGENIS Yes, I agree God designed it to orbit, but you haven’t proven that the orbit is caused, as you say, by the “earth’s gravitational pull.” You haven’t even explained what gravity is in order to make such a proposition. All you know is that the moon goes around the earth. ======================== CASE Orbits have been proven to be established by the gravitational attraction of all that has mass with their respective velocities and directions. Again, you don’t need to know what it is, only what it does and whether you can PREDICT by formulas. I get the feeling that you think orbits are some miracle or mystical phenomena that could never follow any formula except the direct intervention and sustainment of God. But the physical laws we study on earth are the physical laws of all creation. You seem to be implying that there are entirely other laws out there. St. Robert Bellarmine allowed for earthly demonstration to prove otherwise; you seem to create imaginary things you say are hypothetically and gratuitously “possible” without any basis other than to try to keep Scripture literal. That is not the mind of the Church. Man has explored outer space and knows it is a virtual vacuum – empty space. You are trying to suggest the moon is placed in a fixed orbit by God and circles without any regard to the forces of gravity and inertia. That is contrary to reason because it is contrary to demonstration. ======================== SUNGENIS No, you are totally misunderstanding me, or perhaps trying to make it look to your audience that I believe in magic and superstition. Far be it from me to engage in such folly. As you noted in my previous paragraphs, all my explanations are based on science. The problem is that you keep assuming that YOUR science is the only valid science. If someone has an alternate theory of how things operate, scientifically, you have a hard time accepting such possibilities. But there is one thing I’ve learned about science, Mr. Case, and that is that science keeps overturning its own theories, year after year. The more they discover, the more they find out what they don’t know. You can speak about “gravitational attraction” as establishing orbits and I won’t argue with you. But unless you can tell me how your “gravitational attraction” discounts the “gravitation attraction” in a geocentric system, then you’re not going to get very far. Every force and movement you explain in a heliocentric system can be explained, mathematically and scientifically, in a geocentric system. ======================== CASE Scientists have seen observable and repeatable proportions to explain an orbit. Beyond the other characteristics of the aforementioned moving body, the factors of the mass of the earth (the amount of gravitational force depends on it), the distance of the object’s orbit away from the earth, and the speed are crucial to whether or not it stays in orbit, plunges to the earth, or flies out into out space once again in a straight path. If the moon’s mass changes, it would upset that equilibrium. If the mass of the moon substantially increased, it would have too much momentum at that height and speed for the earth to attract it enough.....and it would fly into outer space. If the moon’s mass were to substantially decrease at that speed and distance, the momentum for missing the earth would not be there sufficiently and it would spiral around the earth until it crashed into it. ======================== SUNGENIS Actually, the moon is continually moving away from the earth in its orbit, a few centimeters per year. In fact, if it has moved in the same proportions we have observed it moving away for all time, then this earth could not be more than a 100,000 thousand years old, for the moon would have been off into outer space. ======================== CASE That is why orbits aren’t permanent, as I said before. There are so many changing factors involved that slowly degrade the orbits. But the orbits themselves can be predicted by formulae nonetheless, and the mass, velocity, distance all fit. The degradation shows that God has set the moon within laws of His physical universe, and the degradation of orbit is due to sporadic slight forces as come from elsewhere in the relative vicinity of the universe. ======================== SUNGENIS No comment. ======================== CASE This goes the same with the other characteristics of the formula. If the moon were exactly the same in all respects but were to be moved farther away from the earth’s surface, the strength of the earth would be less at a greater distance and could not have a hold on the moon’s momentum and it would fly into outer space. There is such and interrelationship between the masses of the two bodies, the distance and speed, that any one can throw off the balance and there will be no possibility of an orbit. The formulas have been tested. Man puts satellites in orbit around the earth based upon this formula. It works, and the laws of nature are solid. ======================== SUNGENIS Even that has its problems. Have you ever heard of an orbit decaying? Why does it decay? Because there are net external forces which act upon the object. Hence, the same “laws” of force and inertia you claim keep an object orbiting also cause an object to fall from that orbit. What you then have to explain is how the moon, or even the planets around the sun, can keep their orbits being that they are constantly influenced by unpredictable forces. Those forces include anything from the intermittent gravitational effect of the planets upon one another (which will be different effects depending on the position of the planet) to the bombardment of cosmic particles. So with all these forces acting upon the moon, what keeps it in its pristine orbit around the earth? ======================== CASE As you even said before the moon’s orbit is SLOWLY decaying – so why say “pristine orbit”? No, it is temporary and sufficient. As I said, there are many changing factors. They are so minuscule compared to the mass of the bodies in question that they do not affect substantial orbit until it gets to a certain point of degradation. God may, in His designs, have certain comets periodically come nearby to help retain orbits for a longer time. The universe is perfect clockwork, but you cannot make something mystical out of it and flaunt demonstrable physical laws just because you want a literal interpretation of Scripture. It is not coincidence that orbital science works in its formula. The formula for orbits work, and you are unreasonably using the existence of extraneous minuscule forces to try to say the formulas don’t work. That is a fallacy. ======================== SUNGENIS As I have shown, I am not “making something mystical out of it and flaunting demonstrable physical laws just because I want a literal interpretation of Scripture.” I am using the same gravitational “laws” that you are using. The only thing different is where we place the bodies that are gravitationally interacting with each other. ======================== CASE This formula must apply equally to ALL bodies in our solar system. Mass, momentum, distance, direction, gravitational pull, etc, must all apply to every body at the same time. Both on earth and in outer space there are the same physical laws and principles of nature. The sun is 330,000 times as massive as the earth. It is utterly absurd to say that the sun could be in orbit around the earth. ======================== SUNGENIS First of all, you haven’t proved that a larger body cannot orbit a smaller body. You can say it’s “absurd” all you want, but that doesn’t prove anything. Second, how do you know the sun is 330,000 times the mass of the earth? You don’t. All you know is that, according to Newton’s laws (laws which you haven’t shown me necessarily represent reality), dictate to you that the sun must be that large. ======================== CASE It seems that you doubt almost all commonly accepted fundamentals of scientific laws, and that I would have to reinvent the wheel and re–prove to you each and every one before I use it as a premise. That itself is absurd. The mere PROVEN formulas of mass and vectors of momentum force, and the forces that are needed to change inertia are obvious. You don’t need to know the exact difference in size, an approximation still shows the obvious effects. Scientists are quite certain based on lots of different data how big the sun is and how massive. But you only need an approximation because its size is immense. The thought of a sun so massive, and SO quickly, circling the earth is absurdly contrary to ordinary experience and demonstration of inertia in creation. It would also necessarily mean the force is centered FROM the earth, but such an immense force as to do that would have necessarily pulled the moon to itself immediately. But it doesn’t. ======================== SUNGENIS No, as I stated above, the earth is only a pivot point. The stars, which are dispersed evenly over the entire universal sphere, are what control the balance of gravity, not the earth. As I said in my last post, your heliocentric theory already admits to the fact that the sun must move against these stars to the tune of 500,000 miles per hour. Thus you admit that the sun is the inferior object when compared to the combined force of the stars. I’m saying the same thing. The only difference is that I’m placing the sun in between the earth and the stars, and you are placing the earth in between the sun and the stars. Both our systems work out mathematically and scientifically, but they are, indeed, different models, and that is solely what this discussion is about. In fact, let me offer, on a scientific basis, how the geocentric model accounts for some of the anomalies you have in the heliocentric model. For example, it has been long known in the heliocentric model that there is simply not enough matter in the universe to account for what is theorized regarding the “expanding or contracting universe.” So what does your science do? Well, they invent a form of matter called “Dark Matter,” which they say composes 99% of the needed matter in the universe. Why call it “Dark Matter”? Simply because no one has detected its existence, yet the theory says it has to be there in order to support the expanding/contracting universe theory. Well, let’s just say I find that highly “unscientific,” Mr. Case. It is simply another case of science using a fudge factor to explain something they don’t know too much about. What does my science do? Well, we don’t have a problem with the universe collapsing in on itself, since a star system that rotates once per day at a specific radius from the earth will create the needed centrifugal force to keep the stars in their places, without having them fly off or collapse in on themselves. The math of this model has been worked out precisely. ======================== CASE That would be saying that the earth has such a force on the sun that it makes it go off its straight path towards it and that the sun keeps falling towards it always missing and staying in orbit. It is to say that the mass of the sun is held as if on a string by the earth and swung about itself. Considering the mass ALONE, at a smallest possible speed for the sun, the earth could not hold it in orbit, and when you add the distance and the great speed of the sun it is an utter absurdity. ======================== SUNGENIS Not necessarily. That is only how it would work in YOUR system of “pulls” instead of “pushes,” but you haven’t proved that gravity is a “pull.” Until you explain what gravity IS, you have no basis to make such claims. ======================== CASE It has been proven by demonstration. It is mutual attraction of matter. If the sun were so attracted to falling towards the earth, it would also tend to fall towards all the other planets....and it doesn’t. Because of mass, inertia and momentum, the smaller falls towards the larger. The earth’s moon does so, all the other planets do so towards the sun, and all the planets have smaller moons that fall towards their own planets. All the observable orbits fit this logical observation. ======================== SUNGENIS As I said above, not only have you not proven this to be the case, but you haven’t explained, from your own system, how the differing forces from the same planet upon the earth or the moon do not alter their orbits. In fact, your “laws” actually get in the way of having the same orbits year after year. You have to grapple with the fact that no matter what intermittent and unpredictable planetary forces act upon the moon and earth, they always maintain the same orbit. How is that possible in a system, such as yours, which depends only on the forces of gravity of the smaller object falling toward the larger object? As you can see, the problem with your system is that it is a theoretical model of one sun and one planet obeying Newton’s math of F=ma, but you forgot that you have eight other planets that also put forces upon the earth, as well as the other cosmic forces in space, and from the stars themselves. In short, Mr. Case, it is all very complicated, and no one is quite sure how all the pieces of the puzzle fit together. Scientists have tried to calculate how all these forces interact, but it is a virtual impossibility. Once they put in more than three bodies in motion, even the highest math can’t figure out how the bodies will interact with one another. And this is especially true since they don’t even know what gravity IS in order to explain how the forces are interacting with one another. ========================
  23. AC No, the reason these satellites move an equal distance above and below the equator is because they are orbiting the earth in nearly circular orbits. As you can see from the above “inclination” figure, a satellite in such an orbit will always move equal distances above and below the equator. It has nothing to do with gravitational forces from the stars. ======================== GEO Mere assertion is not going to prove anything. The question you posed in your last was how I could explain the up and down motion of Marisat in MY system. To deny the answer, you would have to show that my system doesn’t work, not that yours works. My claim from the start is that both models work. ======================== AC Well, your system doesn’t work. If gravity is a “pull,” then you can’t explain why the moon, which is hundreds of thousands of miles closer to the stars than the “equilibrium point” between the earth and the stars, doesn’t go flying off toward the stars. ======================== GEO Already explained that above. ======================== AC Nor can you explain how geosynchronous satellites like Hot Bird-2, which moves thousands of kilometers toward the earth each day, don’t just keep right on going. What turns them around? On the other hand, if gravity is a “push,” then you can’t explain why all orbiting objects aren’t pushed away from the earth until they get to the point where the push from the stars balances the push from the earth. ======================== GEO Gravity is an interaction of objects with the medium surrounding them. You are misunderstanding what “push” means. As for problems about satellites wanting to “keep on going,” you have more of a problem in your system than you claim I have in mine. Your system depends on the constant speed of a satellite, such as the moon, so that it doesn’t fall to the earth. Yet it is a fact that the moon is tugged in all directions by differing gravitational pulls by the other planets and the sun. So why doesn’t the moon slow down? And if it slows down because of these gravitational pulls from the planets, as it should, why doesn’t the moon fall to earth because of a decaying orbit? Your system really has no answer for this, but the Geocentric system does, since the moon stays in orbit by the opposing forces between the stars and the earth, which are perfectly balanced. The reason they are oscillating is because of the differential gravity between the stars and the earth. ======================== AC Even if that explanation made sense, which it doesn’t, it still doesn’t account for the zigzagging satellites. Why do they move in a slow and steady zigzag while the others don’t? ======================== GEO I already explained this. The circumference of the loops is too small to show up on the graph, so they result in zig-zags. ======================== AC I mean, what pulls them in a steady westerly direction? And why doesn’t that force pull other satellites to the west? ======================== GEO They are oscillating with respect to their inclination. And we would expect them to go toward one direction, since the rotation of stars around them is always the same. Now, you’ll notice in your list that the satellites have either a zig-zag or figure-eight pattern. Why are they different? Since the satellite is placed in an elliptical position, not a circular position, with respect to the earth, this means that the satellite cannot maintain its position unless it moves faster or slower against the inertial frame. ======================== AC Wait a sec, I thought these satellites weren’t orbiting. What’s this talk about “elliptical” and “circular”? These satellites are supposedly just levitating up there, remember? ======================== GEO I said they are in an elliptical position in respect to the inertial frame, which is the gravitational force between the stars and earth. ======================== AC Sorry, this sounds like gibberish to me. Could you put it in plain English? ======================== GEO Let me try again. They aren’t orbiting. They are hovering above the earth, and oscillating between the forces below and above the earth’s equator. The gravitational force causes what in your system appears as an elliptical orbit around the earth, but what in my system is an oscillation back and forth between the earth and the stars. ======================== AC How does it do that in your system? You can’t go to the Sharper Image and make the object suspended by that magnet toy move in a perfect figure-eight; nor can you make it zigzag. Therefore, your explanation fails. ======================== GEO You need to play with the toy. You will see that if you push it one way, the object will bounce back the other way, and it will keep on oscillating until the friction of the air causes it to slow down. Thus, when the satellite is closest to the earth (the perigee) the satellite would be faster with respect to the inertial field (and from Gary’s perspective it would rotate around the earth in less than 24 hours); and when the satellite is farthest away from the earth (the apogee) the satellite would be moving slower with respect to the inertial field (or from Gary’s perspective it would take more than 24 hours to rotate around the earth). ======================== AC Again, that’s only true if the satellite is in orbit. But you say it’s not orbiting, and therefore, you can’t appeal to the velocity differences at the perigee and apogee of an elliptical orbit. He needs to come up with an explanation that makes sense for an object that’s levitating, not orbiting. ======================== GEO It is not just levitating, it is oscillating across a levitation point. ======================== AC How does an object oscillate across a “levitation point” in a perfect figure-eight? ======================== GEO By increasing the horizontal component of the sine wave. You can prove this by using an oscilloscope. It will produce a figure-eight pattern. This movement is going to create various patterns. The closer the semi-major axis of the orbit lies along the apogee-perigee line, the wider the figure-eight pattern will appear, since it will be oscillating with greater force. ======================== AC What orbit? If these satellites are orbiting, then the earth is rotating. Are you ready to admit that they’re really orbiting, Bob? ======================== GEO No, I’m using “orbit” to accommodate your system, not agree with it. ======================== AC Then please explain how this works in your system. Don’t worry about accommodating mine. ======================== GEO Already explained it. The satellite is oscillating above and below the earth’s equator. In your system its “orbiting” in one plane and it appears to move because the earth rotates against that plane. Now, let me add one more dimension to this picture. Against what background is the figure-eight pattern or zig-zag pattern measured? In your universe it is measured against a rotating earth. In the Geocentric universe it is measured against a rotating star shell. ======================== AC No, they move in figure-eights and zigzags relative to the surface of the earth, whether the earth is rotating or not. It’s just that if the earth’s not rotating, there’s no way to account for that motion, as you have amply demonstrated. ======================== GEO No, I’ve accounted for it, but you don’t believe that I can explain how the GSS can hover above the earth. That’s the whole issue. An oscillating GSS against a stationary earth DOES explain the various patterns. That is a fact. You choose not to accept that model because you can’t accept that the forces of the stars and the earth can create a point at which the GSS can hover. But as you have shown over and over again in these dialogues, you don’t know the physics. You didn’t even know the Lense-Thirring and Machian effects until I brought them to your attention. All you know are Newtonian formulae for orbital mechanics. And since you’ve thrown out Relativity, then I’m at a loss how as to how you’re going to explain all the anomalies in micro and macro physics that Newtonian mechanics can’t begin to solve. I’ve amply demonstrated that the satellites can oscillate between the earth and stars, and you’ve shown no material evidence against that possibility. Now I have. How do you account for it? In fact, you have denied the physics that an object can remain at equilibrium between two magnetic forces, and you have denied that they can oscillate. I suggest you purchase the levitating globe and you will see that principle of physics readily at work. ======================== AC I suggest you do the same and try to get the levitating object to oscillate in a figure-eight and a zigzag. I submit that you can’t. ======================== GEO In fact, you can make an object oscillate in just about any figure you desire, depending on which direction you give the initial push and how strong the magnets are. Let me explain: If the earth is stationary, the stars are rotating around the earth every 24 hours. They will come back to precisely the same point each 24 hour period (allowing a little movement for precession, which I don’t want to get into right now). We can see the satellites move against the background of the stars. On the first night we will see Marisat in a certain position against a particular star. But 24 hours later, we will see that Marisat did a figure-eight against the background of the star. (In case you’re wondering, it makes no difference whether the star rotates around the earth, because every 24 hours the star will always come back to the same position in the sky). ======================== AC If we were at the equator, and if we were able to train our eyes on Marisat-3 for 24 hours, we’d see it move around our sky in a figure-eight pattern. That’s its motion relative to us. It has nothing to do with the stars. If we were somehow able to block our view of the stars and look only at Marisat-3, we’d still see it moving in a figure-eight pattern. ======================== GEO We’re not talking about looking at it from earth. We’re talking about looking at it from the photograph you presented with the stars in the background. You based the motion of the satellite on the stars, not the earth. It has become apparent to me that you either don’t understand or won’t accept that a force outside the earth can affect an object placed above the earth. This is because you fail to accept the Machian principle that there is no difference between a rotating earth and a rotating star system, relative to US. Because of this, I believe this series of dialogues cannot progress any further, as I have stated previously. ======================== AC I do accept that forces outside the earth can effect objects above the earth. I just don’t see how any known (or conjectured) force can make a satellite hover over the earth, while moving in a figure-eight pattern. Your attempts to explain this movement have so far depended entirely on theories you otherwise reject, and conjectures that raise more questions than they answer. ======================== GEO As I said above, Lense-Thirring works in any system. So does Mach. Is Marisat showing us real movement? Yes, it is real. Marisat is actually oscillating due to the fact that it is in a 10 degree inclination with respect to the earth’s equator. Being in that inclination, Marisat will experience opposing gravitational forces from the earth-star gravity field, and thus it must oscillate back and forth between the equilibrium point, which is at the equator. ======================== AC Why is the alleged equilibrium point at the equator? Please explain what leads you to this conclusion. ======================== GEO The Foucault pendulum. ======================== AC Let us say, for the sake of argument, that these “opposing gravitational forces” were able to account for the north-south oscillation of Marisat-3. What accounts for its daily east-west oscillation? There can be no imbalance of forces parallel to the direction of rotation of the stars, so what makes Marisat-3 oscillate east-and-west? And what makes Brasilsat-1 move in a steady westerly direction? ======================== GEO If it’s on an incline with respect to the equator, it will go east-west or northeastsouthwest, or any combination of two compass opposite compass points. ======================== AC I can easily show you that there’s no correlation between a satellite’s east-west oscillation and its inclination. Try again. ======================== GEO Just the evidence, please. But what does Marisat look like in the heliocentric model? Since for you the earth is rotating, then you cannot attribute the movement to an oscillation of the satellite itself, but to the rotation of the earth against the inclination of the satellite. That is why you say that the figure-eights are only there by “appearance.” ======================== AC Yes, that way I don’t have to appeal to imaginary forces that control the movement of artificial satellites, but don’t effect the moon or the sun or the planets, all of which are allegedly able to orbit the earth every day, despite being well beyond the point of equilibrium between the gravitational forces of the stars and the earth. If your explanation were correct, every object beyond the equilibrium point where the geosynchronous satellites orbit – excuse me, levitate – should go flying off toward the stronger gravitational pull from the rotating shell of stars. ======================== GEO Again, this just shows your inability to address the issue because you don’t know the physics. ======================== AC Then explain it to me. I have a degree in Engineering Science and Mechanics. I’m sure I’ll understand what you’re saying. But so far you’ve just asserted things. I’m sorry, but I’m not just going to take your word for it that things behave the way you say they do. And I don’t expect you to take my word for things, either. That’s why I’ve spent so much time creating elaborate figures to illustrate what I’m saying. ======================== GEO Your “elaborate figures” don’t prove anything for you. The apparent motion of an object can be explained in one of two ways: either the object is actually moving or the background behind or in front of it is moving. You can’t tell which is correct unless you have some way of verifying it. Your understanding about opposing gravitational and magnetic fields is wrong. ======================== AC Not if those forces are pulling, which they would be if they were analogous to gravitational attraction. If they’re pushing against each other then, yes, you could suspend something between them. Of course, then you’d no longer have a system that was analogous to the gravitational pull from the earth and the stars. ======================== GEO Pulling or pushing is not the issue. Equilibrium between the stars and the earth is the issue. Your system neglects the stars and attributes all centrifugal and Coriolis forces to the earth alone. Lense-Thirring says that is not our only option. If so, then you haven’t proven that the earth rotates and the stars are fixed, unless, of course, you want to show us where Lense-Thirring went off the track. Moreover, you do not try to answer my argument by showing that a gravitational field from the stars is impossible to happen, rather, you resort to calling them “imaginary forces.” But I invite you to read the Lense-Thirring paper, and you will see that, even from your own science, they are not imaginary forces. ======================== AC Fine, then they’re imaginary from your science, because you reject Relativity. So what forces really account for the motion of these satellites? ======================== GEO If you knew the physics, you wouldn’t dismiss Lense-Thirring based on Relativity. If you don’t accept it, then Relativity goes out the window and you are forced to explain everything from Newtonian mechanics, which can’t be done. ======================== AC I can explain the motion of geosynchronous satellites from Newtonian mechanics, and thereby prove that the earth rotates. And that’s all I’m trying to do. ======================== GEO Not really. All you can do with Newton is give a mathematical formula to the GSS movement, but Newton doesn’t explain why the GSS moves as it does. Neither you nor Newton have a physical explanation for movement. Math does not explain reality. It only puts opposing forces on either side of an equal sign. AC Have you considered the issue of how water drains from sink (or toilet bowl)? You may be aware that if you are located above the equator, water drains in a counter-clockwise flow, while if you are located below the equator (e.g., in Australia), the flow is clockwise. I may have reversed the flows because I am writing from memory, but the point is that the flows reverse as you cross the equator and this phenomenon is attributed to the earth’s rotation. In a static earth, what could be causing this phenomenon? ======================== GEO It is not true that drains go in one direction in the northern hemisphere and the opposite in the southern hemisphere. Even those who believe in a rotating earth say that the Coriolis effect with regard to drains is practically non-existent. In fact, I had a friend do a test for me in Australia last year. He said the water went down his drain in a counter-clockwise direction, the same as in my northern hemisphere. On another plane, the Geocentric system does account for the Coriolis effect, but it does so by basing that force on the differential gravity between the rotating star shell and the stationary earth. This is why the Foucault pendulum works in the Geocentric system the same as it does in the Heliocentric system. If you want to familiarize yourself with the Machian principle of the Coriolis effect, as well as the Lense-Thirring effect supported by Einstein himself in order to help understand why a rotating star system would produce the Coriolis effects, please consult my previous exchanges. Do a word search on Lense-Thirring and you will find it easy. Shuttlecraft AC According to NASA (http://neurolab.jsc.nasa.gov/inclin.htm) the Space Shuttle can be launched into orbit at various inclinations, depending on what a particular mission is intended to achieve. However, the Shuttle is always launched towards the east (see this image: http://neurolab.jsc.nasa.gov/pixs/inclin.jpg). This is done because the Earth (at the latitude of Cape Canaveral) rotates eastward at a speed of about 1500 kilometers per hour. By launching towards the east, the Shuttle adds the speed of the Earth’s rotation to that of its rockets, and thus less fuel is needed to be burned to get the Shuttle into orbit. This speed boost (and the presence of the ocean in the preferred direction of launch, in case of crashes) is why the shuttle is launched from Florida, which is near the equator and thus provides Shuttle launches with a large fraction of the possible benefit given to us for free by the Earth’s rotation. The location of the spaceport used by the European Space Agency (ESA) was chosen for the same reasons. The ESA does not launch it rockets from Europe, but rather from Kourou in French Guiana in order to benefit from the Earth’s rotation (and the same spaceport is also used by the private company Arianespace). From the ESA’s website (http://www.esa.int/export/esaLA/spaceport.html): “Kourou lies at latitude 5°3’, just over 500 km north of the equator. ... Launchers also profit from the ‘slingshot’ effect (the energy created by the speed of the Earth’s rotation around the axis of the Poles) as this increases the speed of the launcher by 460 m per second). Both these factors save fuel and money, and prolong the active life of satellites.” The following site discusses the limitations involved in launching rockets from Vandenberg Air Force Base in California in light of the fact that the Earth rotates to the east, while the ocean (again, in case of crashes) is to the south and west of Vandenberg: From http://www.schnapp.org/sky/sky_vandenberg.html: Vandenberg Air Force Base is located near Lompoc, California, on the coast. The coast of California conveniently juts out into the Pacific at this point. This affords the Air Force and commercial customers very convenient access to westerly and southerly launch trajectories. Launching a satellite to the west is not particularly desirable. Remember that the earth rotates to the east. At these latitudes, the earth’s surface is moving at a pretty good clip, close to 1,000 miles per hour. If you launch to the west, you spend a lot of fuel canceling out that initial velocity. No, for launches to a conventional orbit, Vandenberg is not terribly useful. Most low inclination (more or less equatorial) orbit customers launch from Cape Canaveral instead, where they have a clear path to the east. Instead, Vandenberg specializes in two kinds of launches: Polar orbits and missile tests. They are an ideal location for high inclination (more or less polar) orbits because of their clear path to the south. And the clear southwestern aspect allows them to launch Minuteman III and Peacekeeper missiles toward Kwajalein in the Marshall Islands. So, in short, that the Earth rotates is proven by the practical fact that rockets need less fuel to launch towards the east than to the west. If the Earth didn’t rotate, one could just as easily launch rockets into orbit in either direction. ======================== GEO The information you have given does not prove the earth is rotating. Here is the reason. We all agree that at least something is causing the day/night sequence on earth, and at least something must be causing the stars to appear to go around the earth each night. In the Heliocentric system, the day/night sequence and star movement is attributed to the earth’s rotation; in the Geo-centric system it is attributed to the stars’ rotation around the earth. Both systems will produce the same observable results. It is also true that both the Heliocentric and Geocentric system will produce the same forces on Space Shuttles and satellites. In the Geocentric system, the rotating stars exert a gravitational force upon the earth that, along with the earth’s contribution of gravity, results in the same centrifugal, Coriolis and Euler forces that Newtonian mechanics attributes solely to the earth. As I’ve stated in other exchanges, Mach’s principle allows me to make the equivalency. Here is what Mach himself said on the issue: ...all masses, all motion, indeed all forces are relative. There is no way to discern relative from absolute motion when we encounter them...Whenever modern writers infer an imaginary distinction between relative and absolute motion from a Newtonian framework, they do not stop to think that the Ptolemaic and Copernican are both equally true. (Die Mechanik in ihrer Entwicklung historisch-kritisch dargestellt, eighth ed, Leipzig, p. 222, 1921). Thus, all the forces, as the “slingshot effect” you attribute to a rotating earth can be equally attributed to a rotating star system around a stationary earth. There is no difference. Starlight, parallax & aberration GEO It is explained in one of two ways: (1) the same effect would be observed if the stars are centered around the sun and partake of the sun’s annual motion around the earth. (2) the sun has an aether field attached to it that sweeps past the earth with a period of one year. The sun’s aether would drag the starlight with it and an aberration would be observed. Science knows this as the Fresnel Drag, and it is readily observable. ==================== AC Neither of your explanations can be correct, because they both conflict with observations. (1) If a distant star partakes of the sun’s supposed annual motion around the earth then that star would be moving around in space in a circle 186 million miles in diameter (it would actually be an ellipse of the same size and shape as the earth’s orbit around the sun, or as you would say the sun’s orbit round the earth, but a circle is near enough). The star would appear to move around a small ellipse in the sky once a year, much as I described. This effect is observed; it is parallax. For Proxima Centauri, the major axis of the ellipse is about 1.6 seconds of arc. For other stars, the ellipse is smaller, and we deduce from this that Proxima Centauri is the closest star to the sun. The wobbles in more remote stars, although they are all 186 million miles across, appear smaller to us (in seconds of arc) because they are further away. But stars appear to wobble in another way as well. All the stars in the sky appear to move around small ellipses in the sky once a year, and the major axis of all the ellipses is about 40 seconds of arc. The size of these ellipses is the same for all stars in the sky, it does not depend on how far away the star is. The motion of a particular star in the sky is actually observed to be a combination of these two wobbles. There is one wobble whose size varies from star to star; this is parallax. There is another wobble whose size does not vary from star to star; this is the aberration of light. The motion of the star through space around a circle 186 million miles in diameter explains parallax, it does not explain the aberration of light. The motion of the earth through space around a circle 186 million miles in diameter, however, explains both. ==================== GEO Let me answer your question regarding the “two wobbles.” First, the parallax is explained by the following diagram: GEO The second movement of the stars relative to the earth is explained by the next diagram: GEO The earth is in the center. The DAILY rotation of the stars (or universe) is shown above by the axis marked north, going in the clockwise direction. The ellipses represent the YEARLY motion of the stars (or of the whole universe). The yearly motion is due to the wobble in the universe around the axis from the earth to the point marked “Pole of the Elliptic.” The YEARLY motion is not a rotation, but a back-and-forth motion. In the course of one year, the line marked Sun-A traces out the cylinder shown. At the same time, Stars 1, 2 and 3 trace out their respective paths. The dark circle shows the sun on the first day of winter. The open circle on the other side of the sun’s circuit is its position on the first day of summer, and is the same for stars 1, 2 and 3. If there were no daily rotation, stars 1 and 2 would always be to the left of the earth, and star 3 would always be to the right. This is how the Geocentric model explains the “wobbles” ==================== AC (2) The size of the Fresnel drag depends on the refractive index of the medium through which the light is passing. The resulting angle of aberration can be derived from Snell’s law of refraction using some simple mathematics, and is given by the following formula: angle of aberration (in radians) = (V / c) * (1 - (1 / R)) where V is the speed of the medium, c is the speed of light, R is the refractive index of the medium. The refractive index of space in the inner solar system is very close to 1. Putting this value into the formula gives us an angle of aberration that is very close to 0. No doubt you’ll ask “How close?”. Well, if R is 2 then the angle of aberration would be (18 / 186000) * (1 - 1/2), which is about 4.8E-5 radians, or about 10 seconds of arc. This is half the observed value. R would have to be greater than 2 to get an answer closer to the observed value. On the other hand, if R were 2 or more then there would be all sorts of strange refraction and total internal reflection effects when looking up out of the earth’s atmosphere at the night sky. We do not see these effects, and therefore R is much smaller than 2. So we must conclude that R is greater than 2 to explain the aberration of light (if it is caused by Fresnel drag), and that R is much less than 2 to explain the appearance of the night sky. Since R cannot be both greater than 2 and much less than 2, we must conclude that the aberration of light is not caused by Fresnel drag. It is in fact caused by the motion of the earth around the sun. ==================== GEO As I said in my last exchange, Fresnel drag is only one component of the answer. As you can see by the above diagrams, the other component is the daily and yearly motion of the sun and stars, of considerable degree, to account for the aberration. In fact, it does most of the explaining. But in regard to the Fresnel drag, I only used that term as what current science explains as light-frame dragging. I didn’t say it was the same thing as what is actually occurring. My post stated that the second possible explanation is that the sun has an aether-field around it which would drag the starlight. Although this is the same principle as the Fresnel drag, it is not the same thing, since modern science does not equate aether-fields with Fresnel drag. Gravity & where God “hung” earth AC I pose a few questions: These are not meant to disprove geocentric theory, just to clarify it. Do you believe it theoretically possible to evacuate a space completely, so no aether is within the volume? If so, should gravity not exist within the space? Or does aether, as J. C. Maxwell and others believed, permeate all things? If so, it cannot be responsible for gravity, because, if it easily travels through matter without resistance, how can it exert a force on heavenly bodies? If there is resistance, should not there be an aethereal drag force slowing everything down? ======================== GEO Gravity does not travel through matter. Gravity is a result of the disturbance an object makes in the aether. The more massive the object, the greater the tension it will cause in the aether, which will result in greater gravitational force. Since the aether is at Planck dimensions, that means that its reaction time is about 5.391 x 10^-44, and thus “gravity” can exist simultaneously over very vast distances. Science has already observed this in the immediate reciprocity of gravitational effects between the sun and the earth. Unfortunately for modern science, they have no way of accounting for these instantaneous gravitational effects, since they have concluded nothing can travel faster than light, which takes 8.5 minutes to travel from the sun to the earth. If gravity took 8.5 minutes to travel the 93 million miles, well, we wouldn’t be here to talk about it. Science also knows that the Planck particles exist, but since they’ve rejected aether theory, they are forced to say that the particles come into existence for 10^-44 seconds and then just disappear, popping in and out of existence continually. And they say that Geocentric theory is crazy?! ======================== AC At what height above the earth does gravity “stop”, as you apparently think it does? ======================== GEO Gravity doesn’t stop. Rather, the tension that an object creates on the aether is decreased the farther away one is from the object. ======================== AC Where in the Bible does it explicitly say the sun, or stars move AROUND the earth? ======================== GEO Wisdom 13:2 But have imagined either the fire, or the wind, or the swift air, or the circle of the stars, or the great water, or the sun and moon, to be the gods that rule the world. Job 9:7 He commands the sun, and it rises not; he seals up the stars. Psalm 8:4 When I see your heavens, the work of your fingers, the moon and stars that you set in place – Judges 5:20 From the heavens the stars, too, fought; from their courses they fought against Sisera. Wisdom 7:18-19:18 The beginning and the end and the midpoint of times, the changes in the sun’s course and the variations of the seasons. 19 Cycles of years, positions of the stars, Psalm 19:4-6 4 In them He has placed a tent for the sun, 5 Which is as a bridegroom coming out of his chamber; It rejoices as a strong man to run his course. 6 Its rising is from one end of the heavens, And its circuit to the other end of them; And there is nothing hidden from its heat. Sirach 43:1 The clear vault of the sky shines forth like heaven itself, a vision of glory. 2 The orb of the sun, resplendent at its rising: what a wonderful work of the Most High! 3 At noon it seethes the surface of the earth, and who can bear its fiery heat? 4 Like a blazing furnace of solid metal, it sets the mountains aflame with its rays; By its fiery darts the land is consumed; the eyes are dazzled by its light. 5 Great indeed is the LORD who made it, at whose orders it urges on its steeds. 6 The moon, too, that marks the changing times, governing the seasons, their lasting sign, 7 By which we know the feast days and fixed dates, this light-giver which wanes in its course: 8 As its name says, each month it renews itself; how wondrous in this change! 9 The beauty, the glory, of the heavens are the stars that adorn with their sparkling the heights of God, 10 At whose command they keep their place and never relax in their vigils. A weapon against the flood waters stored on high, lighting up the firmament by its brilliance, 1 Esdras 4:12 The earth is vast, the sky is lofty, the sun swift in his course, for he moves through the circle of the sky and speeds home in a single day. How great is he who does all this Joshua 10:10-12 On this day, when the LORD delivered up the Amorites to the Israelites, Joshua prayed to the LORD, and said in the presence of Israel: Stand still, O sun, at Gibeon, O moon, in the valley of Aijalon! 13 And the sun stood still, and the moon stayed, while the nation took vengeance on its foes. Is this not recorded in the Book of Jashar? The sun halted in the middle of the sky; not for a whole day did it resume its swift course. 14 Never before or since was there a day like this, when the LORD obeyed the voice of a man; for the LORD fought for Israel. Sirach 46:3-4 And who could withstand him when he fought the battles of the LORD? 4 Did he not by his power stop the sun, so that one day became two? Ecclesiastes 1: 5-6 The sun rises and the sun goes down; then it presses on to the place where it rises. 6 Blowing now toward the south, then toward the north, the wind turns again and again, resuming its rounds. ======================== AC In the verses you mention, I have only read statements saying that the Sun moves and the earth does not. I interpret this as meaning simply that the Sun moves relative to the inhabitants of the Earth, which it does. We’re all on the Earth, so it makes more sense this way. God says in several places that he has “established” the Earth, meaning that he has caused it to move in a stable, fixed, elliptical orbit. ======================== GEO If there were any passages in Scripture that say the earth moves, you would have a point, but there are none. The passages which refer to the earth from an astronomical perspective say that it is not moving. For example, Psalm 93:1-2 The LORD reigns, He is clothed with majesty; The LORD has clothed and girded Himself with strength; Indeed, the world is firmly established, it will not be moved. 2 Your throne is established from of old; You are from everlasting. 1 Chronicles 16:30 …tremble before him, all the earth; yea, the world stands firm, never to be moved. Psalm 104:5 He established the earth upon its foundations, So that it will not totter forever and ever. Psalm 119:90 Your faithfulness continues throughout all generations; You established the earth, and it stands. In these references the word “establish” is the Hebrew kun. It is in the Niphal form, which means it was made such by an outside agent (God). But you should know that kun has a wide variety of meanings. The meaning the heliocentrists often attach to the above verses is that “establish” merely means to “begin” or “found.” We often use this meaning in English in saying, “That building was established in 1924,” meaning that it was founded in 1924. But kun means something more. Kun can refer to something fixed and immovable. Kun’s variation in meaning depends on the context it is placed. For example in 1 Chronicles 22:10 it reads: “He shall build a house for My name, and he shall be My son and I will be his father; and I will establish the throne of his kingdom over Israel forever.” Obviously, “establish” cannot mean merely “to begin.” It refers to the continual upholding of the state in which it began. Also, in Judges 16:26, 29: “And Samson grasped the two middle pillars upon which the house rested, and he leaned his weight upon them, his right hand on the one and his left hand on the other.” The word “rested” is the Hebrew kun, depicting a house that is motionless and stationary upon the pillars it was built. Ezra 3:3 gives the same meaning: “They set the altar in its place, for fear was upon them because of the peoples of the lands...” Another issue is the use of the Hebrew mot in Psalm 93:1; 1 Chron 16:30; Psalm 119:90. When applied to physical objects, this word refers exclusively to movement (cf. Jb 41:23; Ps 125:1; 140:10; Is 40:19; 41:7). But in the usages of mot, the only time the earth is said to move is in the apocalyptic language of Is 24:19, which coincides with the apocalyptic language of Mt 24:29- 30; 2 Peter 3:10-13, but that is at the end of the world. ======================== AC God says in several places that he has “established” the Earth, meaning that he has caused it to move in a stable, fixed, elliptical orbit. ======================== GEO But do you see what you are doing here? You are interpreting the verse literally as referring to the earth’s astronomical dimensions, rather than interpreting the verses figuratively as referring to the earth’s strength or longevity. But interpreting literally is the same thing that I do with the verse, but much more in-line with what the verse says, since it also says the earth does not move, whereas your literal interpretation says it does move. ======================== AC You are basically right in that it is possible that the Sun could move around the Earth. The theories of gravity state that it is a question whether one body orbits the other or vice versa is simply a question of which body is most massive. Anyway, yes, through God all things are possible. The reason for the accepted theories of physics is that they are simpler and don’t require God to move everything around. They do not imply that the Bible is wrong, or that God does not exist. On the other hand, your personal beliefs about gravity make no sense whatsoever. ======================== GEO First of all, they are not my “personal beliefs” about gravity. They follow the theory of gravity developed by LaSage over 400 years ago, a theory he received from his predecessors. LaSage’s corpuscular theory answers to the Planck dimensions of the aether I mentioned above. Second, if there is any system that has “personal beliefs” about gravity, it is the Newtonian system. Why? Because Newton did not have a physical explanation for what gravity IS. The only thing Newton did is put the EFFECTS of gravity in mathematical equations. Anybody can do that. But Newton has no explanation for gravity itself, nor does modern science. They speak today of “gravitons,” but no one has ever seen or distinguished a graviton. And without an explanation for gravity, this means that most of what science claims it knows is at best speculative, and at worst, absolutely wrong. As for mass determining orbit, Newtonian mechanics does not even say that. The origin of orbit is not explained by Newton, only how the orbit sustains itself by opposing forces. And in the Einsteinian universe, one cannot say that one object is going around another, since all motion is relative. Some planetary motions, from spacecraft observations AC Our spacecraft to the outer parts of the solar system (Voyager, Magellan, et al) have observed that the superior planets and asteroids are NOT moving in epicycles. For example, when the planet appears from Earth to be moving in retrograde (westward relative to the stars) motion it is NOT actually moving westward, but only appears to be doing so due to Earth's motion. Whether the planet is changing direction (making loops) is easily determined by positional measurements by spacecraft. But, if the planets were making loops, our spacecraft would have never been able to rendezvous with them since our trajectories are based on the correct (heliocentric) picture of the solar system. ==================== GEO You are, of course, referring to the retrograde motion of the outer planets, but this is easily explained in the Geocentric system. GEO In this diagram, the earth is located on each of the nine cross-hairs. It is stationary. The sun is revolving around the earth in the smaller circle, while Mars is revolving around the sun in the larger circle. Each cross-hair frame represents one moment in the sun's revolution around the earth and the same moment of Mars' revolution around the sun. The squiggly line represents how Mars would be viewed from earth at each frame. You can see that by frame #4, Mars has already started its retrograde. Incidentally, this is the same retrograde you would see in the heliocentric model. ==================== AC By-the-way, I am curious whether you assume that the Earth is rotating? There is evidence to prove the rotation of Earth, but I am not sure if you accept Earth’s rotation. Also, I would still like your response to my questions at the bottom of my first e-mail (below) – about the sun being the center of the galaxy. ==================== . GEO No, I do not believe the earth is rotating, and there is no proof that it is. Most of the socalled “proofs” have already been answered in previous exchanges. Sagnac & an Einstein cover-up AC I was a little perplexed by two aspects of your response in a previous exchange. Firstly, in your discussion of the Sagnac effect, are you referring to the same effect so elegantly accounted for here in the link below? Note the author’s assertion that the effect in no way contradicts relativity, although it is not itself a relativistic effect. http://www.mathpages.com/rr/s2-07/2-07.htm ======================== GEO Obviously, Relativity needs to have an explanation for the Sagnac effect, but according to physicist Herbert Ives (which no Relativist I know has ever refuted), Relativity CANNOT explain the Sagnac effect. One of the most important but overlooked books on this subject is The Einstein Myth and the Ives Papers by Dean Turner and Richard Hazelett. I’ve read the book from cover to cover making extensive notes along the way. If you’re a math buff, Ives has pages and pages of calculus to prove his point. I suggest you give it a good review. The Sagnac material is located in Part IV, page 247ff. Moreover, the Relativistic attempts to deal with the Sagnac effect have incorporated aether in the solution, so it is not really a Relativistic solution (See Post, E. J., 1967, Rev. Modern Physics, 39:475). ======================== AC Secondly, early on you seem to suggest that Michelson-Morley experiments fail to detect aether drift because the earth is stationary with respect to the aether. Later on, however, you suggest that heavenly bodies are embedded in an aether rotating with respect to the Earth. If the latter, surely Sagnac apparatus and Michelson-Morley experiments should give results different from those observed? ======================== GEO Actually, the 1887 M-M experiment did have a small positive result. Unfortunately, modern physics books don’t point this out at all. And they detected that small positive result in only 36 trials, and with their somewhat primitive equipment. Dalton Miller’s experiments of 1933, which he repeated over 100,000 times, showed the same positive result as M-M. His apparatus was four times as sensitive as M-M’s. Yet Einstein and R. Shankland tried to discredit Miller’s results by using the few trials that even Miller himself had thrown out due to temperature effects and faulty equipment. Thus, M-M, Sagnac, Miller, as well as Michelson Gale of 1925, all detected an aether in drift. What they couldn’t tell us was whether the aether was moving around the earth or the earth through the aether. But in the geocentric system, of course, the aether drift is caused by the rotating universe, not a rotating earth. By the way, Einstein makes no reference to Sagnac or Michelson-Gale in any of his works. More on gravity AC When you have an earthquake the day is a little bit longer. Most people believe that that is because the motion disturbs the orbit of the earth. Alternatively you could say that the information of the earthquake is instantaneously transmitted to the whole of the universe which then changes the speed of its motion, thereby violating conservation of energy, one of the most basic precepts of Newtonian mechanics. And when you say there is no proof of the existence of gravity, why not try and fly from a high story building. Mach’s experiment does not serve the purpose you advocate, and I do not think that you understand it properly. GEO The following words of Ernst Mach: ...all masses, all motion, indeed all forces are relative. There is no way to discern relative from absolute motion when we encounter them...Whenever modern writers infer an imaginary distinction between relative and absolute motion from a Newtonian framework, they do not stop to think that the Ptolemaic and Copernican are both equally true. (Die Mechanik in ihrer Entwicklung historisch-kritisch dargestellt, eighth ed, Leipzig, p. 222, 1921). I have never advocated that gravity doesn’t exist. I said that the Newtonian system has never explained what gravity IS. The only thing Newton did is put the EFFECTS of gravity in mathematical formulae. That is not hard to do. But Newton himself admitted that he had no explanation for the substance or nature of gravity. Gravity is a result of the disturbance an object makes in the aether. The more massive the object, the greater the tension it will cause in the aether, which will result in greater gravitational force. Since the aether is at Planck dimensions, that means that its reaction time is about 5.391 x 10^- 44, and thus gravity can exist simultaneously over very vast distances. Science has already observed this in the immediate reciprocity of gravitational effects between the sun and the earth. Unfortunately for modern science, they have no way of accounting for these instantaneous gravitational effects, since they have concluded nothing can travel faster than light, which takes 8.5 minutes to travel from the sun to the earth. If gravity took 8.5 minutes to travel the 93 million miles, well, we wouldn’t be here to talk about it. Science also knows that the Planck particles exist, but since they’ve rejected aether theory, they are forced to say that the particles come into existence for 10^-44 seconds and then just disappear, popping in and out of existence continually. And they say that Geocentric theory is crazy?! Church position on geocentricity AC I was reading an old Remnant magazine and I came across on article which addressed Geocentricism. Part of the article reads: The reigning Pope Paul V declared that the proposition that the sun is the center of the universe is “philosophically foolish and absurd and is formally heretical, inasmuch as it expressly contradicts the teaching of many texts of Holy Scripture, both according to their literal meaning and according to the common explanation and interpretation of the Holy Fathers and learned theologians.” This judgment was reaffirmed as infallible by Urban VIII and Alexander VII. (Bull Speculatores Domus Israel, 1664) What would your thoughts be on this, especially the statement that Urban and Alexander infallibly affirmed the geocentric understanding of those verses? Thanks. ======================== GEO The above analysis is a little distorted. First, the quote attributed to Pope Paul V was not from him directly but from the Congregation of the Index of March 5, 1616. It is assumed that Paul V approved these words, however, since he presided at this session of the Inquisition where the matter was discussed and decided. His name, however, does not appear in the decree. Neither Urban VIII nor Alexander VII stated that the March 5, 1616 decree by the Congregation of the Index was “infallible.” To my knowledge, no statement issued by the Congregation of the Index was ever deemed infallible, and no pope endorsing a decision by the Index was ever deemed infallible. Urban VIII’s statement enforcing the Index’s prohibition of Copernicus’ book was issued “in forma communi” (a formal communication) but that has never been considered as infallible. Moreover, Alexander VII’s Speculatores Domus was not a “bull,” but rather a papal “brief.” In 1664, Alexander VII published a new official Index, which differed from prior ones in style but had the same content. The only things Alexander VII added were the Index’s prohibitions from 1596 to 1664, which the previous Index did not have. That was the essence of the “brief.” As such, the prohibitions against the Copernican system were included by Speculatores Domus, but none of these documents have the rank of “infallibility.” Infallibility can only come from a dogmatic ecumenical council or from a papal decree issued ex cathedra and fulfilling all the criteria of Vatican One definition of infallibility. Third, in 1758, Benedict XIV removed Copernicus’ books from the Index, but this was not a decision that was approving of Copernicanism. Benedict was merely following through with what was originally decreed by Pope Paul V in 1616 through the Congregation of the Index that Copernicus’ book would be banned “until corrected.” In 1620, a list of the needed corrections was issued by the Index. They demanded the removal of nine sentences from Copernicus’ book, sentences that had affirmed the heliocentric system as a certainty. Once the certainty of the system was removed, the book was taken off the Index. AC As to the Pope’s attendance at the session of the Inquisition, Fantoli states: “It is clear from both documents that the pope did not take part in the session, contrary to what is generally stated.” ======================== GEO Fantoli is dependent on the same evidence upon which everyone else is dependent. We all know that Pope Paul V’s name does not appear on the March 5, 1616 decree of the Congregation of the Index. That is not in dispute. But everyone recognizes that Paul V was presiding at the Inquisition. Moreover, Paul V would have had to approve of the Congregation’s ban on Copernicus’ books, since he approved of every other decision made by the Congregation of the Index. Thus, “not taking part in the session” does not really prove anything for Fantoli. ======================== AC Instead Fantoli writes: A decree of the Holy Office, September 11, 1822 stated: “The most excellent [cardinals] have decreed that there must be no denial...of permission to print and to publish works which treat of the mobility of the earth and of the immobility of the sun, according to the common opinion of modern astronomers as long as there are no contrary indications, of the basis of the decrees...” This decree was approved two weeks later by Pope Pius VII...With this decree of the Holy Office the official dossier regarding the Copernican question is closed...Thirteen years later in 1835, on the occasion of the new edition of the Index of forbidden books, the Copernican books...as well as Galileo’s Dialogue...were finally removed from the list. ======================== GEO This information neither negates nor disagrees with any of the information I have stated previously. In fact, it just strengthens my case. Already as early as 1616, as I have previously stated, the Congregation of the Index said that Copernicus’ book would be removed from the Index if nine statements asserting the certainty of heliocentrism were excised. Pope Urban VIII affirmed this decision. In 1620, the list of those nine statements were given to the editors of Copernicus’ book. The editors eventually removed those nine statements. This paved the way for Pope Pius VII to take the Copernicus’ book off the Index, which he did. But in doing so, Pius VII confirmed that Copernicanism could not be taught as a certainty, in agreement with Pope Paul V and Pope Urban VIII. Thus, there is no discrepancy in this matter. Moreover, if Pope Pius VII’s decision means the matter is “closed,” that means that Copernicanism still cannot be taught as a certainty, for that was the last word given from the papal office. Moreover, note the wording of Pius VII: “...according to the common opinion of modern astronomers as long as there are no contrary indications of the basis of the decrees...” First, he refers to the “opinion” of modern astronomers, and thus he does not say that their teaching of heliocentrism is a fact. It is only an “opinion.” Second, his statement: “as long as there are no contrary indications, of the basis of the decrees...” refers right back to Pope Paul V and Pope Urban VIII, whose “decrees” said that Copernicanism could not be taught as a certainty. Again, Pius VII is in complete agreement with Paul V and Urban VIII. ======================== AC Re: the question of ‘orthodox’ Catholics interpreting Scripture non-literally when it comes to passages allegedly teaching geocentrism, I ask if it can be said that these Catholics are instead interpreting these passages literally, given the context of the biblical authors? My understanding is that the exact or primary meaning of a word or words is to be upheld for there to be a literal interpretation. Therefore the phrase ‘the sun rises or sets,’ if it is taken to be a scientific affirmation, literally means the sun is in actual movement over the earth. If the phrase is taken to be a sensorial affirmation, then literally it means the sun by appearance moves over the earth. I am assuming – for I have not checked all the passages – that the intention of the biblical authors who speak of the sun rising, etc. is to speak in a common manner and not scientifically. ======================== GEO As I said before, the sun rising or setting language is not the main basis for the geocentric understanding of the Bible. The passages which say the sun was stopped from moving; the passages which say the sun moves in its circuit in one day; and the passages which say the earth does not move, are the bases of the geocentric position as far as Scripture is concerned. The numerous passages that speak of the rising or setting are merely additional information which coincides with the primary information. (The primary passages which teach geocentrism are included in previous exchanges). As such, I can agree that the “rising or setting” language, if we had no other information, could be taken as expressing that the sun moves with respect to a stationary earth; or phenomenologically, that is, it appears the sun is moving. But the problem is that unless you have a way of dealing with the other more explicit Scriptures of the sun’s movement and the earth’s immobility, then you can’t rest your case on the rising and setting language. ======================== AC There must be no clues that the authors intended to propose scientific data to their audience, otherwise it would seem that the Holy See would not have allowed Catholic authors to defend the Copernican theory. ======================== GEO The Holy See allowed Catholic authors to defend Copernicanism as long as they did not treat it as a certainty. The Holy See also agreed that, if it could be proven that the earth went around the sun, then biblical interpretation must incorporate that fact. But the fact remains that Copernicanism was not proven then, and it has not been proven today. ======================== AC Therefore it would seem that one can argue that the biblical authors were simply doing as we do when we commonly speak of the sun rising and setting, intending thereby to only make sensorial affirmations. ======================== GEO You can argue that position all you like, but whether it is right or not, considering the above information I have given, is another story altogether. ======================== AC Also, Pope Leo XIII in Prov. Deus states: From the fact that we must take a position of strenuously defending the Sacred Scriptures it does not follow that we should maintain equally all the opinions expressed by individual Fathers and later by their interpreters in the act of declaring its meaning. In fact, in the case of the explanation of Scriptural passages which deal with physical questions, they held to the opinions of their time with the results that they perhaps did not always judge truthfully and stated things which are no longer approved today. ======================== GEO By what things are these? The heliocentrists who read this passage from Leo assume he is talking about heliocentrism, but Leo doesn’t say that, does he? He could be talking about the size of a mustard seed or whether Saturn was a star (like Augustine believed), or any number of things. Unless he specifically says that he is referring to heliocentrism, you can’t make a case with it. Geocentricity in the 17th c. Other matters in the 20th c. AC You state: The Inquisition of 1615 ... declared the position of Galileo to be scientifically false, and anti-Scriptural... Following this was a decree ...prohibiting various heretical works, and among them were those advocating the Copernican system.... and no pope has ever annulled the decrees of Paul V or Urban VIII. The only thing the Church has done is apologized for the treatment of Galileo, but with no reference to his science views. The way you present matters above it sounds as if the Church has decrees still in force which declare Galileo’s position anti-Scriptural and that advocating the heliocentric position as an opinion is prohibited. Instead, as I had recently pointed out, the decrees against Galileo and the Copernican system were annulled in the 1800’s with the approval of Pius VII. Galileo’s Dialogue was removed from the Index in the 1800’s, and in that same century the Church decreed that one can advocate the opinion that the earth is mobile and the sun is immobile, in accordance with certain previous decrees. ======================== GEO The decrees against Copernicus and Galileo were not annulled. The Church still cannot allow anyone to teach, in the name of the Church, Copernicanism as a CERTAINTY. THAT is the decision that was never nullified. I have no problem if you want to say that someone can hold the opinion that the earth goes around the sun, but that is far different than saying the Church holds Copernicanism as a certainty. ======================== AC Also, you had stated that Paul V clearly approved the decision of the Inquisition against Galileo based on the fact that you say he was present at the session of that Inquisition. Previously I had quoted the view of a renowned Galileo scholar who insisted the Pope was not present at the session (this is not to say, however, that the Pope did not approve the decision of the Inquisition), without going into detail as to how he reached his conclusion. You seemed to dismiss his view saying that others have read the same documents he did , but they reached a different conclusion. However, Annabale Fantoli’s view is based, I believe, on the recently discovered minutes of a weekly cardinals meeting which took place the day after the session of the Inquisition in question. Fantoli in his book provides the Latin text of the meeting, as well as an English translation of it. I believe it is from those minutes that one can see that the Pope had to be notified of what decision the Inquisition had reached against Galileo, thereby showing the Pope was not present at the session in question. Also, Fantoli draws the same conclusion based at the same time on some other document that I didn’t look closely to see what it was. ========================
  24. GEO I never said that NASA couldn’t tell someone where to find the satellite. All I said was that the trajectory on the pictorial was a computer-generated image based on the input the computer receives regarding a rotating earth. I didn’t say the pictorial was wrong, at least not from a rotating earth framework. In fact, Gary Emerson can use that computer-based imagery and know that, from the perspective of a rotating earth framework, Chandra is going to be over a certain place at a certain time. However, Chandra does not have a big pencil that reaches to the earth and by which it marks out a trajectory. Once again, the image we see in NASA’s pictorial is exactly what one would expect to see from a rotating earth. But that doesn’t prove that the earth is rotating. All it proves is that if one assumes a rotating earth, the trajectory will follow the path depicted in the pictorial. But if one assumes a stationary earth and a rotating star system, the trajectory is going to be different. In this case, NASA would have to program the computer to assume a stationary earth, and then feed the information about Chandra’s ellipses, inclination and speed into the computer. The computer, programmed for a stationary earth, would then calculate the pictorial trajectory of Chandra. But in the case of a stationary earth, the earth would be showing the same face toward Chandra about 80% of the time (based on a 2.5 day flight-path and an apogee of 80,000 odd miles). In this case, the trajectory the computer will transcribe on the pictorial will be much simpler, that is, it will not have a long zig-zagging and looping line going all over the globe, rather, it will have a simple descending line in the shape of a horseshoe. It just so happens that the horseshoe trajectory of the stationary earth will circumscribe the zig-zagging and looping trajectory of a rotating earth AT EVERY POINT, since both systems account for Chandra’s whereabouts at all times. The computer could then make two three-dimensional graphs, one for the horseshoe trajectory in the stationary earth and one for the zig-zagging/loop trajectory in the rotating earth, and then, using advanced trigonometry, superimpose one pictorial over the other and we would see the same precise trajectory in each graph. However, this does not prove which one is right, since both models work. ======================== AC If anybody still doubts that the geosynchronous satellites move the way NASA says they do, here’s a picture of a couple of them – AC This is a long-exposure picture, meaning that the camera’s shutter was left open for several seconds. That causes the stars to show up as parallel streaks because the earth rotates while the shutter is open and the film is exposed. The dot near the center of the picture is a geostationary satellite. It’s moving in synch with the earth’s rotation, so it still shows up as a dot. The bright streak that’s moving at an angle to the stars is a geosynchronous satellite in an inclined orbit. It’s either doing part of a zigzag or a figure-eight; it’s impossible to tell which from this picture. So, there you have it. I’ve proved that the earth rotates, because by doing so it causes certain satellites to move in otherwise inexplicable patterns. You made a half-hearted, though utterly unconvincing, attempt to account for this motion. ======================== GEO You assume that the streaks are made by the moving earth camera against the stationary stars. (The Geocentric system says it’s the stars moving against a stationary camera on earth). But merely asserting it doesn’t prove anything for you, since you have no way of telling which is correct. You can’t prove it by the bright diagonal line made by the Geo satellite, because, as I explained in my previous post, any satellite at an inclination with respect to the earth’s equator is going to have some kind of oscillating movement. As you said, however, we can’t tell from the photograph what kind of movement it has, either a figure-eight or a zig-zag, and thus we can’t tell what its inclination is. Nevertheless, in the Geocentric framework, it is oscillating. In your model it is merely following an elliptical path around the earth. Both systems work, so this provides no proof for you. By the way, I think it is worth mentioning that all these photographs are taken at night with the stars as the background. Obviously, they couldn’t take the pictures during the day, since the brightness of the sun would not allow us to see either the stars or the satellites. But this also puts your search for proof at a disadvantage. Why? Because judging movement based on the stars as the background is, shall we say, begging the question, since you must first prove that the stars are stationary and the earth is rotating before you can conclude what is actually moving in the photograph. But obviously, you can’t prove a point by using an unproven assumption. About NASA’s satellite tracking system GEO It seems that, whether inadvertently or intentionally I don’t know, are trying to obfuscate the issue. I hope its not intentional. I never said that NASA couldn’t tell someone where to find the satellite. All I said was that the trajectory on the pictorial was a computer-generated image based on the input the computer receives regarding a rotating earth. Someone else said quite a bit more than that: “Your pictorial exhibit of Chandra’s movement is not real. It is a computer-generated image of the trajectory Chandra would transcribe on the earth if the earth were rotating.” But because he knows the earth isn’t rotating, he knows that Chandra doesn’t follow the path over the ground that NASA predicts. In fact, he said, “by all appearances, you’ve been duped by a computer.” But then I pointed out that astronomers use that same data to aim their telescopes, and so now this person has to do some frantic backpedaling. Now he says: “I didn’t say the pictorial was wrong, at least not from a rotating earth framework.” In fact, Gary Emerson can use that computer-based imagery and know that, from the perspective of a rotating earth framework, Chandra is going to be over a certain place at a certain time. Which means, of course, that the computer-based imagery is accurate. If it says Chandra’s going to be “over a certain place at a certain time,” it will be. And if an astronomer happens to be somewhere beneath that flight-path, he can program NASA’s data into his computer-controlled telescope, and the telescope will point straight up at the appointed time. So, it seems that this other person is now conceding that Chandra really does follow the ground-track that NASA predicts. ======================== AC Therefore, my original question remains unanswered: If the earth isn’t rotating, how do you get a satellite to move around it in a crazy pattern like the one shown above? ======================== GEO I said the computer charts the pattern as if the earth were rotating. Chandra isn’t making the lines on the graph, the computer is. The computer can do nothing else than what is put into it. If the computer is programmed to assume a rotating earth, then it must chart the course of Chandra on the earth AS IF someone on the ground watching Chandra were rotating against Chandra’s orbit every 24 hours. Since Chandra’s orbit is at an incline, this will account for the vertical lines on the graph. Since Chandra is circling west to east around the earth, this will account for the horizontal lines on the graph. But this doesn’t mean the earth is rotating. It could also mean that the earth is actually stationary but that the computer must figure out what a graph would look like if the earth were rotating and Chandra’s whereabouts, with respect to a specific spot on the earth, needed to be charted. If you then tell the computer that the earth is stationary and ask it to tell you Chandra’s whereabouts with respect to a specific spot on earth, it will do so, but it will make a chart on the graph which is different than the squiggly lines in Emerson’s original graph. Both graphs are true records of the systems they represent, but neither graph proves that the system they represent is the actual system operating. As previously mentioned, Chandra does not have a big pencil that reaches to the earth and by which it marks out a trajectory. ======================== AC But if it did, it would mark out exactly the trajectory shown on NASA’s maps. Again, the fact that astronomers use this same trajectory data to aim their telescopes proves that Chandra is right where NASA says it will be relative to the surface of the earth. ======================== GEO No, it only proves that the system they are using will account for Chandra’s whereabouts with respect to a rotating earth, but it does not prove that the system is a reality, that is, that the earth is rotating. Once again, the image we see in NASA’s pictorial is exactly what one would expect to see from a rotating earth. But that doesn’t prove that the earth is rotating. All it proves is that if one assumes a rotating earth, the trajectory will follow the path depicted in the pictorial. ======================== AC No, the satellite follows that trajectory whether the earth rotates or not. ======================== GEO No, you haven’t shown that. You haven’t asked NASA to produce a pictorial of Chandra’s lines assuming a non-rotating earth. NASA could do so if they wished, or perhaps if you made that request of them. ======================== AC In other words, if you’re standing somewhere on that looping yellow line, and you look up at the right time, you’ll see the Chandra satellite. If the earth rotates, the ground-track makes sense. It’s simply the result of superimposing a highly elliptical orbit over a rotating globe. ======================== GEO Yes, you will see Chandra, but that’s because the computer has already shown you where the line of sight would be assuming that the earth is rotating. You must realize, and it has been hard for you to do so throughout these dialogues, that your system is not the ONLY one that can explain the motions we see in the sky. You think that just because his system works, then it discounts all other systems. But that is a fallacy. I think you’ve seen those tricky drawings that, if looked at in one way, the drawing looks like a lamp. If looked at another way, it looks like two people kissing. Same principle here. ======================== AC But if the earth isn’t rotating, that means that somehow Chandra is able to go looping wildly around it, like a bee circling its nest. ======================== GEO No, because I’m saying that if the earth were not rotating, then the computer would not make all those squiggly lines. It would make a simple horseshoe shape or thereabouts. But if one assumes a stationary earth and a rotating star system, the trajectory is going to be different. In this case, NASA would have to program the computer to assume a stationary earth, and then feed the information about Chandra’s ellipses, inclination and speed into the computer. The computer, programmed for a stationary earth, would then calculate the pictorial trajectory of Chandra. But in the case of a stationary earth, the earth would be showing the same face toward Chandra about 80% of the time (based on a 2.5 day flight-path and an apogee of 80,000 odd miles). In this case, the trajectory the computer will transcribe on the pictorial will be much simpler, that is, it will not have a long zig-zagging and looping line going all over the globe, rather, it will have a simple descending line in the shape of a horseshoe. ======================== AC Yes, if the earth weren’t rotating, a satellite in an elliptical orbit would follow a simple trajectory relative to the ground. But because the earth is rotating, that satellite follows the bizarre trajectory shown in the image above instead. But whether the earth is rotating or not, that is Chandra’s actual flight-path over the ground. That’s where you have to aim your telescope if you want to see it. How can you account for that flight-path if the earth isn’t rotating? ======================== GEO I’ve already accounted for it by saying that the flight path is what an observer would see from earth based on a rotating earth. The computer gives that to him. The computer will say to him: “If you want to see Chandra, be at such and such a location at such and such a time” and it will be correct. Or the computer can say: “Point your telescope in such and such a direction at such and such a time” and you will be able to see Chandra. Why? Because the computer has already assumed a rotating earth. But you could also tell the computer that the earth is not rotating and then the computer will come back and tell you: “Be at such and such a place at such and such a time” and you will be able to see Chandra. In both cases, the computer will give you the exact same location, whether its for a rotating earth or a stationary earth. The only thing that will be different is the trajectory line the computer traces out in each system. It just so happens that the horseshoe trajectory of the stationary earth will circumscribe the zigzagging and looping trajectory of a rotating earth AT EVERY POINT, since both systems account for Chandra’s whereabouts at all times. ======================== AC Except that if the earth isn’t rotating, then Chandra isn’t in an elliptical orbit; it’s in a bizarre, spirally, loopy orbit. Mathematically, you could assume that the earth is stationary, and that Chandra is moving in a bizarre, spirally, loopy orbit. But in real life, it’s just not possible for a satellite to move like that (not without a lot of fuel and a very powerful rocket). Therefore, we can rule out the idea that the earth is stationary. Instead, the flight-path over the ground followed by Chandra is the result of superimposing its simple elliptical flight-path over a rotating earth. ======================== GEO No, because you’re basing that answer on the graph NASA gave you, not on proof that the earth is rotating. If NASA’s computer graph is made by merely assuming a rotating earth, then the graph doesn’t prove anything for you. Hence, Chandra wouldn’t have the “bizarre, spirally, loopy orbit” that you claim, since you are basing that answer on an unproven premise that the computer graph represents the ONLY possible reality. More about inertial effect on satellites GEO First, let me reiterate from a previous post that the gravitational force from the rotating stars (which is confirmed by the Lense-Thirring effect, and which you still have not addressed), and the gravitational force from the earth itself, both act upon the satellites. ======================== AC The “Lense-Thirring effect” is an effect predicted by general relativity. You don’t believe in general relativity. Therefore, the Lense-Thirring effect is not available to you as an explanation for how satellites move the way they do in a non-relativistic, geocentric world. You can’t have it both ways. You can’t disclaim relativity and then appeal to it as the explanation of how satellites move. ======================== GEO I can, because one of the more convincing ways to cast doubt on your opponent’s scientific presuppositions is use his own science against him. If you adopt Relativity, then you must adopt the aspects of Relativity that disprove your Heliocentric universe. You need to prove your case. If part of the scientific theory with which you are working (and of which I have made you aware) denies what you are proposing, then it behooves me to point that anomaly out to you. Second: Let’s take a closer look at Marisat-3 which produces a figure-eight pattern. The reason it produces this pattern is that, first of all, its position with respect to the equator is about 10 degrees in inclination. ======================== AC Only if it’s orbiting, which you deny. Remember, “inclination” is the angle between a satellite’s orbital plane and the plane of the equator... ======================== GEO Not really. In my system, if Marisat is placed in position off the equator by the Pythagorean equivalent to what would be 10 degrees in your orbital inclination, then it will act with respect to that off-center placement. Since the equilibrium of gravitational forces lies only on the equator, then any satellite which is inclined to any degree against the equator is going to produce some type of oscillation. ======================== AC What makes you think there’s an “equilibrium of gravitational forces” that “lies only on the equator”? I think a lawyer would say this answer “assumes facts not in evidence.” You need to substantiate claims like this, not just assert them as conclusions following words like “since” and “because.” ======================== GEO First, this is not a court of law. It is a scientific forum. Both of us present evidence to this case which some might consider hearsay, conjecture, circumstantial evidence, etc. As I’ve said before, the mere fact that you cannot tell us what gravity IS, means you are working with a system of evidence which lacks the physical evidence required in a court of law. Your mathematical formulae without a physical model, if I may use your analogy, are merely circumstantial evidence, which, as you know, can be interpreted in a number of ways. Second, I am again using the scientific evidence that you use in your system, and upon which gravitational mechanics works, that is, that there is a neutral point of gravity and the centrifugal effect at the equator. We know this because the Foucault pendulum does not work at the equator. There are two explanations for this: your system or my system. In my system, the central plane of the geocentric universe cuts through the earth’s equator, and thus explains the neutrality of the Foucault pendulum at that point. How would this occur in a Geocentric universe in which the satellite is not orbiting the earth but is merely hovering about the earth, moving only slightly? Let me make an analogy. Let’s say you are in a room. On one side of the room there is a 1,000 lb electro-magnet. On the other side there is a 100 lb electro-magnet. Both magnets are turned on and operating. There is a force from each magnet. Somewhere in the room, closer to the 100 lb magnet, there is an equilibrium of counteracting magnetic forces, such that if a metal object were placed in that equilibrium, the metal object would remain suspended within the magnetic force. (There are plenty of experiments that do this very thing). To test this out, you place a steel ball in the equilibrium position between the two magnets. You see that the ball floats in mid air, suspended by the force of both magnets. ======================== AC Okay, no problem so far. ======================== GEO Now, before I go any further, if you haven’t figured it out already, the 1,000 pound magnet represents the force of gravity from the stars. The 100 pound magnet the force of gravity from the earth. ======================== AC If you are planning to argue that geosynchronous satellites are floating in an equilibrium position between the earth (the 100 lb. magnet) and the stars (the 1,000 lb. magnet), you’re going to have to explain why the moon, which is way past this “equilibrium point,” doesn’t go zipping off toward the stars. How can it orbit the earth if it’s hundreds of thousands of miles on the stellar side of this alleged “equilibrium point”? ======================== GEO Because the moon is moving, which causes additional centrifugal effects, that a geosynchronous satellite does not have. The steel balls represent the satellites. And one point of clarification before I proceed: the force of the stars, according to the Lense-Thirring Effect, is created by the billions of stars all acting upon the earth at the same time. These stars, as they rotate in their shell, produce large gravitational effects, according to the Lense-Thirring principle, supported by Einstein himself. ======================== AC But you say Einstein’s theories are “an absolute farce.” Therefore, you can’t appeal to them to explain how things move in you non-relativistic, geocentric model. ======================== GEO See my previous answer. This immense amount of stars makes up for the fact that the stars are far away. I say this to counter the idea that you were perpetuating, that the stars have a negligible force on the earth. ======================== AC Even if we assume that each star exerts tremendous force on us, the fact that we’re surrounded on all sides by such stars means the forces would cancel out and the net effect would be zero. That is especially true if, as you say, the earth is the center of the universe. ======================== GEO If you were at the very center of the earth the forces would cancel each other out, for then you would be equidistant from all the stars. But the fact is that, anywhere on the surface of the earth, you are 4,000 miles closer to one side of the star system than the other, since the diameter of the earth is about 8000 miles. Now back to the analogy. You place a steel ball in the exact place where the equilibrium of the magnets exists. What do you see? The steel ball remains motionless. But on your second attempt, you place the steel ball just a little left or right of the equilibrium point. What are you going to see? ======================== AC You’re going to see the ball accelerate toward the stronger magnet and smash into it. Once you move away from the equilibrium point, you move into an area where one magnet is stronger than the other. Therefore, the net force on the ball is going to pull it toward that magnet, as shown in this figure: In the above figure, you can see that at the equilibrium point between the two magnets, the force of attraction on the ball is balanced, and therefore, as you say, the ball isn’t going to move. But if you move the ball closer to the 1,000 lb. magnet the force of attraction from the 1,000 lb. magnet is going to be stronger than the force of attraction from the 100 lb. magnet, and the ball will accelerate toward the 1,000 lb. magnet. (The same thing will happen, of course, if you move the ball closer to the 100 lb. magnet.) ======================== GEO No, that’s simply not true. You can go to a novelty store and prove this to yourself. They sell toys in which an object is suspended between two magnets. The object just hovers in space. You can also rotate the object, and it will act as if its in zero gravity (except for the friction forces close to the surface of the earth which will cause it to slow down). For an additional $150, you can purchase a Jim-Dandy levitating world globe. It is a 9 inch diameter globe that levitates in mid air suspended between two electro-magnets. I like it because it gives a perfect picture of why Job 26:7 and Psalm 93:1 say that God “hangs the earth upon nothing” and that it “does not move.” You’re going to see the steel ball oscillate, left to right and right to left, indefinitely, because the unequal magnetic force on either side of it will constantly tug at it. ======================== AC Nope, you’re going to see it accelerate toward the stronger magnet and smash into it. In order to oscillate, when you move the ball closer to the 1,000 lb. magnet, the force of attraction from the 100 lb. magnet would have to be stronger than the force of attraction from the 1,000 lb. magnet, in order to pull the ball back toward the equilibrium point. Obviously, that’s not the case, and therefore, oscillation is not physically possible in this scenario. ======================== GEO I suggest that you go to Sharper Image and play with the toy. If you placed the steel ball just a little above the equilibrium point, then you would see it oscillate up and down, indefinitely. If on a diagonal, you would see it oscillate diagonally. This is precisely what is happening with the Geo satellites. ======================== AC I’ve lost track: Is this your third or your fourth different attempt to account for the motion of geosynchronous satellites? It seems that each attempt is getting more absurd than the one before. ======================== GEO I don’t know to which “third or fourth different attempts” you are referring. If you thinks I have given a different explanation, then I think you are required to say where I have done so, rather than make it appear as if I’ve been all over the map on this issue. I have never posited anything except the differential gravitational pull between the stars and the earth as the basis for my explanation. The only other thing I have offered is that the same gravitational forces could create an electro-magnetic field. If they are not placed precisely on the equator, but are inclined to one degree or another, then they will show some type of oscillation, indefinitely. They will oscillate with respect to the equator, since the equator is the balance point where all the forces cancel each other out. ======================== AC Here’s another one of those unfounded assertions preceded by the word “since.” ======================== GEO See my previous answer on this question. That is why all the diagrams you show have the center point of the oscillation precisely on the equator. ======================== AC No, the reason these satellites move an equal distance above and below the equator is because they are orbiting the earth in nearly circular orbits. As you can see from the above “inclination” figure, a satellite in such an orbit will always move equal distances above and below the equator. It has nothing to do with gravitational forces from the stars. ======================== GEO Mere assertion is not going to prove anything. The question you posed last was how I could explain the up and down motion of Marisat in MY system. To deny the answer, you would have to show that my system doesn’t work, not that his works. My claim from the start is that both models work. The reason they are oscillating is because of the differential gravity between the stars and the earth. ======================== AC Even if that explanation made sense, which it doesn’t, it still doesn’t account for the zigzagging satellites. Why do they move in a slow and steady zigzag while the others don’t? ======================== GEO I already explained this in the last post. The circumference of the loops is too small to show up on the graph, so they result in zig-zags. You’ve pointed out that the satellites have either a zig-zag or figure-eight pattern. Why are they different? Since the satellite is placed in an elliptical position, not a circular position, with respect to the earth, this means that the satellite cannot maintain its position unless it moves faster or slower against the inertial frame. ======================== AC Wait a sec, I thought these satellites weren’t orbiting. What’s this talk about “elliptical” and “circular”? These satellites are supposedly just levitating up there, remember? ======================== GEO I said they are in an elliptical position in respect to the inertial frame, which is the gravitational force between the stars and earth. The gravitational force causes what in your system appears as an elliptical orbit around the earth, but what in my system is an oscillation back and forth between the earth and the stars. Thus, when the satellite is closest to the earth (the perigee) the satellite would be faster with respect to the inertial field (and from your perspective it would rotate around the earth in less than 24 hours); and when the satellite is farthest away from the earth (the apogee) the satellite would be moving slower with respect to the inertial field (or from your perspective it would take more than 24 hours to rotate around the earth). ======================== AC Again, that’s only true if the satellite is in orbit. But you say it’s not orbiting, and therefore, you can’t appeal to the velocity differences at the perigee and apogee of an elliptical orbit. You need to come up with an explanation that makes sense for an object that’s levitating, not orbiting. ======================== GEO It is not just levitating, it is oscillating across a levitation point. This movement is going to create various patterns. The closer the semi-major axis of the orbit lies along the apogeeperigee line, the wider the figure-eight pattern will appear, since it will be oscillating with greater force. ======================== AC What orbit? If these satellites are orbiting, then the earth is rotating. Are you ready to admit that they’re really orbiting? ======================== GEO No, I’m using “orbit” to accommodate your system, not agree with it. When the semimajor axis lies along the equator, you will get neither a figure-eight nor a zig-zag, but you will get a slight north-south oscillation. You will get a zig-zag pattern when the semi-major axis is between the apogee-perigee line and the line of the equator. Or, to put it another way, when the period of the ellipses is slightly more or slightly less than 24 hours, you will get a zig-zag pattern. ======================== AC Yep, if the satellite is orbiting. ======================== GEO Already explained above. In effect, the figure-eights and the zig-zags are being produced from the same effect, except the diameter of some of the loops is too small to show up in the computer graph, so they turn out as zig-zags rather than the larger figure-eights. ======================== AC That’s exactly right. In my world. Are you conceding that these satellites are orbiting in elliptical orbits having a period of approximately 24 hours, and that the earth is rotating beneath them? ======================== GEO Already answered above. Now, let me add one more dimension to this picture. Against what background is the figure-eight pattern or zig-zag pattern measured? In your universe it is measured against a rotating earth. In the Geocentric universe it is measured against a rotating star shell. ======================== AC No, they move in figure-eights and zigzags relative to the surface of the earth, whether the earth is rotating or not. It’s just that if the earth’s not rotating, there’s no way to account for that motion, as you have amply demonstrated. ======================== GEO No, I have amply demonstrated that the satellites can oscillate between the earth and stars, and you have shown no material evidence against that possibility. In fact, you have denied the physics that an object can remain at equilibrium between two magnetic forces, and you have denied that they can oscillate. I suggest you purchase the levitating globe and you will see that principle of physics readily at work. Let me explain: If the earth is stationary, the stars are rotating around the earth every 24 hours. They will come back to precisely the same point each 24 hour period (allowing a little movement for precession, which I don’t want to get into right now). We can see the satellites move against the background of the stars. On the first night we will see Marisat in a certain position against a particular star. But 24 hours later, we will see that Marisat did a figure-eight against the background of the star. (In case you’re wondering, it makes no difference whether the star rotates around the earth, because every 24 hours the star will always come back to the same position in the sky). ======================== AC If we were at the equator, and if we were able to train our eyes on Marisat-3 for 24 hours, we’d see it move around our sky in a figure-eight pattern. That’s its motion relative to us. It has nothing to do with the stars. If we were somehow able to block our view of the stars and look only at Marisat-3, we’d still see it moving in a figure-eight pattern. ======================== GEO It has become apparent to me that you either don’t understand or won’t accept that a force outside the earth can affect an object placed above the earth. This is because you fail to accept the Machian principle that there is no difference between a rotating earth and a rotating star system, relative to US. Because of this, I believe this series of dialogues cannot progress any further, as I have stated previously. Is Marisat showing us real movement? Yes, it is real. Marisat is actually oscillating due to the fact that it is in a 10 degree inclination with respect to the earth’s equator. Being in that inclination, Marisat will experience opposing gravitational forces from the earth-star gravity field, and thus it must oscillate back and forth between the equilibrium point, which is at the equator. ======================== AC Let us say, for the sake of argument, that these “opposing gravitational forces” were able to account for the north-south oscillation of Marisat-3. What accounts for its daily east-west oscillation? There can be no imbalance of forces parallel to the direction of rotation of the stars, so what makes Marisat-3 oscillate east-and-west? And what makes Brasilsat-1 move in a steady westerly direction? ======================== GEO If it’s on an incline with respect to the equator, it will go east-west or northeastsouthwest, or any combination of two compass opposite compass points. But what does Marisat look like from your universe? Since for you the earth is rotating, then you cannot attribute the movement to an oscillation of the satellite itself, but to the rotation of the earth against the inclination of the satellite. That is why you say that the figure-eights are only there by “appearance.” ======================== AC Yes, that way I don’t have to appeal to imaginary forces that control the movement of artificial satellites, but don’t effect the moon or the sun or the planets, all of which are allegedly able to orbit the earth every day, despite being well beyond the point of equilibrium between the gravitational forces of the stars and the earth. If your explanation were correct, every object beyond the equilibrium point where the geosynchronous satellites orbit – excuse me, levitate – should go flying off toward the stronger gravitational pull from the rotating shell of stars. ======================== GEO Again, this just shows your inability to address the issue because you don’t know the physics. Your understanding about opposing gravitational and magnetic fields is wrong. Moreover, you do not try to answer my argument by showing that a gravitational field from the stars is impossible to happen. Rather, you resort to calling them “imaginary forces.” But I invite you to read the Lense-Thirring paper, and you will see that, even from your own science, they are not imaginary forces. And in my science, they are very real, even though we don’t use Relativity theory to prove they exist. You have no answer for this, except to stoop to ridicule, simply because you are not familiar with the science I am presenting to you. But does that prove the earth rotates? Not by any means. Since, as I have shown above, the same satellite movements can be explained from the perspective of a rotating star shell as much as they can be explained from a rotating earth. Thus, using the “movements” of the Geo satellites proves nothing for anyone. All it proves is the same thing I’ve been saying all along – there is no proof for a rotating earth as opposed to a rotating star shell. ======================== AC And I’ll say what I’ve said all along. your latest attempt to account for the motions of these satellites is no more plausible than any of your previous attempts. In each case you have to appeal to relativistic forces, which he otherwise says are “an absolute farce,” and you have to appeal to orbital mechanics even though you claim these objects aren’t orbiting. What you have not been able to do is come up with a credible explanation for this motion without borrowing from sources you claim not to believe. ======================== GEO Already answered. What I have in my favor is that because of the Lense-Thirring principle, which was supported by Einstein himself (a position which I detailed earlier). ======================== AC In which case you can’t use it, because you say Einstein’s relativity theories, from which the Lense-Thirring effect was derived, are “an absolute farce.” You can’t have it both ways. ======================== GEO Again, I don’t use Lense-Thirring to explain my universe, but you need it to explain yours. If you don’t accept it, then Relativity goes out the window and you are forced to explain everything from Newtonian mechanics, which can’t be done. Then I have a viable PHYSICAL explanation for the forces that would cause Marisat to oscillate in the figure-eight pattern, and therefore my assertion that no one can disprove that system still stands; and which also means that no one can prove that a rotating earth is the only explanation. ======================== AC I have proved that your latest explanation is no more viable than the others. I’m eager to see what you try next. ======================== GEO Obviously, you haven’t proved anything. You’ve just made assertions without knowing the physics behind what you claim. ======================== More on the ways of Chandra AC Yes, but those lines represent Chandra’s actual flight-path relative to the ground. ======================== GEO You still don’t seem to understand what I’ve been saying. I do not disagree with you that the lines “represent” Chandra’s actual flight-path. I’ve said over and over again that a person could use those lines to know where Chandra is located relative to the earth. But here’s what you are either ignoring, denying or just simply refuse to accept: It is Chandra’s actual flight-path relative to a ROTATING ground, not a STATIONARY ground. If the earth were STATIONARY the lines would be different. That is a fact. Thus, the lines do not prove the earth is rotating. They only prove what the flight path looks like assuming that the earth is rotating. Unless you deal with that point, this dialogue can go no further. ======================== AC You still don’t seem to get what’s happening here. That meandering line is Chandra’s actual flight-path over the ground. That’s where you have to aim your telescope if you want to see the satellite. If that line predicts that Chandra will pass over eastern Nevada, on a north-south heading, that means that an astronomer in eastern Nevada would be able to point his telescope straight up at the appointed time and see the satellite. If that line didn’t represent Chandra’s actual flight-path over the ground, astronomers wouldn’t be able to use that ground-track data to aim their telescopes. ======================== GEO The only reason he can point his telescope up at the appointed time is because, according to the computer-generated chart of Chandra’s flight path (which is based on a rotating earth), the computer will tell him to look at a certain place in the sky over Nevada at a certain time. But if you told the computer that the earth was not rotating and asked it to compute when Chandra was going to be over at Nevada, the computer would give you the exact same time as it would in the rotating earth calculation. That is a fact. ======================== AC Now, it being the case that this is Chandra’s actual flight-path, our two respective systems must explain how a satellite can pass over the surface of the earth in that squiggly, loopy pattern. In my system, the trajectory is easily explained as the superimposition of an elliptical orbit over a rotating earth. But in your system, where the earth doesn’t move, only the satellite moves. Therefore, the satellite must somehow steer itself along that wild, squiggly, loopy course. How does it do that? ======================== GEO If you then tell the computer that the earth is stationary and ask it to tell you Chandra’s whereabouts with respect to a specific spot on earth, it will do so, but it will make a chart on the graph which is different than the squiggly lines in your original graph. ======================== AC Right, and if you then aim your telescope where this new graph predicts the satellite will be, it won’t be there. ======================== GEO You’ve just admitted that your objection is a mere assertion, since you provided no evidence to back it up. You’ve never asked NASA to provide you with a graph of Chandra’s flight path based on a non-rotating earth, have you? So you have no way of knowing what this graph would show. But I think your intuition is bright enough to know that the non-rotating earth graph will not have the squiggly lines you showed us in a rotating earth. Both graphs are true records of the systems they represent, but neither graph proves that the system they represent is the actual system operating. ======================== AC These aren’t graphs of cosmological systems, they’re graphs of the actual flight-path of a satellite relative to the ground. That satellite either passes over the specific places NASA’s graph says it will, or it doesn’t. Under no circumstances, though, can two different ground-tracks both be “true.” One represents where the satellite will pass over the ground; the other does not. ======================== GEO Two ground-tracks can both be right, since one is from a rotating earth and the other from a non-rotating earth. You can prove this for yourself. Hang a can of paint from the ceiling, with a pinhole in the bottom, and swing the paint-can in a precise elliptical orbit. Beneath the can of paint, place a basketball. On your first try, let the paint stream from the can onto the basketball without turning the basketball. You’re going to see a simple elliptical paint line made on the basketball. On your second try, put the paint-can in the same precise elliptical orbit and let the paint stream from the can onto the basketball while you’re turning the basketball. You’re not going to see a simple elliptical paint line. You’re going to see that line go all over the basketball, back and forth, and side to side. But neither the paint can nor the basketball has moved its position relative to one another. The only thing that has changed is that the basketball was rotated in one scenario but not the other. Chandra does not have a big pencil that reaches to the earth and by which it marks out a trajectory. ======================== AC But if it did, it would mark out exactly the trajectory shown on NASA’s maps. Again, the fact that astronomers use this same trajectory data to aim their telescopes proves that Chandra is right where NASA says it will be relative to the surface of the earth. ======================== GEO As the above example shows, it would only mark out that specific trajectory on a rotating earth, not a stationary earth. It only proves that the system they are using will account for Chandra’s whereabouts with respect to a rotating earth, but it does not prove that the system is a reality, that is, that the earth is rotating. ======================== AC Then please tell me how a satellite can follow that squiggly, loopy trajectory over the ground if the earth isn’t rotating. You keep avoiding that question. ======================== GEO No, I don’t avoid the question. I’ve answered it many times. Here’s the rub. It doesn’t follow a squiggly loopy trajectory in a stationary earth. Until you ask NASA to show you a graph of Chandra’s trajectory in a stationary earth, then you really have no basis for making the above statement. Once again, the image we see in NASA’s pictorial is exactly what one would expect to see from a rotating earth. But that doesn’t prove that the earth is rotating. All it proves is that if one assumes a rotating earth, the trajectory will follow the path depicted in the pictorial. ======================== AC No, the satellite follows that trajectory whether the earth rotates or not. ======================== GEO No, you haven’t shown that. You haven’t asked NASA to produce a pictorial of Chandra’s lines assuming a non-rotating earth. NASA could do so if they wished. ======================== AC Again, the lines I showed represent Chandra’s actual flight-path over the ground. You yourself acknowledged this when you said that this chart showed that “Chandra is going to be over a certain place at a certain time.” Exactly! Now, how do you account for that flight-path if the earth is stationary? ======================== GEO Since you haven’t proven that the flight path would be the same in a stationary earth (because you haven’t asked NASA to provide you with one), then your objection above is baseless. And as I will show below by using your own pictorials, you’ve actually disproven your own claims. More on satellites AC The “Lense-Thirring effect” is an effect predicted by general relativity. You don’t believe in general relativity. Therefore, the Lense-Thirring effect is not available to you as an explanation for how satellites move the way they do in a non-relativistic, geocentric world. You can’t have it both ways. He can’t reject relativity and then appeal to it as the explanation of how satellites move. ======================== GEO I can, because one of the more convincing ways to cast doubt on your opponent’s scientific presuppositions is to use his own science against him. ======================== AC But that’s not what you’re doing. You’re not using Relativity against me, you’re using it for yourself and offering it as the actual reason that geosynchronous satellites levitate above the earth without falling. I don’t appeal to Relativity to explain how geosynchronous satellites work. I say they’re simply in orbit around a rotating earth. If you think they’re really levitating in space, you need to explain how they do that, and if you can’t do that without appealing to theories you otherwise reject, then all you’ve apparently proved is that geocentrism can’t work without relativity. Let me quote you: Lense-Thirring works in either a Relativity framework or a non-Relativity framework. So does Mach’s principle. That’s what stunned Einstein, and it was why he was forced to agree with it. That’s why we use it. It bodes well for us, but not for you, since the effect of Lense-Thirring shows that there is more than one explanation for gravity, centrifugal and Coriolis forces on the earth. I think the problem is that you’ve never looked into either Lense-Thirring or Mach, and thus, regarding the physics, you are shooting from the hip in most of this discussion. ======================== GEO Since the equilibrium of gravitational forces lies only on the equator, then any satellite which is inclined to any degree against the equator is going to produce some type of oscillation. ======================== AC What makes you think there’s an “equilibrium of gravitational forces” that “lies only on the equator”? I think a lawyer would say this answer “assumes facts not in evidence.” You need to substantiate claims like this, not just assert them as conclusions following words like “since” and “because.” ======================== GEO Second, I am again using the scientific evidence that you use in your system, and upon which gravitational mechanics works, that is, that there is a neutral point of gravity and the centrifugal effect at the equator. ======================== AC My system denies that there’s a “neutral point of gravity” at the equator. That’s your allegation, not mine. Please substantiate it. And since we’ve agreed that Relativity is a farce, Lense-Thirring is out. ======================== More on Coriolis and the pendulum GEO As noted above, Lense-Thirring is not out. I would suggest you read up on Lense- Thirring before you so presumptuously dismiss it. Second, the Newtonian system (unless you want to throw that out too) agrees that the Foucault pendulum will not operate at the equator. Thus, call it what you will (I call it a “neutral point of gravity” which may not be the best term) but a rose by any other name is still a rose. There is no Coriolis effect at the equator, but there is one above and below it. That means something. And it is upon that basis I make my statements. What is your explanation for it? How would this occur in a Geocentric universe in which the satellite is not orbiting the earth but is merely hovering about the earth, moving only slightly? Let me make an analogy. Let’s say you are in a room. On one side of the room there is a 1000 lb electro-magnet. On the other side there is a 100 lb electro-magnet. Both magnets are turned on and operating. There is a force from each magnet. Somewhere in the room, closer to the 100 lb magnet, there is an equilibrium of counter acting magnetic forces, such that if a metal object were placed in that equilibrium, the metal object would remain suspended within the magnetic force. (There are plenty of experiments that do this very thing). To test this out, you place a steel ball in the equilibrium position between the two magnets. You see that the ball floats in mid air, suspended by the force of both magnets. ======================== AC Okay, no problem so far. ======================== GEO Now, before I go any further, if you haven’t figured it out already, the 1000 pound magnet represents the force of gravity from the stars. The 100 pound magnet the force of gravity from the earth. ======================== AC If you are planning to argue that geosynchronous satellites are floating in an equilibrium position between the earth (the 100 lb. magnet) and the stars (the 1,000 lb. magnet), you’re going to have to explain why the moon, which is way past this “equilibrium point,” doesn’t go zipping off toward the stars. How can it orbit the earth if it’s hundreds of thousands of miles on the stellar side of this alleged “equilibrium point”? ======================== GEO Because the moon is moving, which causes additional centrifugal effects, that a geosynchronous satellite does not have. Which tends to pull the moon away from the earth, not toward it. If the moon went fast enough, it would reach escape velocity, and then it really would go zipping off toward the stars. So any motion on the moon’s part only paints you into a tighter corner. I ask you again, then, because the moon is way past the “equilibrium point” between the earth and the stars, and because it’s moving, why doesn’t it go zipping off toward the stars? The moon, because of its size, also has a gravitational effect against the earth’s gravitational effect. I’ve already explained this. In the LaSagean system, the two bodies interact with each other as they disturb the aether around them. The geosynchronous satellites don’t have any gravity for us to contend with, nor do they move, therefore, their equilibrium point with the earth is going to be far different than the moon’s. The steel balls represent the satellites. And one point of clarification before I proceed, the force of the stars, according to the Lense-Thirring Effect, is created by the billions of stars all acting upon the earth at the same time. These stars, as they rotate in their shell, produce large gravitational effects, according to the Lense-Thirring principle, supported by Einstein himself. ======================== AC But you say Einstein’s theories are “an absolute farce.” Therefore, you can’t appeal to them to explain how things move in your non-relativistic, geocentric world. ======================== GEO See my previous answer. ======================== AC Well, now that we’ve both agreed that Relativity is bunk, why don’t you tell us what force really accounts for this motion? ======================== GEO First, I’m glad to hear that you think Relativity is “bunk.” You’re halfway home. Second, neither gravity nor magnetism is dependent on Einstein’s theory of Relativity. They work perfectly fine without Einstein. Thus, we don’t need Einstein to tell us that apples fall from trees or that GSS can hover above the earth. The only thing for which we need Einstein is to use his own theories to show that the forces of the stars can account for the GSS, just as well as any earth-centered explanation, and he must agree. This immense amount of stars makes up for the fact that the stars are far away. I say this to counter the idea that Gary was perpetuating that the stars have a negligible force on the earth. ======================== AC They’re still surrounding us on all sides. Whether the force is strong or weak, the net effect is zero. ======================== GEO If you were at the very center of the earth the forces would cancel each other out, for then you would be equi-distant from all the stars. But the fact is that, anywhere on the surface of the earth, you are 4,000 miles closer to one side of the star system than the other, since the diameter of the earth is about 8000 miles. ======================== AC Given the immense distance from the stars, 4,000 miles one way or the other is negligible. ======================== GEO You don’t know it’s “negligible.” Also, you don’t know how far away the stars are, nor do you know what kind of force they exert. You apparently have never studied the physics in Lense-Thirring’s papers. In fact, those papers show that a 4,000 mile difference would present the same effect of gravity upon an object as the gravitational formulae in the Newtonian system, which are based on the inverse square of the distance between objects. Now back to the analogy. You place a steel ball in the exact place where the equilibrium of the magnets exists. What do you see? The steel ball remains motionless. But on your second attempt, you place the steel ball just a little left or right of the equilibrium point. What are you going to see? ======================== AC You’re going to see the ball accelerate toward the stronger magnet and smash into it. Once you move away from the equilibrium point, you move into an area where one magnet is stronger than the other. Therefore, the net force on the ball is going to pull it toward that magnet. The equilibrium point between the two magnets, the force of attraction on the ball, is balanced, and therefore, as you say, the ball isn’t going to move. But if you move the ball closer to the 1,000 lb. magnet the force of attraction from the 1,000 lb. magnet is going to be stronger than the force of attraction from the 100 lb. magnet, and the ball will accelerate toward the 1,000 lb. magnet. The same thing will happen, of course, if you move the ball closer to the 100 lb. magnet. Those who believe there is empirical proof that earth rotates &-or orbits are asked to give it: an open discussion 140 ======================== GEO No, that’s not true. You can go to a novelty store and prove this to yourself. They sell toys in which an object is suspended between two magnets. ======================== AC Those magnets repel from each other. Are you claiming that the earth repels satellites? ======================== GEO No, I’m saying that magnetic forces put in the right position can cause a body to levitate in mid-air. Since magnetic forces are analogous to gravitational forces, then the model fits. The object just hovers in space. You can also rotate the object, and it will act as if its in zero gravity (except for the friction forces close to the surface of the earth which will cause it to slow down). For an additional $150, you can purchase a Jim-Dandy levitating world globe. It is a 9 inch diameter globe that levitates in mid air suspended between to electro-magnets. I like it because it gives a perfect picture of why Job 26:7 and Psalm 93:1 say that God “hangs the earth upon nothing” and that it “does not move.” ======================== AC So, the earth is like a giant magnet that repels everything away from it, and yet somehow the whole universe is attracted to it and rotates around it every day? If you’re right that there’s an equilibrium point between the repulsive forces of the earth and the stars, why aren’t low-flying satellites pushed away by the stronger force from the earth until they reach this equilibrium point? ======================== GEO I never said the earth repels. That’s your statement. I said there is an equilibrium in the force of the stars with the force of the earth, analogous to (not the same as) a globe floating between two electro-magnets. You’re going to see the steel ball oscillate, left to right and right to left, indefinitely, because the unequal magnetic force on either side of it will constantly tug at it. ======================== AC Nope, you’re going to see it accelerate toward the stronger magnet and smash into it. In order to oscillate, when you move the ball closer to the 1,000 lb. magnet, the force of attraction from the 100 lb. magnet would have to be stronger than the force of attraction from the 1,000 lb. magnet, in order to pull the ball back toward the equilibrium point. Obviously, that’s not the case, and therefore, oscillation is not physically possible in this scenario. ======================== GEO I suggest you go to Sharper Image and play with the toy. ======================== AC Oh, I don’t deny that you can suspend an object between two magnets that are both repelling that object. But if you want to propose that as an explanation of how geosynchronous satellites don’t fall, you’ll open up quite a can of worms for yourself. Let me know if that’s the direction you really want to go, and I’ll give you a list of phenomena you simply can’t explain. In fact, geosynchronous satellites would probably be the only thing you could explain in such a system. ======================== GEO “Can of worms”? “List of phenomena”? I’m open for anything thing you have. If you placed the steel ball just a little above the equilibrium point, then you would see it oscillate up and down, indefinitely. If on a diagonal, you would see it oscillate diagonally. This is precisely what is happening with the Geo satellites. ======================== AC I’ve lost track: Is this your third or his fourth different attempt to account for the motion of geosynchronous satellites? It seems that each attempt is getting more absurd than the one before. ======================== GEO I don’t know what “third or fourth different attempts” you are referring to. If you think I have given a different explanation, then I think you are required to show where I have done so, rather than make it appear as if I’ve been all over the map on this issue. I have never posited anything except the differential gravitational pull between the stars and the earth as the basis for my explanation. ======================== AC Is it a “pull” or a “push” ? ======================== GEO If I sometimes use “pull,” it’s to accommodate you, so you won’t get confused with the “push” alternatives in an opposing system. If they are not placed precisely on the equator, but are inclined to one degree or another, then they will show some type of oscillation, indefinitely. They will oscillate with respect to the equator, since the equator is the balance point where all the forces cancel each other out. That is why all the diagrams you show have the center point of the oscillation precisely on the equator. ========================
×